SlideShare a Scribd company logo
Hỗ trợ trực tuyến
Fb www.facebook.com/DayKemQuyNhon
Mobi/Zalo 0905779594
Tài liệu chuẩn tham khảo
Phát triển kênh bởi
Ths Nguyễn Thanh Tú
Đơn vị tài trợ / phát hành / chia sẻ học thuật :
Nguyen Thanh Tu Group
Đ Ề T H I T H Ử V À O L Ớ P 1 0
T H P T M Ô N T O Á N
Ths Nguyễn Thanh Tú
eBook Collection
TUYỂN TẬP ĐỀ THI TOÁN VÀO LỚP 10
CHUYÊN KHOA HỌC TỰ NHIÊN HÀ NỘI
NĂM 2022 CÓ LỜI GIẢI CHI TIẾT
WORD VERSION | 2023 EDITION
ORDER NOW / CHUYỂN GIAO QUA EMAIL
TAILIEUCHUANTHAMKHAO@GMAIL.COM
vectorstock.com/28062405
Website:
Liên hệ tài liệu word môn toán: TÀI LIỆU TOÁN HỌC
TUYỂN TẬP ĐỀ THI TOÁN
VÀO LỚP 10 CHUYÊN KHOA HỌC TỰ NHIÊN HÀ NỘI
LỜI NÓI ĐẦU
Nhằm đáp ứng nhu cầu về của giáo viên toán THCS và học sinh luyện thi vào lớp 10 môn toán,
website giới thiệu đến thầy cô và các em bộ đề thi vào lớp 10 chuyên Đại học khoa học
tự nhiên Hà Nội. Đây là bộ đề thi mang tính chất thực tiễn cao, giúp các thầy cô và các em học sinh luyện
thi vào lớp 10 có một tài liệu bám sát đề thi để đạt được thành tích cao, mang lại vinh dự cho bản thân, gia
đình và nhà trường. Bộ đề gồm nhiều Câu toán hay được các thầy cô trên cả nước sưu tầm và sáng tác, ôn
luyện qua sẽ giúp các em phát triển tư duy môn toán từ đó thêm yêu thích và học giỏi môn học này, tạo
được nền tảng để có những kiến thức nền tốt đáp ứng cho việc tiếp nhận kiến thức ở các lớp, cấp học trên
được nhẹ nhàng và hiệu quả hơn.
Các vị phụ huynh và các thầy cô dạy toán có thể dùng có thể dùng tuyển tập đề toán này để giúp
con em mình học tập. Hy vọng Tuyển tập đề thi toán vào lớp 10 chuyên Đại học khoa học tự nhiên này sẽ
có thể giúp ích nhiều cho học sinh phát huy nội lực giải toán nói riêng và học toán nói chung.
Mặc dù đã có sự đầu tư lớn về thời gian, trí tuệ song không thể tránh khỏi những hạn chế, sai sót.
Mong được sự góp ý của các thầy, cô giáo và các em học!
Chúc các thầy, cô giáo và các em học sinh thu được kết quả cao nhất từ bộ đề này!
Website:
Liên hệ tài liệu word môn toán: TÀI LIỆU TOÁN HỌC
MỤC LỤC
PHẦN 1: ĐỀ THI
Đề số Đề thi Trang
1. Đề thi vào lớp 10 chuyên Đại học KHTN Hà Nội năm 2022 (vòng 1)
2. Đề thi vào lớp 10 chuyên Đại học KHTN Hà Nội năm 2022 (vòng 2)
3. Đề thi vào lớp 10 chuyên Đại học KHTN Hà Nội năm 2021 (vòng 1)
4. Đề thi vào lớp 10 chuyên Đại học KHTN Hà Nội năm 2021 (vòng 2)
5. Đề thi vào lớp 10 chuyên Đại học KHTN Hà Nội năm 2020 (vòng 1)
6. Đề thi vào lớp 10 chuyên Đại học KHTN Hà Nội năm 2020 (vòng 2)
7. Đề thi vào lớp 10 chuyên Đại học KHTN Hà Nội năm 2019 (vòng 1)
8. Đề thi vào lớp 10 chuyên Đại học KHTN Hà Nội năm 2019 (vòng 2)
9. Đề thi vào lớp 10 chuyên Đại học KHTN Hà Nội năm 2018 (vòng 1)
10. Đề thi vào lớp 10 chuyên Đại học KHTN Hà Nội năm 2018 (vòng 2)
11. Đề thi vào lớp 10 chuyên Đại học KHTN Hà Nội năm 2017 (vòng 1)
12. Đề thi vào lớp 10 chuyên Đại học KHTN Hà Nội năm 2017 (vòng 2)
13. Đề thi vào lớp 10 chuyên Đại học KHTN Hà Nội năm 2016 (vòng 1)
14. Đề thi vào lớp 10 chuyên Đại học KHTN Hà Nội năm 2016 (vòng 2)
15. Đề thi vào lớp 10 chuyên Đại học KHTN Hà Nội năm 2015 (vòng 1)
16. Đề thi vào lớp 10 chuyên Đại học KHTN Hà Nội năm 2015 (vòng 2)
17. Đề thi vào lớp 10 chuyên Đại học KHTN Hà Nội năm 2014 (vòng 1)
18. Đề thi vào lớp 10 chuyên Đại học KHTN Hà Nội năm 2014 (vòng 2)
19. Đề thi vào lớp 10 chuyên Đại học KHTN Hà Nội năm 2013 (vòng 1)
20. Đề thi vào lớp 10 chuyên Đại học KHTN Hà Nội năm 2013 (vòng 2)
21. Đề thi vào lớp 10 chuyên Đại học KHTN Hà Nội năm 2012 (vòng 1)
22. Đề thi vào lớp 10 chuyên Đại học KHTN Hà Nội năm 2012 (vòng 2)
23. Đề thi vào lớp 10 chuyên Đại học KHTN Hà Nội năm 2011 (vòng 1)
24. Đề thi vào lớp 10 chuyên Đại học KHTN Hà Nội năm 2011 (vòng 2)
25. Đề thi vào lớp 10 chuyên Đại học KHTN Hà Nội năm 2010 (vòng 1)
26. Đề thi vào lớp 10 chuyên Đại học KHTN Hà Nội năm 2010 (vòng 2)
27. Đề thi vào lớp 10 chuyên Đại học KHTN Hà Nội năm 2009 (vòng 1)
28. Đề thi vào lớp 10 chuyên Đại học KHTN Hà Nội năm 2009 (vòng 2)
PHẦN 2: HƯỚNG DẪN GIẢI
Website:
Liên hệ tài liệu word toán zalo: TÀI LIỆU TOÁN HỌC
1
LỜI GIẢI ĐỀ THI TUYỂN SINH VÀO LỚP 10
NĂM HỌC: 2023 – 2024
Trường THPT Chuyên Khoa học tự nhiên
Môn thi : Toán (vòng 1)
Thời gian làm bài: 120 phút (không kể thời gian phát đề)
Câu I. (3,5 điểm)
1. Giải phương trình 2 2
6 2023 3 5 2025 5.
+ + + + = + + +
x x x x x
2. Giải hệ phương trình
( )( )
3 3 2 2
6 3 2 12
2 6 15 19 6 12.
 + + =

+ + + + + =

x y x y
x y x y y x x y
Câu II. (2,5 điểm)
1. Giả sử n là số nguyên sao cho 3
3 1011
−
n chia hết cho 1008 . Chứng minh rằng 1
−
n chia
hết cho 48 .
2. Với , ,
a b c là các số dương thỏa mãn điều kiện 1
+ + =
ab bc ca . Chứng minh rằng
2 2 2
1 1 1
1 1 1 4
1 1 1
   
+ + + >
   
+ + +
   
a b c
Câu III. (3 điểm) Cho hai đường tròn ( )
O và ( )
′
O cố định cắt nhau tại A và B sao cho O nằm
ngoài ( )
′
O và ′
O nằm ngoài ( )
O . Trên đường tròn ( )
O lấy điểm P di chuyển sao cho P nằm trong
đường tròn ( )
′
O . Đường thẳng AP cắt ( )
′
O tại C khác A .
1. Chứng minh rằng hai tam giác OBP và ′
O BC đồng dạng.
2. Gọi Q là giao điểm của hai đường thẳng OP và ′
O C . Chứng minh rằng   90
+ =

QBC ABP
.
3. Lấy điểm D thuộc ( )
O sao cho AD vuông góc ′
O C . Chứng minh rằng trung điểm của
đoạn thẳng DQ luôn nằm trên một đường tròn cố định khi P thay đổi.
Câu IV. (1 điểm)
Giả sử A là tập hợp con của tập hợp gồm 30 số tự nhiên đầu tiên { }
0,1,2,3, ,29
… sao cho với k
nguyên bất kỳ, , ∈
a b A bất kỳ (có thể =
a b ) thì 30
+ +
a b k không là tích của hai số nguyên liên
tiếp. Chứng minh rằng số phần tử của tập hợp A nhỏ hơn hoặc bằng 10.
Website:
Liên hệ tài liệu word toán zalo: TÀI LIỆU TOÁN HỌC
2
2. PHẦN LỜI GIẢI
Câu 1: (3,5 diểm)
1. Giải phương trình
2 2
6 2023 3 5 2025 5.
+ + + + = + + +
x x x x x
2. Giải hệ phương trình
( )( )
3 3 2 2
6 3 2 12
2 6 15 19 6 12.
 + + =

+ + + + + =

x y x y
x y x y y x x y
Lời giải.
1. Điều kiện xác định: 3
≥ −
x .
Cách 1. Xét 2
>
x thì vì ( ) ( )
2 2
6 2023 5 2025 2 0
+ + − + + = − >
x x x x x và ( )
3 5
+ − =
x 2 0
− >
x
nên suy ra
2 2
6 2023 3 5 2025 5.
+ + + + ≥ + + +
x x x x x
Tương tự nếu 3 2
− ≤ <
x thì
2 2
6 2023 3 5 2025 5.
+ + + + ≤ + + +
x x x x x
Do đó, ta phải có 2
=
x và nghiệm này thoả mãn điều kiện xác định. Vậy 2
=
x là nghiệm duy nhất
của phương trình
Cách 2. Phương trình đã cho có thể viết lại thành
2 2
6 2023 5 5 2025 3.
+ + − = + + − +
x x x x x
Giả sử hai vế cùng dấu, bình phương hai vế và rút gọn, ta được
( ) ( )( )
2 2
5 6 2023 5 2025 3 .
+ + = + + +
x x x x x
Bình phương một lần nữa, khai triển và rút gọn, ta được
( )( )
2
2 5 2020 0.
− + + =
x x x
Vì 2
5 2020 0
+ + =
x x vô nghiệm nên 2
=
x và nghiệm này thoả mãn.
Vậy, phương trình đã cho có nghiệm duy nhất 2
=
x .
2. Biến đổi phương trình thứ hai thành
( )( )
2 2
6 2 3 1 12.
+ + + + =
x y x xy y
Kết hợp với phương trình thứ nhất ta suy ra
( )( ) ( )( )
2 2
6 3 2 6 2 3 1 .
+ + = + + + +
x y x y x y x xy y
Mà ta thấy 6 0
+ ≠
x y nên chia cả hai vế phương trình trên cho 6
+
x y ta được
2 2
2 3 1 3 2 .
+ + + = +
x xy y x y
Biến đổi phương trình trên ta được
Website:
Liên hệ tài liệu word toán zalo: TÀI LIỆU TOÁN HỌC
3
( )( ) ( ) ( )
( )( )
2 2
0 2 3 3 2 1
2 2 1
2 1 1
= + + − − +
= + + − + − + +
= + − + −
x xy y x y
x y x y x y x y
x y x y
.
nên suy ra 2 1 0
+ − =
x y hoặc 1 0
+ − =
x y .
• Nếu 2 1 0
+ − =
x y thì ta có 1 2
= −
y x . Thay vào phương trình thứ nhất ta được
Giải phương trình ta thu được các nghiệm ( ) ( )
28 45
, 0,1 , ,
11 11
−
 
=  
 
x y .
• Nếu 1 0
+ − =
x y thì ta có 1
= −
y x . Thay voà phương trình thứ nhất ta được
( )( )
6 6 3 2 2 12.
+ − + − =
x x x x
Giải phương trình ta thu được các nghiệm ( ) ( )
4 9
, 0,1 , ,
5 5
−
 
=  
 
x y .
Vậy, hệ đã cho có ba nghiệm ( ) ( )
28 45 4 9
, 0,1 , , , ,
11 11 5 5
− −
   
=    
   
x y .
Câu 2: (2,5 điểm)
1. Giả sửn là số nguyễn sao cho 3
3 1011
−
n chia hết cho 1008. Chứng minh rằng 1
−
n chia hết
cho 48 .
2. Với , ,
a b c là các số dương thỏa mãn điều kiện 1
+ + =
ab bc ca . Chứng minh rằng
2 2 2
1 1 1
1 1 1 4.
1 1 1
   
+ + + >
   
+ + +
   
a b c
Lời giải.
1. Ta có các biến đổi sau 3 3
1008 3 1011 3 3 1008
− = − −
n n
∣ khi và chỉ khi 3
1008 3 3
− =
n
∣
( )
3
3 1
−
n tương đương với
( )( )
3 2 3
336 1 1 1 1
− = − + + = − + −
n n n n n n n
∣
Vì 336 chia hết cho 16 mà ( )
2
1 1 1
+ += + +
n n n n lẻ nên 1
−
n chia hết cho 16 .
Ngoài ra vì 336 chia hết cho 3 mà ( )( )
3
1 1
− = − +
n n n n n tích ba số nguyên liên tiếp chia hết cho 3
nên 1
−
n chia hết cho 3 .
Vì ( )
3,16 1
= nên 1
−
n chia hết cho 3 16 48
⋅ =. Phép chứng minh hoàn tất.
2. Ta cần chứng minh
2 2 2
1 1 1
1 1 1 4.
1 1 1
   
+ + + >
   
+ + +
   
a b c
Ta biến đổi tương đương bất đẳng thức (1) như sau.
Website:
Liên hệ tài liệu word toán zalo: TÀI LIỆU TOÁN HỌC
4
( )
( )( )( ) ( )( )( )
( )
( )
( )
( )
( )
( )
2 2 2
2 2 2
2 2 2 2 2 2
2 2 2 2 2 2 2
2 2
2
2
2 2 2
1 4
1 1 1
2 2 2 4 1 1 1
3( ) 2 4
3( ) 2 ( ) 2 4
3( ) 2 1 2 4
3( ) 4 2.
   
+ + +
⇔ >
   
+ + +
   
⇔ + + + > + + +
⇔ + + + <
⇔ + + + − + + <
⇔ + − + + <
⇔ − + + <
a b c
a b c
a b c a b c
abc a b b c c a
abc ab bc ac abc a b c
abc abc a b c
abc abc a b c
Theo bất đẳng thức −
AM GM và giả thiết ta có 3 2 2 2
1 3
= + + ≥
ab bc ac a b c kéo theo
2 1
3( ) 2
9
≤ <
abc . Suy ra bất đẳng thức ( )
2 đúng và bài toán được chứng minh.
Câu 3: (3 điểm)
Cho hai đường tròn ( )
O và ( )
′
O cắt nhau tại A và B sao cho O nằm ngoài ( )
′
O và ′
O nằm
ngoài ( )
O . Trên đường tròn ( )
O lấy điểm P di chuyển sao cho P nằm trong đường tròn ( )
′
O .
Đường thẳng AP cắt ( )
′
O tại C khác A .
1. Chứng minh rằng hai tam giác OBP và ′
O BC dồng dạng.
2. Gọi Q là giao điểm của hai đường thẳng OP và ′
O C . Chứng minh rằng
  90
+ =

QBC ABP
3. Lấy điểm D thuộc ( )
O sao cho AD vuông góc với ′
O C . Chứng minh rằng trung điểm của
đoạn thẳng DQ luôn nằm trên một đường tròn cố định khi P thay đổi.
Lời giải.
1. Ta có    
2 2 ′
= = =
BOP BAP BAC BO C . Kết hợp với 1
=
′
′
=
OB O B
OP O C
, ta thu được ∼
OBP
′
O BC .
Website:
Liên hệ tài liệu word toán zalo: TÀI LIỆU TOÁN HỌC
5
2. Vì ∼ ′
 
OBP O BC nên ta được  
′
=
OPB O CB , từ đó kéo theo tứ giác BCQP nội tiếp. Do
đó,   
= =
QBC QPC OPA . Hơn nữa, vì OAP cân tại O nên ta được  
1
90
2
+ =

AOP OPA .
Như vậy,
   
1
90
2
+ = + = 
QBC ABP OPA AOP
3. Gọi M là trung điểm của DQ . Đặt ′
∩ =
AD O C E . Vì các tứ giác BCQP và ADBP nội
tiếp nên ta được    
= = =
BQC BPC ADB EDB, vì thế tứ giác BDEQ nội tiếp đường tròn
đường
kính DQ (với tâm đường tròn là M ). Do đó, ta được
   
1
2
= = =
BMO DMB DEB AEB
Mặt khác, ta cũng có
   
1
180 180
2
= − = − =
 
BOM BOD BAD BAE
Như vậy, ta được ∼
 
OBM ABE (g.g). Gọi , ,
F X Y lần lượt là trung điểm của , ,
′
AB OO OB. Khi
đó, MY và EF là các đường trung tuyến tương ứng của OBM và ABE , vì thế ta được
∼
 
OMY AEF . Kết hợp với việc tứ giác ′
AEO F nội tiếp (vì   =
′ ′
=
AEO AFO )
90 ,

XY là đường
trung bình của ′
OBO và sử dụng tính đối xứng, ta được
     
=
′
= = ′ =
′
=
OMY AEF AO F AO O BO O OXY
Như vậy, tứ giác OMXY nội tiếp. Mặt khác, vì, , , ′
O B O là các điểm cố định nên ,
X Y cũng là các
điểm cố định. Do vậy, điểm M luôn chạy trên đường tròn ( )
OXY cố định.
Câu 4: (1 diểm)
Giả sử tập A là tập con của tập hợp gồm 30 số tự nhiên đầu tiên { }
0,1,2, ,29
… sao cho với k
nguyên bất kỳ và , ∈
a b A bất kỳ (có thể =
a b ) thì 30
+ +
a b k không là tích của hai số nguyên
liên tiếp. Chứng minh rằng số phần tử của tập hợp A nhỏ hơn hoặc bằng 10.
Lời giải. Ta trình bày hai lời giải
Cách 1. Trước hết ta loại các số mà bản thân nó không thể xuất hiện trong A , bao gồm:
• Các số có dạng
( )
1
: 0,1,3,6,10,15,24,28
2
+
n n
;
• Các số có dạng
( )
1 30
:16,18,25
2
+ +
n n
;
• Các số có dạng
( )
1 30
:13
2
+ −
n n
.
Website:
Liên hệ tài liệu word toán zalo: TÀI LIỆU TOÁN HỌC
6
Đối với các số còn lại, ta ghép cặp chúng như sau:
( ) ( ) ( ) ( ) ( ) ( ) ( ) ( )
2,4 , 5,7 , 8,12 , 9,11 , 19,23 , 20,22 , 24,26 , 27,29 ,14,17.
(gồm 8 cặp số và 2 số lẻ không trong cặp nào). Ta thấy các số không cùng một cặp không thể cùng
thuộc A , cho nên A chỉ chứa đúng một số trong mỗi cặp. Do đó A chỉ có tối đa 10 phần tử. Cách
2. Với hai số nguyên liên tiếp , 1
+
a a , ta có ( ) ( )
1 0,2,6,12,20,26 mod30
+ ≡
a a . Với ∈
a A, xét
=
b a và 0
=
k ta có 2a không đồng dư với ( )
0,2,6,12,20,26 mod30 nên a không đồng dư với
( )
0,1,3,6,10,13,15,16,18,21,25,28 mod30 . Suy ra
{ }
2,4,5,7,8,9,11,12,14,17,19,20,22,23,24,26,27,29
⊂ =
A B
và nếu phân hoạch B thành 10 tập
{ } { } { } { } { } { } { } { } { } { }
2,4 , 5,7 , 8,12 , 11,9 , 14,22 , 17,19 , 20 , 23,27 , 24,26 , 29
thì mỗi tập con này chứa nhiều nhất một phần tử của A . Do đó, A chứa tối đa 10 phần tử. Thực ra
ta có thể chứng minh được số phần tử của A nhiều nhất bằng 10 , chỉ cần chọn
{ }
2,5,8,11,14,17,20,23,26,29 .
=
A
Website:
Liên hệ tài liệu word toán zalo: TÀI LIỆU TOÁN HỌC
1
LỜI GIẢI ĐỀ THI TUYỂN SINH VÀO LỚP 10
NĂM HỌC: 2023 – 2024
Trường THPT Chuyên Khoa học tự nhiên
Môn thi : Toán (vòng 2)
Câu 1 : (3,5 điểm):
1) Giải phương trình
2 2
2 1 2 4 6 4 5
+ + + = −
x x x x x
2) Giải hệ phương trình
( )
3 3
30
30 120
 + =


+ = + + +


xy x y
x y x y
Câu 2 (2.5 điểm):
1) Tìm tất cả các cặp số nguyên dương (x;y) thỏa mãn
( )( ) ( )
4 1 3 1 7 2 3 7 2
+ + + = + +
x y y x y y
2) Với x,y,z là những số thực dương, tìm giá trị nhỏ nhất của biểu thức:
14 6 14 6 14 6
2 2 2 2 2 2
3 3 3
− + − + − +
= + +
+ + + + + +
x x y y z z
M
x y zx zy y z xy xz z x yz yx
Câu 3 (3 điểm). Cho tam giác ABC nhọn với AB < AC nội tiếp trong đường tròn (O) có tiếp
tuyến tại A của (O) cắt BC ở T sao cho TB > BC Gọi P và E lần lượt là trung điểm của TA và
TC.
1) Chứng minh rằng tứ giác APEB nội tiếp.
2) Gọi giao điểm thứ hai của AE với (O) là F. Láy G thuộc (O) sao cho FG song song với
AC. Chửng minh rằng  .
=
ATG TAF
3) Gọi H là trực tâm của tam giác ABC,D là giao điểm của AH và BC. M là trung điểm
BC. K đối xứng với A qua BC. N thuộc đường thẳng AM sao cho KN song song với HM. Lấy S
thuộc BC sao cho NS ⊥ NK . Dựng R thuộc tia AK sao cho AR.AH = 2
.
AD Q là điểm sao cho
PQ ⊥ AS và SQ ⊥ AO . Chứng minh rằng điểm đối xứng của A qua QR thuộc đường tròn
đường kinh DN.
Câu 4 (1 điểm). Viết 100 số nguyên dương đầu tiên 1,2,...,100 vào một bảng ô vuông kích thước
10 10
× một cách tuỳ ý sao cho mỗi ô vuông được viết đúng một số. Chứng minh rằng tồn tại hai
ô kề nhau (hai ô có cạnh chung) mà hai số được viết ở hai ô này có hiệu lớn hơn hoặc bằng 10.
Website:
Liên hệ tài liệu word toán zalo: TÀI LIỆU TOÁN HỌC
2
PHẦN LỜI GIẢI
Câu 1: (3,0 điểm):
1) Điều kiện: 0 5
≤ <
x . Ta biến đổi phương trình thành
2 4 6 4 6 4 4 5 5
+ + + + = + − + −
x x x x x x x x
Sử dụng hằng đẳng thức, ta thu được
2 2
( 4 6) (2 5 )
+ + = + −
x x x x
Suy ra 4 6 2 5
+ + = + −
x x x x (do từng vế đều không âm), hay
4 6 5
+ = + −
x x x
Bình phương hai vế của phương trình này ta có
( )
4 6 5 2 5
+ = + − + −
x x x x x
Hay ( )
4 1 2 5
+
= −
x x x . Tiếp tục hay Tiếp tục ta bình phương hai vế với điều kiện
4 1 0
+ ≥
x (đã thoả mãn được )
( )
2
16 8 1 4 5
+ +
= −
x x x x
Giải phương trình trên ta thu được
1 1
   
2 10
= =
x và x (đều thoả mãn điều kiện).
Vậy, phương trình đã cho có đúng hai nghiệm
1 1
,  
2 10
= =
x x
2) Đặt ,
=+ =
S x y P xy . Ta có
( )
3 3 3 3
( ) 3 3
+ = + − + = −
x y x y xy x y S SP
Khi đó hệ phương trình trở thành
3 3
30
3 30 120
=



− = + +


SP
S SP S
Thay SP = 30 vào phương trình thứ hai ta có
3 3
120 120
= + +
S S
hay ( )
3 3
120 120
+ = + + +
S S S S Ta nhận thấy
Nếu 3
120
> +
S S thì 3
120
> +
S S , suy ra
( )
3 3
120 120
+ > + + +
S S S S
loại.
Nếu 3
120
< +
S S thì 3
120
< +
S S suy ra
( )
3 3
120 120
+ < + + +
S S S S
loại.
Website:
Liên hệ tài liệu word toán zalo: TÀI LIỆU TOÁN HỌC
3
Như vậy ta có 3
3
120,   120  
= + − −
S S hay S S Giải phương trình ta thu được S = 5 khi
đó
30
6
= =
P
S
. Vậy ta có
5
6
+ =


=

x y
xy
Theo Vi-ét đảo thì x,y là hai nghiệm của phương trình
2
5 6 0
− + =
x x
Giải phương trình ta được ( ) ( ) ( )
, 2,3 , 3,2 .
=
x y
Vậy hệ đã cho có hai nghiệm (x,y) là ( )
2,3 và ( )
3,2
Câu 2 : (3,0 điểm):
1) Cách 1. Ta có các biến đổi phương trình sau
( )( ) ( )
4 1 3 1 7 2 3 7 2
+ + + = + +
x y y x y y
( )
2
2 1 2 3 7 21 2 3 7 2
⇔ − + + + + = + +
x y y y x y y
( )( )
2 1 3 7 1 2 21
⇔ − + + − =
x y y k y
(1)
Ta chứng minh UCLN ( )
2 1;3 7 1 2 1
− + + − =
x x y x
.Thật vậy, nếu UCLN(
2 1;3 7 1 2
− + + −
x x y x
) > 1 thì gọi p là ước nguyên tố chung của 2 1,3 7 1 2
− + + −
x x y x
. Suy ra p
|3 7
+
y y
. chú ý là 3 7
+
y y
đều không chia hết cho 3, 7 nên 3,7.
≠
p Lại có p| 21y
nên p ∈{3, 7}
mâu thuẫn.
Vậy UCLN( )
2 1;3 7 1 2 1
− + + − =
x x y x
Ta xét hai trường hợp sau
• Nếu x là số chẵn thì 2 1
−
x
chia hết cho 3 và 3 7 1 2
+ + −
x y x
chia 3 dư 1.
Khi đó, từ phương trình (1) ta có
2 1 3
3 7 1 2 7
 − =

+ + − =

x y
x y x y
Suy ra 2 3 1
= +
x y
,Chi ý là 3 1,3
≡
y
(mod 8) nên 3 1
+
y
không chia hết cho 8. Từ đó
2   1
= =
x và y . . Vậy (x, y) = (2, 1)
• Nếu x là số lẻ thì 2 1
−
x
chia 3 dư 1 và 3 7 1 2
+ + −
x y x
chia hết cho 3.
Khi đó, từ phương trình (1) ta có
2 1 7
3 7 1 2 3
 − =

+ + − =

x y
x y x y
Suy ra 2 7 1
= +
x y
. Về phải chia 7 dư 1 nên về trái chia 7 dư 1. Từ đó 3 ,
= ∈
x k k N * và
thay vào phương trình được
Website:
Liên hệ tài liệu word toán zalo: TÀI LIỆU TOÁN HỌC
4
( )( )
2
2 1 2 2 1 7
− + + =
k k k y
Vì UCLN ( ) { }
2
2 1;2 2 1 1,3
− + + ∈
k k k
nên UCLN( )
2
2 1;2 2 1 1.
− + + =
k k k
Vì
2
2 2 1 1
+ + >
k k
nên 2 1 1
− =
k
suy ra 1
=
k và 7 7
=
y
nên 1
=
y và 3 3
= =
x k . Vậy
( ) ( )
, 3,1 .
=
x y
Vậy tất cả các cặp số (x, y) thỏa mãn là (2, 1), (3, 1) .
Cách 2. Phương trình đã cho có thể viết lại thành
( )( )
2 7 1 2 3 1 0
− − − − =
x y x y
Tới đây giải giống hai trường hợp ở trên.
2) Ta có ( ) ( )
14 6 14 6 6 6 6
3 3 3 4 3 5 7 3 5 4 5
− + = + − + ≥ − + = +
x x x x x x x
theo bất đẳng thức AM-GM. Lại có cũng theo bất đẳng thức AM-GM, thì
( ) ( ) ( ) ( )
6 6 6 6 6 4 4 4 2
4 5 1 1 3 3 1 3 2
+ = + + + + + + ≥ + + ≥ +
x x x x x x x x x
Suy ra
4 2
2 2 2 2
2
≥ ∑ + ∑
+ + + +
x x
M
x y xz yz x y xz yz
và áp dụng bất đẳng thức cộng mẫu cho về trái, ta có
( )
( )
( ) ( )
( )
2
2 2 2 2 2 2 2 2 2 2
2 2 2 2 2 2 2 2 2 2 2 2
2( ) 3 6
3
2 2
+ + + + + + + + + +
≥ ≥ =
+ + + + + + + + + +
x y z x y z x y y z z x xy yz zx
M
x y y z z x xy yz zx x y y z z x xy yz zx
Dấu bằng xảy ra khi và chỉ khi x = y = z = 1 Giá trị nhỏ nhất của M là 3.
Câu 3 : (3,0 điểm)
G
F
E
P
T
O
A
B
C
Website:
Liên hệ tài liệu word toán zalo: TÀI LIỆU TOÁN HỌC
5
1) Vì AT là tiếp tuyến của nên ta được 2
.
=
TA TBTC . Như vậy, ta được
2
1 1
. . .
2 2
= = =
TPTA TA TBTC TBTE và vì tứ giác APEB là tứ giác nội tiếp.
2) Vì EP là đường trung bình của ∆ TAC , ∆ FGC là hình thang cân và AT là tiếp tuyến
của (O) nên ta thu được     
= = = =
AEP EAC FAC GCA TAG và    .
= = =
GAC FCA TAF PAE Như
vậy, ta được ∆ AEP ~∆ ACG (g-g) và dẫn đến =
AE AP
AC AG
. Lại chú ý 2
=
AT AP và
2
=
AC EP , ta thu được
2 2
= = =
AE AE AP AT
EP AC AG AG
. Kết hợp với  
=
AEP TAG ta thu được
  ~  
∆ ∆
AEP TAG (C.G.C) và vì thế  .
=
ATG TAF
3) ta xét bổ đề sau:
∆ABC, đường thẳng qua B vuông AC cắt AC,(ABC) tại F,D. E thuộc (ABC) thoả
DE//AC. Đường thẳng qua E vuông EF cắt BF tại G, đg thẳng qua B vuông AG cắt đường thẳng
qua C vuông AD tại H, L trung điểm AH. CMR AEL = 90
Giải: BH giao EG tại J khi đó J thuộc (BFE). Mặt khác gọi K trung điểm BC thì F,K,J
thẳng do BFK = ABE = BEJ = BFJ. Mà do FK vuông AD vuông CH nên J là trung điểm BH =>
JL // AB nên L thuộc EG => đpcm
Quay lại bài toán: Dựng hbh DKNG, DS' là đg kính của (ADG) khi đó KS'D = AS'D =
AGD = DNK => S trùng S'. NG giao BC tại F, NE vuông AD tại E. Khi đó A,F,E,G cùng thuộc
1 đg tròn ( EF = DN = AG )
• Đường thẳng qua E vuông AF cắt đg thẳng qua T vuông AS tại J. Khi đó theo bổ đề,
trung điểm Q' của AJ thuộc SG ( để ý T thuộc (AFGE) ). Mà Q'P vuông AS => Q' trùng Q. Hơn
nữa biến đổi tỉ số cho ta R là trung điểm AE nên QR // JE vuông AF kết hợp thêm nếu cho JE cắt
AF tại A' thì A' thuộc (EF) hay A' thuộc (DN) nên ta có đpcm.
Câu 4: (1,0 điểm):
Lời giải. Cách 1. Ta giải bài toán tổng quát: Điền các số 1.2.... 2
n với n > 1 vào các ô
vuông của bảng cỡn nn Khi đó tồn tại hai ô vuông kề nhau (chung cạnh) chứa hai số x, y mà |x -
y| ≥ n Kí hiệu ,
k k
m M tương ứng là số nhỏ nhất và số lớn nhất của hàng thứ k với k = 1, 2 ,...,n.
Chú ý là 1 2
, ,
m m ..., 1 2
, , , .,
……
n n
m M M M đôi một phân biệt. Đặt
m= max ( 1 2
, ,
m m ..., n
m ) và M= min ( 1 2
, , .,
…… n
M M M ) .
Xét hai trường hợp
• Nếu m < M thì ta có ≤ ≤
k k
m m M với mọi k = 1, 2 ,...,n. Điều này suy ra với hàng
k bất kỷ thì tồn tại hai số ≤ <
k k
a m b và với mỗi hàng k ta chọn cặp ( ,
k k
a b )
thuộc hai ô kề nhau ở hàng k. Vì 1 2
, , ..
… n
b b b lớn hơn m và các số 1 2
, , ..
… n
b b b là
đôi một phân biệt nên tồn tại k { }
1,2, .,
∈ … n . sao cho ≥ +
k
b n m và do đó
( ) .
− ≥ + − =
k k
b a m n m n
Website:
Liên hệ tài liệu word toán zalo: TÀI LIỆU TOÁN HỌC
6
• Nếu m > M thì gọi i, j {
∈ 1, 2 ,...,n} là các chỉ số sao cho i
m > M = j
M Khi đó tại
mỗi cột luôn tồn tại các số không vượt quá M (ví dụ như số hàng j) và các số lớn
hơn M (ví dụ như số ở hàng i). Khi đó với cột k bất kỳ tồn tại ,
k k
a b sao cho
≤ <
k k
a M b và ,
k k
a b thuộc hai ô kề nhau của cột k. Tương tự như trường hợp đầu
tiên thì ta cũng có tồn tại k {
∈ 1, 2 ,...,n}mà ≥ +
k
b M n . Suy ra
( ) .
− ≥ + − =
k k
b a M n m n
Trong mọi tình huống ta đều có điều phải chứng minh.
Cách 2. Giả sử phản chứng, tồn tại cách điền để không tồn tại x,y như vậy. Ta lần lượt điền các
số bắt đầu từ 1 vào bảng ô vuông, xét thời điểm đầu tiên mà cả n cột đều đã được điền số. Ta xét
hai trường hợp sau:
• Nếu tồn tại cột nào đó đã được phủ hoàn toàn, ta sẽ đổi vai trò của hàng và cột (xét
thời điểm đầu tiên cả n hàng đều có số). Nếu khi đó lại tồn tại một hàng được phủ
hoàn toàn, thì tức là hai thời điểm đang xét trùng nhau. Điều này là không thể vì ô
vừa được thêm phải là ô cuối cùng của hàng và cột đó, xét thời điểm ngay trước đó
cho ta tất cả các hàng đều đã được điền.
• Nếu không tồn tại cột đã phủ hoàn toàn, do mỗi cột đều có ô đã điền, ta có thể chọn
ra ở cột thứ i được cặp ô ,
i i i
A B mà A được điền còn i
B thì chưa.
Cần có 1
− ≤ −
i i
B A n kéo theo ≤
i
B max ( 1 2
, , ..,
… n
A A A ) + n -1,∀i
Mà các số từ 1 đến max( )
i
A đều đã được điền nên ( )
max
>
i i
B A , vô lý do n số i
B
phân biệt.
Các trường hợp cho ta giả sử sai và ta có điều phải chứng minh.
Website:
Liên hệ tài liệu word môn toán: TÀI LIỆU TOÁN HỌC
Câu I. (4 điểm)
1) Giải hệ phương trình
( ) 3 2
3 2
6 5 5 42
5 6 30 42
xy x y x
x x y x y
 + + + =


+ + + =


2) Giải phương trình :
( ) ( )( )
( )
3 3 3
6 3 2 3 6 3 24
x x x x
+ + − + + − =
Câu II. (2 điểm)
1) Tìm tất cả các cặp số nguyên ( )
,
x y thỏa mãn đẳng thức :
( )
2022
2 2
25 354 60 36 305 5 6
y x x y y x
+ + = + + −
2) Trên bàn có 8 hộp rỗng (trong các hộp không có viên bi nào). Người ta thực hiện các lần thêm bi
vào các hộp theo quy tắc sau : mỗi lần ta chọn ra 4 hộp bất kỳ và bỏ vào hộp 1 viên, một hộp 2
viên, hai hộp còn lại mỗi hộp 3 viên. Hỏi số lần thêm bi ít nhất có thể để nhận được số bi ở 8 hộp
trên là số tự nhiên liên tiếp ?
Câu III. (3 điểm)
Cho hình chữ nhật ( )
ABCD AB AD
< nội tiếp đường tròn ( )
O . Trên cạnh AD lấy hai điểm E và F ( ,
E F
không trùng với , )
A D sao cho E nằm giữa A và F, đồng thời
1
2
ABE DCF BOC
∠ + ∠ = ∠
1) Chứng minh rằng BE và CF cắt nhau tại một điểm nằm trên đường tròn ( )
O
2) Đường thẳng qua O song song với BC cắt ,
BE CF theo thứ tự tại , .
M N Chứng minh rằng
1
180
2
DAM ADN AOD
∠ + ∠ + ∠ = °
3) Dựng hình chữ nhật MNPQ sao cho NQ song song với ,
BD đồng thời MP song song với .
AC
Chứng minh rằng đường tròn ngoại tiếp hình chữ nhật MNPQ tiếp xúc với đường tròn ( )
O
Câu IV. (1 điểm)
Cho , ,
a b c là những số thực dương. Chứng minh rằng :
2 6 2 4 3 32
3 2
a a b a c a b c a
a b a c b c b c a b c
+ + + +
+ + + ≥
+ + + + + +
SỞ GIÁO DỤC VÀ ĐÀO TẠO
HÀ NỘI
ĐỀ CHÍNH THỨC
Đề thi gồm 01 trang
Đề số 1
KỲ THI VÀO LỚP 10 CHUYÊN KHOA HỌC TỰ NHIÊN
NĂM HỌC 2022 – 2023
Môn: TOÁN (VÒNG 1)
Thời gian làm bài: 120 phút
(Không kể thời gian giao đề)
Website:
Liên hệ tài liệu word môn toán: TÀI LIỆU TOÁN HỌC
Câu I. (3,5 điểm)
1) Với , ,
a b c là những số thực dương thỏa mãn điều kiện
1 1 1
1.
a b c
+ + =Chứng minh rằng :
( )( )( )
1 1 1 1
2
abc
a bc b ca c ab a bc b ca c ab
 
+ + =
 
+ + + + + +
 
2) Giải hệ phương trình
2 2
2 3 6
3 2 1 2 2 6
x xy y
x y x y
 + + =


+ +
= + +


Câu II. (2,5 điểm)
1) Tìm tất cả các cặp số nguyên dương ( )
;
x y thỏa mãn đẳng thức :
( )( ) ( )
3 3
3 2 3 4
5 5
x y x y xy x y x y xy
+ + + = + + +
2) Với , ,
a b c là những số thực dương thỏa mãn các điều kiện sau
( )( )
2 2
2 2 3 3
3; 2 10; 2 2 14
1 1 4 2 2 2 2
c b a b a b a c
a b ab a b a b
 ≤ < ≤ + ≤ + + ≤


+ + + ≤ + + +


. Tìm giá trị lớn nhất của biểu thức:
4 4 2 2
4 2 4
P a b b c
= + + +
Câu III. (3 điểm)
Cho tam giác ABC nhọn, không cân, nội tiếp đường tròn (O). Điểm P nằm trong tam giác ABC .
Gọi ,
E F lần lượt là hình chiếu vuông góc của P trên các cạnh ,
CA CB . Giả sử tứ giác BCEF nội tiếp
trong đường tròn ( )
K
1) Chứng minh rằng AP vuông góc với BC
2) Chứng minh rằng 2
AP OK
=
3) Đường thẳng qua P vuông góc với AP cắt đường tròn tại hai điểm ,
Q R . Chứng minh rằng đường
tròn tâm A bán kính AP tiếp xúc với đường tròn ngoại tiếp KQR
∆
Câu IV. (1 điểm)
Cho các điểm 1 2 30
, ,.....,
A A A theo thứ tự nằm trên một đường thẳng sao cho độ dài các đoạn 1
k k
A A +
bằng k (đơn vị dài), với 1,2,....,29
k = . Ta tô màu mỗi đoạn thẳng 1 2 29 30
,.....,
A A A A bởi 1 trong 3 màu (mỗi
đoạn được tô bởi đúng 1 màu). Chứng minh rằng với mọi cách tô màu,, ta luôn chọn được hai số nguyên
dương 1 29
j i
≤ ≤ ≤ sao cho hai đoạn 1
i i
A A+ và 1
j j
A A + được tô cùng màu và i j
− là bình phương của số
nguyên dương.
SỞ GIÁO DỤC VÀ ĐÀO TẠO
HÀ NỘI
ĐỀ CHÍNH THỨC
Đề thi gồm 01 trang
Đề số 2
KỲ THI VÀO LỚP 10 CHUYÊN KHOA HỌC TỰ NHIÊN
NĂM HỌC 2022 – 2023
Môn: TOÁN (VÒNG 2)
Thời gian làm bài: 120 phút
(Không kể thời gian giao đề)
Website:
Liên hệ tài liệu word môn toán: TÀI LIỆU TOÁN HỌC
Bài 1. Giải phương trình ( )( )
13 5 18 8 61 3 5 8
x x x x x
− + + = + + − + .
Bài 2. Giải hệ phương trình:
( )
( ) ( )
4 4 2 2
4
6 1 1
2
x y x y
x x y x y
 + + =


+ =−


.
Bài 3. Tìm số nguyên dương n nhỏ nhất, biết rằng khi chia n cho 7, 9, 11, 13 được các số dư tương
ứng là 3, 4, 5, 6 .
Bài 4. Cho tam giác nhọn ABC có điểm P nằm trong tam giác ( P không nằm trên các cạnh). Gọi
, ,
J K L lần lượt là tâm đường tròn nội tiếp các tam giác PBC , PCA , PAB .
1. Chứng minh rằng    450
BJC CKA ALB
+ + = °.
2. Giả sử PB PC
= và PC PA
< . Gọi , ,
X Y Z lần lượt là hình chiếu vuông góc của , ,
J K L trên
các cạnh , ,
BC CA AB . Dựng hình bình hành XYWZ . Chứng minh W nằm trên phân giác 
BAC .
Bài 5. Cho tập { }
1; 2; 3;...;2021
A = . Tìm số nguyên dương k lớn nhất ( )
2
k > sao cho ta có thể chọn
được k số phân biệt từ tập A mà tổng của hai số phân biệt bất kỳ trong k số được chọn không
chia hết cho hiệu của chúng.
 HẾT 
SỞ GIÁO DỤC VÀ ĐÀO TẠO
HÀ NỘI
ĐỀ CHÍNH THỨC
Đề thi gồm 01 trang
Đề số 3
KỲ THI VÀO LỚP 10 CHUYÊN KHOA HỌC TỰ NHIÊN
NĂM HỌC 2021 – 2022
Môn: TOÁN (VÒNG 1)
Thời gian làm bài: 120 phút
(Không kể thời gian giao đề)
Website:
Liên hệ tài liệu word môn toán: TÀI LIỆU TOÁN HỌC
Câu 1.
1) Với , ,
a b c là các số thực thỏa mãn 0
a b c
+ + ≠ và ( )( )( ) 1
a b b c c a
+ + + =
.
Chứng minh rằng 2 2 2
1 ( )
( ) 1 ( ) 1 ( )
a b abc ab a b c
a a b c abc b a b c abc a b c
+ + + +
+ = ⋅
+ + + + + + + + + +
2) Giải hệ phương trình:
( )
2 2 2 2
4 4 2 11
3 2 31 9 18 13
x y xy x y
xy x y x y xy
 + + + =


+ + = + +


.
Câu 2.
1) Tìm ,
x y nguyên dương thỏa mãn 3 29 2 .
x y
+ =
2) Với , ,
a b c là các số thực dương thỏa mãn điều kiện ( )
2 9
a b c ab bc ca
+ + + + + =
.
Tìm giá trị lớn nhất của biểu thức 2 2 2
1 1 1
10 21 10 21 10 21
a b c
M
a a b b c c
+ + +
= + + ⋅
+ + + + + +
Câu 3. Cho hình thoi ABCD có 
BAD nhọn có đường tròn nội tiếp ( )
O . Các điểm ,
M N lần lượt thuộc
các cạnh ,
CB CD sao cho MN tiếp xúc ( )
O tại P và tam giác CMN không cân. MN lần lượt cắt
,
AB AD tại ,
E F . Gọi ,
K L lần lượt là trực tâm các ,
BME DNF
∆ ∆ .
1) Chứng minh OP đi qua trung điểm I của KL .
2) Gọi H là trực tâm tam giác CMN . Chứng minh
1
2 2
OI EF
CH MN
− =
− .
3) Gọi ,
EK FL lần lượt cắt BD tại ,
S T . NS cắt MT tại Q . Đường tròn nội tiếp tam giác CMN
tiếp xúc MN với tại G . Chứng minh PQ song song với .
GH
Câu 4. Giả sử 1 2 2021
, ,.....,
a a a là những số thực thỏa mãn 2021
1 2
2 2 2
1 2 2021
...... 0
1 1 1
a
a a
a a a
+ + + =
+ + +
.
Chứng minh rằng tồn tại số nguyên ( )
1 2021
k k
≤ ≤ sao cho 1 2
2 2 2
1 2
2 2 1
......
1 1 1 8
k
k
ka
a a k
a a a
+
+ + + ≤ ⋅
+ + +
……………………….HẾT………………………
SỞ GIÁO DỤC VÀ ĐÀO TẠO
HÀ NỘI
ĐỀ CHÍNH THỨC
Đề thi gồm 01 trang
Đề số 4
KỲ THI VÀO LỚP 10 CHUYÊN KHOA HỌC TỰ NHIÊN
NĂM HỌC 2021 – 2022
Môn: TOÁN (VÒNG 2)
Thời gian làm bài: 120 phút
(Không kể thời gian giao đề)
Website:
Liên hệ tài liệu word môn toán: TÀI LIỆU TOÁN HỌC
Câu I. (4 điểm)
1) Giải hệ phương trình :
( )
2 2
3 2
7
9 70
x y xy
x xy x y
 + + =


= + −


2) Giải phương trình: ( )( )
11 5 8 2 1 24 3 5 2 1
x x x x
− + − = + − −
Câu II. (2 điểm)
1) Tìm ,
x y nguyên dương thỏa mãn 2 2 2 2
16 99 9 36 13 26
x y xy x y x y
− + = + + +
2) Với ,
a b là những số thực dương thỏa mãn
2 2
2 2 3 5 ;8 12 2 3 5 10
a b a b a b ab
≤ + ≤ + ≤ + + +
Chứng minh rằng:
2 2
3 8 10 21
a b ab
+ + ≤
Câu III. (3 điểm)
Cho tam giác ABC có 
BAC là góc nhỏ nhất trong ba góc của tam giác và nội tiếp đường tròn (O). Điểm
D thuộc cạnh BC sao cho AD là phân giác .
BAC Lấy các điểm ,
M N thuộc (O) sao cho đường thẳng
,
CM BN cùng song song với đường thẳng AD
1) Chứng minh rằng AM AN
=
2) Gọi giao điểm của đường thẳng MN với các đường thẳng ,
AC AB lần lượt là , .
E F Chứng minh
rằng bốn điểm , , ,
B C E F cùng thuộc một đường tròn
3) Gọi ,
P Q theo thứ tự là trung điểm của các đoạn thẳng , .
AM AN Chứng minh rằng các đường
thẳng , ,
EQ FP ADđồng quy.
Câu IV. (1 điểm) Với , ,
a b clà những số thực dương thỏa mãn 3.
a b c
+ + =Chứng minh rằng:
( )
( )
( )
( )
( )
( )
2 2 2
2 2 2
4
2 2 2
a a bc b b ca c c ab
b ab c c bc a a ca b
+ + +
+ + ≥
+ + +
SỞ GIÁO DỤC VÀ ĐÀO TẠO
HÀ NỘI
ĐỀ CHÍNH THỨC
Đề thi gồm 01 trang
Đề số 5
KỲ THI VÀO LỚP 10 CHUYÊN KHOA HỌC TỰ NHIÊN
NĂM HỌC 2020 – 2021
Môn: TOÁN (VÒNG 1)
Thời gian làm bài: 120 phút
(Không kể thời gian giao đề)
Website:
Liên hệ tài liệu word môn toán: TÀI LIỆU TOÁN HỌC
Câu I. (4 điểm)
1) Giải hệ phương trình:
( )( )
2 3 3
( )( 1) 4
5 12 13 243
x y x
y xy x y x y y
+ + =



+ + + + + + + =


2) Giải phương trình: 7 7 7
( 12) (2 12) (24 3 ) 0
x x x
− + − + − =
Câu II. (2 điểm)
1) Tìm tất cả các số nguyên dương a, b, c sao cho cả ba số 2 2 2
4 5 ;4 5 ;4 5
a b b c c a
+ + + đều là bình phương
của số nguyên dương.
2) Từ một bộ bốn số thực ( , , , )
a b c d ta xây dựng bộ số mới ( , , , )
a b b c c d d a
+ + + + và liên tiếp xây dựng
các bộ số mới theo quy tắc trên. Chứng minh rằng nếu ở hai thời điểm khác nhau, ta thu được cùng một bộ
số (có thể khác thứ tự) thì bộ số ban đầu phải có dạng ( , , , )
a a a a
− −
Câu III. (3 điểm) Cho tam giác ABC cân tại A với 90o
BAC
∠ < . Gọi P là giao điểm của BE với trung
trực của BC. Gọi K là hình chiếu vuông góc của P lên AB. Gọi Q là hình chiếu vuông góc của E lên
AP. Gọi giao điểm của EQ và PK là F .
1) Chứng minh rằng bốn điểm A, E, P, F cùng thuộc một đường tròn.
2) Gọi giao điểm của KQ và PE là L. Chứng minh LA vuông góc LE.
3) Gọi giao điểm của FL và AB là S . Gọi giao điểm của KE và AL là T . Lấy R là điểm đối xứng với A
qua L . Chứng minh rằng đường tròn ngoại tiếp tam giác AST và đường tròn ngoại tiếp tam giác BPR tiếp
xúc với nhau.
Câu IV. (1 điểm) Với a, b, c là những số thực dương thỏa mãn 3
a b c
+ + =. Chứng minh rằng
2
1 1 1 4
3 1 1 3
a b c
a b c abc bc ca ab
   
+ + − + ≥ + + +
   
   
SỞ GIÁO DỤC VÀ ĐÀO TẠO
HÀ NỘI
ĐỀ CHÍNH THỨC
Đề thi gồm 01 trang
Đề số 6
KỲ THI VÀO LỚP 10 CHUYÊN KHOA HỌC TỰ NHIÊN
NĂM HỌC 2020 – 2021
Môn: TOÁN (VÒNG 1)
Thời gian làm bài: 120 phút
(Không kể thời gian giao đề)
Website:
Liên hệ tài liệu word môn toán: TÀI LIỆU TOÁN HỌC
Bài 1.
a, Giải phương trình: 2
2
26 5
2 26 5 3 30
30
x
x x
x
+
+ += +
+
b, Giải hệ phương trình:
2 2
2
2
( 2 )(2 3 4 ) 27
x y
x y y xy
 + =


+ + + =


Bài 2.
a, Tìm tất cả các cặp số nguyên thỏa mãn: 2 2
( 1)( ) 3 1
x x y xy x
− + + = −
b, Với x,y là các số thực thay đổi thỏa mãn 1≤ y ≤ 2 và xy + 2 ≥ 2y, tìm giá trị nhỏ nhất của biểu
thức
2
2
4
1
x
M
y
+
=
+
Bài 3. Cho hình vuông ABCD, đường tròn (O) nội tiếp hình vuông tiếp xúc với các cạnh AB, AD
tại hai điểm E,F. Gọi G là giao điểm các đường thẳng CE và BF.
a, Chứng minh rằng năm điểm A,F,O,G,E cùng nằm trên một đường tròn
b, Gọi giao điểm của đường thẳng FB và đường tròn là M(M ≠ F). CMR M là trung điểm của
đoạn thẳng BG.
c, CMR trực tâm của tam giác GAF nằm trên đường tròn (O)
Bài 4. Cho x, y, z là các số thực dương thỏa mãn xy + yz + xz = 1. Chứng minh rằng:
3
2 2 2 2 2 2
1 1 1 2
3
1 1 1 1 1 1
 
 
+ + ≥ + +
 
+ + + + + +
 
x y z
x y z x y z
SỞ GIÁO DỤC VÀ ĐÀO TẠO
HÀ NỘI
ĐỀ CHÍNH THỨC
Đề thi gồm 01 trang
Đề số 7
KỲ THI VÀO LỚP 10 CHUYÊN KHOA HỌC TỰ NHIÊN
NĂM HỌC 2019 – 2020
Môn: TOÁN (VÒNG 1)
Thời gian làm bài: 150 phút
(Không kể thời gian giao đề)
Website:
Liên hệ tài liệu word môn toán: TÀI LIỆU TOÁN HỌC
Bài 1.
a. Giải hệ phương trình:
( )( )
2 2
2
3 4 8
.
2 8
x y xy
x y x xy
 + + =


+ + + =


b. Giải phương trình:
( )
2
2
27 27 2
2 5 2
2 5
x x x
x
x x
+ + +
=
+ −
+ − +
Bài 2.
a. Chứng minh rằng với mọi số nguyên dương n , ta luôn có
( ) ( ) ( )
7 7 7
7 7 7
27 5 10 10 27 5 5 10 27
n n n
     
+ + + + + + + +
     
chia hết cho 42 .
b. Với ,
x y là các số thực dương thay đổi thỏa mãn điều kiện
2 2
4 4 17 5 5 1
x y xy x y
+ + + + ≥ .
Tìm giá trị nhỏ nhất của biểu thức: 2 2
17 17 16
P x y xy
= + + .
Bài 3. Cho tam giác ABC cân tại A, có đường tròn nội tiếp ( )
I . Các điểm ,
E F theo thứ tự
thuộc các cạnh ,
CA AB ( E khác C và A; F khác B và A) sao cho EF tiếp xúc với đường tròn
( )
I tại điểm P . Gọi ,
K L lần lượt là hình chiếu vuông góc của ,
E F trên BC . Giả sử FK cắt EL
tại điểm J . Gọi H là hình chiếu vuông góc của J trên BC .
a) Chứng minh rằng HJ là phân giác của góc EHF .
b) Kí hiệu 1 2
,
S S lần lượt là diện tích của các tứ giác BFJL và CEJK . Chứng minh rằng
2
1
2
2
S BF
S CE
=
.
c) Gọi D là trung điểm của cạnh BC . Chứng minh rằng ba điểm , ,
P J D thẳng hàng.
Bài 4. Cho M là tập tất cả 4039 số nguyên liên tiếp từ 2019
− đến 2019 . Chứng minh rằng trong
2021 số đôi một phân biệt được chọn bất kì từ M luôn tồn tại ba số phân biệt có tổng bằng 0 .
SỞ GIÁO DỤC VÀ ĐÀO TẠO
HÀ NỘI
ĐỀ CHÍNH THỨC
Đề thi gồm 01 trang
Đề số 8
KỲ THI VÀO LỚP 10 CHUYÊN KHOA HỌC TỰ NHIÊN
NĂM HỌC 2019 – 2020
Môn: TOÁN (VÒNG 2)
Thời gian làm bài: 150 phút
(Không kể thời gian giao đề)
Website:
Liên hệ tài liệu word môn toán: TÀI LIỆU TOÁN HỌC
Bài 1.
a) Giải phương trình 2 3
x x 2 x 1 2 x 1
− + + = + .
b) Giải hệ phương trình
2
2 2
xy y 1 y
x 2y 2xy 4 x
 + = +


+ + =+


.
Bài 2.
a) Tìm tất cả các cặp số nguyên ( )
x; y thỏa mãn ( )( )
2
x y 3x 2y 2x y 1
+ + = + − .
b) Với a, b là các số thực dương thay đổi thỏa mãn
b
a 2b 2
3
+ = + . Tìm giá trị nhỏ nhất
của biểu thức
a b
M
a 2b b 2a
= +
+ +
.
Bài 3. Cho tam giác ABC có đường tròn nội tiếp ( )
I tiếp xúc với các cạnh BC, CA, AB lần lượt tại
các điểm D, E, F. Gọi K là hình chiếu vuông góc của B trên đường thẳng DE và M là trung điểm
của đoạn thẳng DF.
a) Chứng minh rằng hai tam giác BKM và DEF đồng dạng với nhau.
b) Gọi L là hình chiếu của vuông góc của C trên đường thẳng DF và N là trung điểm của
đoạn thẳng DE. Chứng minh rằng hai đường thẳng MK và NL song song với nhau.
c) Gọi J, X lần lượt là trung điểm của các đoạn thẳng KL và ID. Chứng minh rằng đường
thẳng JX vuông góc với đường thẳng EF.
Bài 4. Trên mặt phẳng cho hai điểm P và Q phân biệt. Xét 10 đường thẳng nằm trong mặt
phẳng trên thỏa mãn các tính chất sau:
i) Không có hai đường thẳng nào song song hoặc trùng nhau.
ii) Mỗi đường thẳng đi qua P hoặc Q, không có đường thẳng nào đi qua cả P và Q.
Hỏi 10 đường thẳng trên có thể chia mặt phẳng thành tối đa bao nhiêu miền? Hãy giải thích.
SỞ GIÁO DỤC VÀ ĐÀO TẠO
HÀ NỘI
ĐỀ CHÍNH THỨC
Đề thi gồm 01 trang
Đề số 9
KỲ THI VÀO LỚP 10 CHUYÊN KHOA HỌC TỰ NHIÊN
NĂM HỌC 2018 – 2019
Môn: TOÁN (VÒNG 1)
Thời gian làm bài: 120 phút
(Không kể thời gian giao đề)
Website:
Liên hệ tài liệu word môn toán: TÀI LIỆU TOÁN HỌC
Câu 1.
a) Giải hệ phương trình :
( )( )
3 3 3 3
( ) 2
7 1 1 31
xy x y
x y x y x y
+ =



+ + + + + =


b) Giải phương trình: ( )
9 3 3 2 7 5 3 2
x x x x
+ − = + −
Câu 2.
a) Cho ,
x y là các số nguyên sao cho 2 2 2
2 ; 2
x xy y xy y x
− − − − đều chia hết cho 5. Chứng minh
2 2
2 2
x y x y
+ + + cũng chia hết cho 5
b) Cho 1 2 50
, ,......,
a a a là các số nguyên thỏa mãn: 1 2 50
1 ...... 50
a a a
≤ ≤ ≤ ≤ , 1 2 50
..... 100
a a a
+ + + = .
Chứng minh rằng từ các số đã cho có thể chọn được một vài số có tổng là 50
Câu 3. Cho ngũ giác lồi ABCDE nội tiếp (O) có / /
CD BE . Hai đường chéo CE và BD cắt nhau tại
P. Điểm M thuộc BE sao cho  
MAB PAE
= . Điểm K thuộc AC sao cho MK song song AD, điểm L
thuộc đường thẳng AD sao cho ML // AC. Đường tròn ngoại tiếp tam giác KBC cắt BD, CE tại Q
và S (Q khác B, S khác C)
a) Chứng minh 3 điểm K, M, Q thẳng hàng
b) Đường tròn ngoại tiếp tam giác LDE cắt BD, CE tai T và R (T khác D, R khác E). Chứng
minh M, S, Q, R,T cùng thuộc một đường tròn
c) Chứng minh đường tròn ngoại tiếp tam giác PQR tiếp xúc (O)
Câu 4. Cho , ,
a b c là các số thực dương. Chứng minh rằng
1 1
2
ab bc
a b b c a b b c
  
+ + ≤
  
 
+ + + +
 
 
SỞ GIÁO DỤC VÀ ĐÀO TẠO
HÀ NỘI
ĐỀ CHÍNH THỨC
Đề thi gồm 01 trang
Đề số 10
KỲ THI VÀO LỚP 10 CHUYÊN KHOA HỌC TỰ NHIÊN
NĂM HỌC 2018 – 2019
Môn: TOÁN (VÒNG 2)
Thời gian làm bài: 150 phút
(Không kể thời gian giao đề)
Website:
Liên hệ tài liệu word môn toán: TÀI LIỆU TOÁN HỌC
Câu 1. (3.5 điểm)
a) Giải hệ phương trình
2 2
2 3
1
2
x y xy
x x y y

   



  



b) Giải phương trình     
2
2 1 1 1 1 2 1
x x x x x
       
Câu 2 (2.5 điểm)
a) Chứng minh rằng không tồn tại các số nguyên x, y thỏa mãn đẳng thức
2 2
12 26 15 4617
x xy y
  
b) Với a, b là các số thực dương. Tìm giá trị lớn nhát của biểu thức
  2 2
1 1 1
M a b
ab
a b b a
 

 
   
 
 
  
 
Câu 3 (3.0 điểm)
Cho hình thoi ABCD có
 0
90
BAD  . Đường tròn tâm I nội tiếp tam giác ABD tiếp xúc với BD
và BA lần lượt tại J và L. Trên đường thẳng IJ lấy điểm K sao cho BK song song ID.
a) Chứng minh rằng  
CBK ABI
 .
b) Chứng minh rằng KC KB
 .
c) Chứng minh rằng bốn điểm C, K, I ,L cùng nằm trên một đường tròn.
Câu 4. (1.0 điểm)
Tìm tập hợp số nguyên dương n sao cho tồn tại một cách sắp xếp các số 1;2;3;...;n thành
1 2 3
; ; ;...; n
a a a a mà khi chia các số 1 1 2 1 2 3 1 2
; ; ;...; ...an
a a a a a a a a cho n ta được các số dư đôi một khác
nhau.
SỞ GIÁO DỤC VÀ ĐÀO TẠO
HÀ NỘI
ĐỀ CHÍNH THỨC
Đề thi gồm 01 trang
Đề số 11
KỲ THI VÀO LỚP 10 CHUYÊN KHOA HỌC TỰ NHIÊN
NĂM HỌC 2017 – 2018
Môn: TOÁN (VÒNG 1)
Thời gian làm bài: 120 phút
(Không kể thời gian giao đề)
Website:
Liên hệ tài liệu word môn toán: TÀI LIỆU TOÁN HỌC
Câu 1 (3.5 điểm).
1. Giải hệ phương trình 2 2
3
3
x y x y
x y xy

   



   



2. Với a, b là các số thực dương thỏa mãn 1
ab a b
   . Chứng minh rằng:
  
2 2
2 2
1
1 1 2 1 1
a b ab
a b a b

 
   
Câu 2 (2.5 điểm). 1. Giả sử p và q là các số nguyên tố thỏa mãn đẳng thức    
2
1 1
p p q q
   .
a) Chứng minh rằng tồn tại số nguyên dương k sao cho 2
1 , 1
p kq q kp
    .
b) Tìm tất cả các số nguyên tố p, q thỏa mãn đẳng thức    
2
1 1
p p q q
   .
2. Với a, b, c là các số thực dương thỏa mãn 2
ab bc ca abc
    . Tìm giá trị lớn nhất của
biểu thức 2 2 2
1 1 1
2 2 2 2 2 2
a b c
M
a a b b c c
  
  
     
Câu 3 (3.0 điểm). Cho tam giác ABC nhọn với AB AC
 . Gọi E, F lần lượt là trung điểm của
CA, AB. Đường trung trực của EF cắt BC tại D. Giả sử P nằm trong góc 
EAF và nằm ngoài tam
giác AEF sao cho  
EF
PEC D
 và  
PEB DFE
 . Đường thẳng PA cắt đường tròn ngoại tiếp tam
giác PEF tại Q khác P.
a) Chứng minh rằng   
EQF BAC EDF
  .
b) Tiếp tuyến tại P của đường tròn ngoại tiếp tam giác PEF cắt CA, AB lần lượt tại M, N.
Chứng minh rằng bốn điểm C, M, B, N cùng nằ trên một đường tròn. Gọi đường tròn này là
đường tròn  
K .
c) Chứng minh rằng đường tròn  
K tiếp xúc với đường tròn ngại tiếp tam giác AEF.
Câu 4 (1.0 điểm). Cho n là số nguyên dương với 5
n  . Xét đa giác lồi n cạnh. Người ta muốn
kẻ một số đường chéo của đa giác mà các đường chéo này chia đa giác thành đúng k miền, mõi
miền là mọt ngũ giác lồi (hai miền bất kì không có điểm chung trong).
a) Chứng minh rằng ta có thể thực hiện được với 2018, 672
n k
 
b) Với 2017, 672
n k
  ta có thể thực hiện được không? Hãy giải thích.
SỞ GIÁO DỤC VÀ ĐÀO TẠO
HÀ NỘI
ĐỀ CHÍNH THỨC
Đề thi gồm 01 trang
Đề số 12
KỲ THI VÀO LỚP 10 CHUYÊN KHOA HỌC TỰ NHIÊN
NĂM HỌC 2017 – 2018
Môn: TOÁN (VÒNG 2)
Thời gian làm bài: 150 phút
(Không kể thời gian giao đề)
Website:
Liên hệ tài liệu word môn toán: TÀI LIỆU TOÁN HỌC
Câu 1 (3.5 điểm).
a) Giải hệ phương trình
 
  
3 3
2 2
x + y + xy x + y = 4
xy + 1 x + y = 4









b) Giải phương trình
8 3
7 2 5
5
x
x x

   
Câu 2 (2.5 điểm)
a)Tìm tất cả các giá tri của m sao cho tồn tại cặp số nguyên  
;
x y thỏa mãn hệ phương trình :
 
2
2 2
2 3
2 6
mxy m
m x y m

  



   



b) Với x, y là những số thực thỏa mãn các điều kiện 0 2;2 2
x y x y xy
     .
Tìm giá trị lớn nhất của biểu thức    
2 2 2 2
P x x 1 y y 1
   
Câu 3 (3.0 điểm). Cho tam giác ABC nhọn nội tiếp đường tròn  
O với AB AC
 . Phân giác của
góc 
BAC cắt BC tại D và cắt đường tròn  
O tại E khác A. M là trung điểm của đoạn thẳng AD.
Đường thẳng BM cắt đường tròn  
O tại P khác B. Giả sử các đường thẳng EP và AC cắt nhau tại
N.
a) chứng minh rằng tứ giác APNM nội tiếp và N là trung điểm của đoạn thẳng AC.
b) Giả sử đường tròn  
K ngoại tiếp tam giác EMN cắt đường thẳng AC tại Q khác N. Chứng
minh rằng B và Q đối xứng nhau qua AE.
c) Giả sử đường tròn  
K cắt đường thẳng BM tại M. Chứng minh rằng RA vuông góc RC.
Câu 4 (1.0 điểm).
Số nguyên a được gọi là số “đẹp” nếu với mọi cách sắp xếp theo thứ tự tùy ý của 100 số 1,
2, 3,…, 100 luôn tồn tại 10 số hạng liên tiếp có tổng lớn hơn hoặc bằng a. Tìm số “đẹp” lớn nhất
SỞ GIÁO DỤC VÀ ĐÀO TẠO
HÀ NỘI
ĐỀ CHÍNH THỨC
Đề thi gồm 01 trang
Đề số 13
KỲ THI VÀO LỚP 10 CHUYÊN KHOA HỌC TỰ NHIÊN
NĂM HỌC 2016 – 2017
Môn: TOÁN (VÒNG 1)
Thời gian làm bài: 120 phút
(Không kể thời gian giao đề)
Website:
Liên hệ tài liệu word môn toán: TÀI LIỆU TOÁN HỌC
Câu 1(3.5 điểm).
a) Giải hệ phương trình
2 2
2 2
4 5
4 8 5 10 1
x y
x xy x y

  



    



b) giải phương trình
3
2
2
64 4
5 6 5
5 6 6
x x
x x
x x

  
 
Câu 2 (2.5 điểm).
a) Với x, y là những số nguyên thỏa mãn đẳng thức
2 2
1 1
2 3
x y
 
 . Chứng minh
2 2
40
x y
  .
b) Tìm tất cả các cặp số nguyên  
;
x y thỏa mãn đẳng thức sau 4 2 3
2
x x y
  .
Câu 3 (3.0 điểm) Cho hình vuông ABCD nội tiếp đường tròn  
O . P là điểm thuộc cung nhỏ AD
của đường tròn  
O và P khác A, D. Các đường thẳng PB, PC lần lượt cắt AD tại AD tại M, N.
Đường trung trực của AM cắt đường thẳng AC, PB lần lượt tại E, K. Đường trung trực DN cắt
các đường thẳng BD, PC lần lượt tại F, L.
a) Chứng minh rằng ba điểm K, O, L thẳng hàng.
b) Chứng minh đường thẳng PO đi qua trung điểm của EF
c) Giả sử đường thảng EK cắt đường thẳng FL và AC cắt nhau tại T. Đường thẳng ST cắt
các đường thẳng PB, PC lần lượt tại U và V. Chứng minh rằng bốn điểm K, L, V, U cùng thuộc
một đương tròn.
Câu 4(1.0 điểm) Chứng minh rằng với mọi số tự nhiên n 3
 luôn tồn tại cách xếp bộ n số
1,2,3,...,n thành bộ số 1 2 3
, , ,..., n
x x x x sao cho
2
i k
j
x x
x

 với mọi bộ chỉ số  
; ;
i j k mà
1 i j k n
    .
SỞ GIÁO DỤC VÀ ĐÀO TẠO
HÀ NỘI
ĐỀ CHÍNH THỨC
Đề thi gồm 01 trang
Đề số 14
KỲ THI VÀO LỚP 10 CHUYÊN KHOA HỌC TỰ NHIÊN
NĂM HỌC 2016 – 2017
Môn: TOÁN (VÒNG 2)
Thời gian làm bài: 150 phút
(Không kể thời gian giao đề)
Website:
Liên hệ tài liệu word môn toán: TÀI LIỆU TOÁN HỌC
Câu 1. (3,0 điểm).
1). Giả sử là hai số thực phân biệt thỏa mãn .
a). Chứng minh rằng .
b). Chứng minh rằng .
2). Giải hệ phương trình .
Câu 2. (3,0 điểm).
1). Tìm các số nguyên không nhỏ hơn 2 sao cho chia hết cho .
2). Với là những số thực thỏa mãn đẳng thức Tìm giá trị lớn nhất và nhỏ
nhất của biểu thức .
Câu 3. (3,0 điểm). Cho tam giác nhọn không cân có tâm đường tròn nội tiếp là điểm .
Đường thẳng cắt tại . Gọi lần lượt là các điểm đối xứng của qua .
1). Chứng minh rằng song song với .
2). Gọi lần lượt là trung điểm các đoạn thẳng . Đường tròn ngoại tiếp tam
giác cắt đường tròn ngoại tiếp tam giác tại khác . Chứng minh rằng bốn điểm
cùng nằm trên một đường tròn.
3). Chứng minh rằng ba điểm thẳng hàng.
Câu 4. (1,0 điểm).
1). Cho bảng ô vuông 2015 x2015. Kí hiệu ô là ô ở hang thứ ,
cột thứ . Ta viết các số nguyên dương từ 1 đến 2015 vào các ô của
bảng theo quy tắc sau:
i). Số 1 được viết vào ô (1,1)
ii). Nếu số được viết vào ô ( ) thì số được viết vào
ô .
iii). Nếu số được viết vào ô thì số được viết vào ô (xem hình 1).
Khi đó số 2015 được viết vào ô .
Hãy xác định và
2). Giả sử là các số thực dương thỏa mãn
Chứng minh rằng .
;
a b 2 2
3 3 2
a a b b
   
3
a b
 
3 3
45
a b
 
2 2 2
2 3 5
4 5
x y xy
x y xy

  


  


 
;
x y 1
xy   
1 1
x y
 
;
x y
2 2
2 1 0.
x y y
  
3 1
xy
P
y


ABC
 I
AI BC D ;
E F D ;
IC IB
EF BC
; ;
M N J ; ;
DE DF EF
AEM
 AFN P A
; ; ;
M N P J
; ;
A J P
( ; )
i j i
j
k ( ; )
i j 1
i  1
k 
( 1; 1)
i j
 
k (1; )
j 1
k  ( 1; 1)
j 
 
;
m n
m .
n
; ;
a b c 4.
ab bc ac abc
   
 
2 2 2
2
a b c a b c ab bc ac
       
SỞ GIÁO DỤC VÀ ĐÀO TẠO
HÀ NỘI
ĐỀ CHÍNH THỨC
Đề thi gồm 01 trang
Đề số 15
KỲ THI VÀO LỚP 10 CHUYÊN KHOA HỌC TỰ NHIÊN
NĂM HỌC 2015 – 2016
Môn: TOÁN (VÒNG 1)
Thời gian làm bài: 120 phút
(Không kể thời gian giao đề)
Website:
Liên hệ tài liệu word môn toán: TÀI LIỆU TOÁN HỌC
Câu 1. (3,0 điểm).
1) Với là các số thỏa mãn
.
Chứng minh rằng .
2) Giải hệ phương trình .
Câu 2. (3,0 điểm).
1) Tìm số tự nhiên để và là số chính phương (số chính phương là bình phương
của một số nguyên)
2) Tìm nguyên thỏa mãn đẳng thức .
3) Giả sử là các số thực lớn hơn 2. Tìm giá trị nhỏ nhất của biểu thức:
.
Câu 3. (3,0 điểm). Cho tam giác nhọn không cân với . Gọi là trung điểm của
đoạn thẳng . Gọi là hình chiếu vuông góc của trên đoạn . Trên tia đối của tia
lấy điểm sao cho .
1) Chứng minh rằng .
2) Gọi là điểm đối xứng với qua . Đường thẳng cắt tại .Chunwgs minh rằng
bốn điểm cùng thuộc một đường tròn,gọi đường tròn này là .
3) Đường tròn ngoại tiếp tam giác cắt tại và . Chứng minh rằng song song
với .
Câu 4. (1,0 điểm). Ký hiệu là tập hợp gồm diểm phân biệt trên một mặt phẳng. Giả sử tất
cả các điểm của không cùng nằm trên một đường thẳng. Chứng minh rằng có ít nhất
đường thẳng phân biệt mà mỗi đường thẳng đi qua ít nhất hai điểm của .
; ;
a b c
       
3 3 3 3
3 3 3 24 3 3 3
a b c a b c b c a c a b
           
   
2 2 2 1
a b b c c a
   
  3 3 2
2 2 5
27 7 26 27 9
x y xy
x y y x x x

   


      


n 5
n 30
n
 
;
x y 1 3
x y x y
    
; ;
x y z
4 4 4
y
x z
P
y z z x x y
  
     
ABC
 AB AC
 M
BC H B AM AM
N 2
AN MH

BN AC

Q A N AC BQ D
; ; ;
B D N C ( )
O
AQD
 ( )
O G D NG
BC
S 2015
S 2015
S
SỞ GIÁO DỤC VÀ ĐÀO TẠO
HÀ NỘI
ĐỀ CHÍNH THỨC
Đề thi gồm 01 trang
Đề số 16
KỲ THI VÀO LỚP 10 CHUYÊN KHOA HỌC TỰ NHIÊN
NĂM HỌC 2015 – 2016
Môn: TOÁN (VÒNG 2)
Thời gian làm bài: 150 phút
(Không kể thời gian giao đề)
Website:
Liên hệ tài liệu word môn toán: TÀI LIỆU TOÁN HỌC
Câu 1. 1) Giải phương trình ( )( )
2
1 1 2 2 1 8.
+ + − + − =
x x x
2) Giải hệ phương trình
2 2
2 2
1
2 4
 − + =


+ + =


x xy y
x xy y
Câu 2. 1) Giả sử x, y, z là ba số dương thỏa mãn điều kiện x + y + z = xyz. Chứng minh rằng:
( )
( )( )( )
2 2 2
5 4 3
2 3
.
1 1 1
+ +
+ + =
+ + +
+ + +
xyz x y z
x y z
x y y z z x
x y z
2) Tìm nghiệm nguyên của phương trình: ( )
2 2
3 .
+ + + + +
x y x y x y xy
Câu 3. Cho tam giác ABC nhọn với AB < BC. Gọi D là điểm thuộc cạnh BC sao cho AD là phân
giác của ∠BAC . Đường thẳng qua C song song với AD cắt trung trực của AB tại F.
1) Chứng minh tam giác ABF đồng dạng với tam giác ACE.
2) Chứng minh rằng các đường thẳng BE, CF, AD đồng quy tại một điểm, gọi điểm đó là G.
3) Đường thẳng qua G song song với AE cắt đường thẳng BF tại Q. Đường thẳng QE cắt đường
tròn ngoại tiếp tam giác GEC tại P khác E. Chứng minh rằng các điểm A, P, G, Q, F cùng thuộc
một đường tròn.
Câu 4. Giả sử a, b, c là các số thực dương thỏa mãn đẳng thức ab + bc + ca = 1 . Chứng minh rằng
( ) 4 2 4 2 4 2
5
2 .
9
+ + ≤ + + +
abc a b c a b b c c a
SỞ GIÁO DỤC VÀ ĐÀO TẠO
HÀ NỘI
ĐỀ CHÍNH THỨC
Đề thi gồm 01 trang
Đề số 17
KỲ THI VÀO LỚP 10 CHUYÊN KHOA HỌC TỰ NHIÊN
NĂM HỌC 2014 – 2015
Môn: TOÁN (VÒNG 1)
Thời gian làm bài: 120 phút
(Không kể thời gian giao đề)
Website:
Liên hệ tài liệu word môn toán: TÀI LIỆU TOÁN HỌC
Câu 1.
1. Giả sử x, y là những số thực dương thỏa mãn:
2 4 4
2 2 4 4 8 4
2 4 8
4
y y y y
x y x y x y x y
+ + + =
+ + + −
Chứng minh rằng: 5y = 4x
2. Giải hệ phương trình:
2 2
2 2
2 3 12
6 12 6
x y xy
x x y y y x
 − + =


+ = + +


Câu 2.
1. Cho x, y là những số nguyên lớn hơn 1 sao cho 4x2
y2
– 7x + 7y là số chính phương.
Chứng minh rằng: x = y.
2. Giả sử x, y là những số thực không âm thỏa mãn: x3
+ y3
+ xy = x2
+ y2
Tìm giá trị lớn
nhất và nhỏ nhất của biểu thức:
1 2
2 1
x x
P
y y
+ +
= +
+ +
Câu 3. Cho tam giác ABC nội tiếp đường tròn (O) và điểm P nằm trong tam giác thỏa mãn PB =
PC. D là điểm thuộc cạnh BC (D khác B và D khác C) sao cho P nằm trong đường tròn ngoại tiếp
tam giác DAC và đường tròn ngoại tiếp tam giác DAC. Đường thẳng PB cắt đường tròn ngoại
tiếp tam giác DAB tại E khác B. Đường thẳng PC cắt đường tròn ngoại tiếp tam giác DAC tại F
khác C.
1. Chứng minh rằng bốn điểm A, E, B, F cùng thuộc một đường tròn.
2. Giả sử đường thẳng AD cắt đường tròn (O) tại Q khác A, đường thẳng AF cắt đường thẳng
QC tại L. Chứng minh rằng tam giác ABE đồng dạng với tam giác CLF.
3. Gọi K là giao điểm của đường thẳng AE và đường thẳng QB. Chứng minh rằng:
   
QKL PAB QLK PAC
+ = + .
Câu 4. Cho tập hợp A gồm 31 phần tử và dãy gồm m tập hợp của A thỏa mãn đồng thời các
điều kiện sau:
i) Mỗi tập hợp thuộc dãy có ít nhất hai phần tử.
ii) Nếu hai tập hợp thuộc dãy có chung nhau ít nhất hai phần tử thì số phần tử của
hai tập hợp này khác nhau.
Chứng minh rằng: m ≤ 900
SỞ GIÁO DỤC VÀ ĐÀO TẠO
HÀ NỘI
ĐỀ CHÍNH THỨC
Đề thi gồm 01 trang
Đề số 18
KỲ THI VÀO LỚP 10 CHUYÊN KHOA HỌC TỰ NHIÊN
NĂM HỌC 2014 – 2015
Môn: TOÁN (VÒNG 2)
Thời gian làm bài: 150 phút
(Không kể thời gian giao đề)
Website:
Liên hệ tài liệu word môn toán: TÀI LIỆU TOÁN HỌC
Câu 1. (3,0 điểm).
1). Giải phương trình
.
2). Giải hệ phương trình
.
Câu 2. (3,0 điểm).
1). Cho các số thực thỏa mãn . Chứng minh rằng
.
2). Có bao nhiêu số nguyên dương có chữ số sao cho chia hết cho ?
Câu 3. (3,0 điểm). Cho tam giác nhọn nội tiếp đường tròn với Đường phân
giác của góc cắt tại điểm khác Gọi là trung điểm của và là điểm đối
xứng với qua tâm . Giả sử đường tròn ngoại tiếp tam giác cắt đoạn thẳng tại
điểm khác
1). Chứng minh rằng tam giác và tam giác đồng dạng.
2). Chứng minh rằng vuông góc với
Câu 4. (1,0 điểm).
Giả sử là các số thực dương thỏa mãn điều kiện Tìm giá
trị nhỏ nhất của biểu thức .
3 1 2 3
x x
   
1 1 9
2
1 3 1 1
4 2
x y
x y
x xy
y xy


    




  
 
  
   

 

 

 



; ; 0
a b c      8
a b b c c a abc
   
        
3
4
a b c ab bc ca
a b b c c a a b b c b c c a c a a b
     
        
5 abcde  
10
abc d e
  101
ABC
 ( )
O .
AB AC


BAC ( )
O D .
A M AD E
D O ABM
 AC
F .
A
BDM
 BCF

EF .
AC
; ; ;
a b c d 1.
abc bcd cda dab
   
 
3 3 3 3
4 9
P a b c d
   
SỞ GIÁO DỤC VÀ ĐÀO TẠO
HÀ NỘI
ĐỀ CHÍNH THỨC
Đề thi gồm 01 trang
Đề số 19
KỲ THI VÀO LỚP 10 CHUYÊN KHOA HỌC TỰ NHIÊN
NĂM HỌC 2013 – 2014
Môn: TOÁN (VÒNG 1)
Thời gian làm bài: 120 phút
(Không kể thời gian giao đề)
Website:
Liên hệ tài liệu word môn toán: TÀI LIỆU TOÁN HỌC
Câu 1. (3,0 điểm).
1). Giải hệ phương trình
.
2). Giải phương trình
.
Câu 2. (3,0 điểm).
1). Tìm các cặp số nguyên thỏa mãn
.
2). Với là các số thực dương thỏa mãn tìm giá trị nhỏ nhất của biểu thức
.
Câu 3. (3,0 điểm). Cho tam giác nhọn nội tiếp đường tròn có trực tâm . Gọi là
điểm nằm trên đường tròn ngoại tiếp tam giác ( khác và ) và nằm trong tam
giác . cắt tại khác cắt tại khác . cắt tại cắt
tại . Đường tròn ngoại tiếp tam giác và đường tròn ngoại tiếp tam giác cắt nhau
tại khác
1). Chứng minh rằng ba điểm thẳng hàng.
2). Giả sử là phân giác góc Chứng minh rằng khi đó đi qua trung điểm của
Câu 4. (1,0 điểm). Giả sử dãy số thực có thứ tự thỏa mãn các điều kiện
và
Chứng minh rằng
3 3
1
7 7
x y y x xy
xy y x

     


   


2
3 1 3 1 1
x x x x
      
 
;
x y
2 2
5 8 20412
x y
 
 
;
x y 1,
x y
 
2 2
1 1
1
P x y
x y
 

 
  
 
 

 
ABC
 ( )
O H P
HBC
 P ,
B C H
ABC
 PB ( )
O M ,
B PC ( )
O N C BM AC ,
E CN AB
F AME
 ANF

Q .
A
; ;
M N Q
AP .
MAN PQ .
BC
1 2 192
x x x
  

1 2 192 0
x x x
   
 1 2 192 2013.
x x x
   

192 1
2013
.
96
x x
 
SỞ GIÁO DỤC VÀ ĐÀO TẠO
HÀ NỘI
ĐỀ CHÍNH THỨC
Đề thi gồm 01 trang
Đề số 20
KỲ THI VÀO LỚP 10 CHUYÊN KHOA HỌC TỰ NHIÊN
NĂM HỌC 2013 – 2014
Môn: TOÁN (VÒNG 2)
Thời gian làm bài: 150 phút
(Không kể thời gian giao đề)
Website:
Liên hệ tài liệu word môn toán: TÀI LIỆU TOÁN HỌC
Câu 1. (3,0 điểm).
1). Giải phương trình: .
2). Giải hệ phương trình .
Câu 2. (3,0 điểm).
1). Tìm tất cả các cặp số nguyên thỏa mãn đẳng thức
.
2). Giả sử là các số thực dương thỏa mãn điều kiện . Tìm giá trị nhỏ nhất
của biểu thức
.
Câu 3. (3,0 điểm). Cho tam giác nhọn nội tiếp đường tròn tâm . Gọi là một điểm trên
cung nhỏ ( khác và không đi qua ). Giả sử là một điểm thuộc đoạn thẳng
sao cho đường tròn đường kính cắt cung nhỏ tại điểm khác .
1). Gọi là điểm đối xứng với điểm qua . Chứng minh rằng ba điểm thẳng hàng.
2). Đường tròn đường kính cắt tại điểm khác . Chứng minh rằng là tâm đường
tròn nội tiếp tam giác .
Câu 4. (1,0 điểm). Giả sử là các số thực dương thỏa mãn: ; ; .
Tìm giá trị nhỏ nhất của biểu thức: .
  
9 2012 6 2012 9 6
x x x x
      
2 2
2 4
2 4
x y y
x y xy

   


   


 
;
x y
    
1 5 2
x y xy x y x y
      
 
;
x y   
1 1 4
x y
  
2
2
y
x
P
y x
 
ABC
 O M

BC M ;
B C AM O P AM
MP 
BC N M
D M O ; ;
N P D
MP MD Q M P
AQN

; ;
a b c 3
a b c
   1
c b
  a b c
 
 
   
2 1
1 1 1
ab a b c ab
a b c
Q
   
  

SỞ GIÁO DỤC VÀ ĐÀO TẠO
HÀ NỘI
ĐỀ CHÍNH THỨC
Đề thi gồm 01 trang
Đề số 21
KỲ THI VÀO LỚP 10 CHUYÊN KHOA HỌC TỰ NHIÊN
NĂM HỌC 2012 – 2013
Môn: TOÁN (VÒNG 1)
Thời gian làm bài: 120 phút
(Không kể thời gian giao đề)
Website:
Liên hệ tài liệu word môn toán: TÀI LIỆU TOÁN HỌC
Câu 1. (3,0 điểm).
1). Giải hệ phương trình .
2). Giải phương trình .
Câu 2. (3,0 điểm).
1). Tìm hai chữ số cuối cùng của số .
2). Tìm giá trị lớn nhất của hàm số , với .
Câu 3. (3,0 điểm). Cho tam giác nhọn ( ) nội tiếp đường tròn . Giả sử là
hai điểm thuộc cung nhỏ sao cho song song với và tia nằm giữa hai tia
. Gọi là hình chiếu vuông góc của điểm trên và là hình chiếu vuông góc của
điểm trên .
1). Giả sử cắt tại điểm . Chứng minh nằm trên đường tròn .
2). Gọi giao điểm của và là khác . Giả sử cắt tại . Chứng minh rằng bốn
điểm cùng thuộc một đường tròn.
Câu 4. (1,0 điểm). Với mỗi số nguyên lớn hơn hoặc bằng 2 cố định, xét các tập số thực đôi
một khác nhau . Kí hiệu là số các giá trị khác nhau của tổng
( ). Tìm giá trị lớn nhất và giá trị nhỏ nhất của .
 
  3 3
2
9 3 6 26 2
xy x y
xy x y x y

  



    



  
4 2 4 2 2
x x x
    
106 2012
41 57
A  
2
3 2 1 5 4
y x x x
   
1 5
2 2
x
 
ABC
 AB AC
 ( )
O ;
M N

BC MN BC AN
;
AM AB P C AN Q
M AB
CP QM T T ( )
O
NQ ( )
O R N AM PQ S
; ; ;
A R Q S
n n
 
1 2
; ;...; n
X x x x
  
C X i j
x x

1 i j n
    
C X
SỞ GIÁO DỤC VÀ ĐÀO TẠO
HÀ NỘI
ĐỀ CHÍNH THỨC
Đề thi gồm 01 trang
Đề số 22
KỲ THI VÀO LỚP 10 CHUYÊN KHOA HỌC TỰ NHIÊN
NĂM HỌC 2012 – 2013
Môn: TOÁN (VÒNG 2)
Thời gian làm bài: 150 phút
(Không kể thời gian giao đề)
Website:
Liên hệ tài liệu word môn toán: TÀI LIỆU TOÁN HỌC
Câu 1 (3,0 điểm).
1). Giải hệ phương trình .
2). Giải phương trình .
Câu 2. (3,0 điểm).
1). Chứng minh rằng không tồn tại các bộ ba số nguyên thỏa mãn đẳng thức:
.
2). Tìm tất cả các cặp số nguyên thỏa mãn đẳng thức
.
Câu 3. (3,0 điểm). Cho hình bình hành với . Đường phân giác của góc cắt
đường tròn ngoại tiếp tam giác tại khác . Kẻ đường thẳng đi qua và vuông góc
với . Đường thẳng lần lượt cắt các đường thẳng tại .
1). Chứng minh rằng .
2). Chứng minh rằng là tâm đường tròn ngoại tiếp tam giác .
3). Gọi giao điểm của và là , chứng minh rằng
.
Câu 4. (1,0 điểm). Với là những số thực dương, tìm giá trị nhỏ nhất của biểu thức
.
  2
2
1 3
( 2) 1
x y x y
y x y x









   
   
 
2
3 7
2 1
x
x
x
x




 
; ;
x y z
4 4 4
7 5
x y z
  
 
;
x y
   
4 4 3
1 1
x x y
   
ABCD  90
BAD   
BCD
BCD
 O C d A
CO d ;
CB CD ;
E F
OBE ODC
 
O CEF

OC BD I
. . . .
IB BE EI ID DF FI

;
x y
 
3
3
3 3 3
3
4
8
y
x
x y y x y
P 
  

SỞ GIÁO DỤC VÀ ĐÀO TẠO
HÀ NỘI
ĐỀ CHÍNH THỨC
Đề thi gồm 01 trang
Đề số 23
KỲ THI VÀO LỚP 10 CHUYÊN KHOA HỌC TỰ NHIÊN
NĂM HỌC 2011 – 2012
Môn: TOÁN (VÒNG 1)
Thời gian làm bài: 120 phút
(Không kể thời gian giao đề)
Website:
Liên hệ tài liệu word môn toán: TÀI LIỆU TOÁN HỌC
Câu 1. (3,0 điểm).
1). Giải phương trình : .
2). Giải hệ phương trình: .
Câu 2. (3,0 điểm).
1). Với mỗi số thực ta gọi phần nguyên của là số nguyên lớn nhất không vượt quá và ký
hiệu là . Chứng minh rằng với mọi số nguyên dương , biểu thức không
biểu diễn được dưới dạng lập phương của một số nguyên dương.
2). Với là các số thực dương thỏa mãn đẳng thức , Tìm giá trị nhỏ nhất của
biểu thức : .
Câu 3. (3,0 điểm). Cho hình thang với song song . Các góc và là các góc
nhọn. Hai đường chéo và cắt nhau tại . là điểm bất kỳ trên đoạn thẳng ( không
trùng với ). Giả sử đường tròn ngoại tiếp tam giác cắt đoạn thẳng tại khác và
đường tròn ngoại tiếp tam giác cắt đoạn thẳng tại khác
1). Chứng minh rằng năm điểm cùng nằm trên một đường tròn. Gọi đường tròn
này là .
2). Giả sử các đường thẳng và cắt nhau tại , chứng minh rằng cũng nằm trên
đường tròn .
3). Trong trường hợp thẳng hàng, chứng minh rằng .
Câu 4. (1,0 điểm). Giả sử là một tập con của tập các số tự nhiên . Tập có phần tử nhỏ nhất
là 1, phần tử lớn nhất là 100 và mỗi thuộc ( ) luôn tồn tại cũng thuộc sao cho
( có thể bằng ). Hãy tìm một tập có số phần tử nhỏ nhất.
  
1 1
3 1
x
x x  
  
  
2 2 2 2
2 2
1
2
4
x y xy
x y x y
x y





  



 

a a a
a
 
  n
2
3
1 1
27 3
n n
 
 
  
 
 
 
; ;
x y z 5
xy yz zx
  
     
2 2 2
3 3 2
6 5 6 5 6 5
x y z
P
x y z
 

    
ABCD BC AD

BAD 
CDA
AC BD I P BC P
;
B C BIP
 PA M P
CIP
 PD N P
; ; ; ;
A M I N D
( )
K
BM CN Q Q
( )
K
; ;
P I Q
PB BD
PC CA

A  A
x A 1
x  ;
a b A
x a b
  a b A
SỞ GIÁO DỤC VÀ ĐÀO TẠO
HÀ NỘI
ĐỀ CHÍNH THỨC
Đề thi gồm 01 trang
Đề số 24
KỲ THI VÀO LỚP 10 CHUYÊN KHOA HỌC TỰ NHIÊN
NĂM HỌC 2011 – 2012
Môn: TOÁN (VÒNG 2)
Thời gian làm bài: 150 phút
(Không kể thời gian giao đề)
Website:
Liên hệ tài liệu word môn toán: TÀI LIỆU TOÁN HỌC
Câu I.
1) Giải hệ phương trình





=
+
=
+
+
.
2
23
12
8
3
2
2
2
2
y
x
xy
y
x
2) Giải phương trình
.
1
8
3
1
2
4
3
1
2 3
2
+
+
=
+
−
+
+ x
x
x
x
Câu II.
1) Tìm tất cả các số nguyên không âm (x, y) thoả mãn đẳng thức
( )( ) ( )( ) .
25
1
2
4
1
1 2
2
=
+
+
+
+
+
+ xy
y
x
xy
y
x
2) Với mỗi số thực a, ta gọi phần nguyên của số a là số nguyên lớn nhất không vượt quá
a và ký hiệu là [a]. Chứng minh rằng với mọi n nguyên dương ta luôn có.
( )
n
n
n
n
n
=






+
+
+
+
+
1
1
...
3
.
2
7
2
.
1
3 2
Câu III.
Cho đường tròn (O) với đường kính AB = 2R. Trên đường thẳng tiếp xúc với đương tròn
(O) tại A ta lấy điểm C sao cho góc 0
30
=
ACB . Gọi H là giao điểm thứ hai của đường thăng BC
với đường tròn (O).
1) Tính độ dài đương thẳng AC, BC và khoảng cách từ A đến đương thẳng BC theo R.
2) Với mỗi điểm M trên đoạn thẳng AC, đường thẳng BM cắt đường tròn (O tại điểm N
(khác B). Chứng minh rằng bốn điểm C, M, N, H nằm trên cùng một đường tròn và
tâm đường tròn đó luôn chạy trên một đường thẳng cố định khi M thay đổi trên đoạn
thẳng AC.
Câu IV. Với a,b là các số thực thoả mãn đẳng thức
4
9
)
1
)(
1
( =
+
+ b
a , hãy tìm giá trị nhỏ nhất của
biểu thức 4
4
1
1 b
a
P +
+
+
= .
SỞ GIÁO DỤC VÀ ĐÀO TẠO
HÀ NỘI
ĐỀ CHÍNH THỨC
Đề thi gồm 01 trang
Đề số 25
KỲ THI VÀO LỚP 10 CHUYÊN KHOA HỌC TỰ NHIÊN
NĂM HỌC 2010 – 2011
Môn: TOÁN (VÒNG 1)
Thời gian làm bài: 120 phút
(Không kể thời gian giao đề)
Website:
Liên hệ tài liệu word môn toán: TÀI LIỆU TOÁN HỌC
Câu I.
1) Giải phương trình x 3 3x 1 4
+ + + =
2) Giải hệ phương trình
( )( )
2 2
5x 2y 2xy 26
3x 2x y x y 11.
 + + =


+ + − =


Câu II.
1) Tìm tất cả các số nguyên dương n để 2
n 391
+ là số chính phương.
2) Giả sử x, y, z là những số thực dương thỏa mãn điều kiện x + y + z = 1. Chứng minh rằng:
2 2
xy z 2x 2y
1.
1 xy
+ + +
≥
+
Câu III.
Cho tam giác ABC có ba góc nhọn và M là điểm nằm trong tam giác. Kí hiệu H là hình
chiếu của M trên cạnh BC và P, Q, E, F lần lượt là hình chiếu của H trên các đường thẳng MB,
MC, AB, AC. Giả sử bốn điểm P, Q, E, F thẳng hàng.
1) Chứng minh rằng M là trực tâm của tam giác ABC.
2) Chứng minh rằng BEFC là tứ giác nội tiếp.
Câu IV. Trong dãy số gồm 2010 số thực khác 0 được sắp xếp theo thứ tự 1 2 3 2010
a ,a ,a ,...,a ta
đánh dấu tất cả các số dương và tất cả các số mà tổng của nó với một số liên tiếp liền ngay sau
nó là một số dương.
Chứng minh rằng nếu trong dãy số đã cho có ít nhất một số dương thì tổng của tất cả các số
được đánh dấu là một số dương.
SỞ GIÁO DỤC VÀ ĐÀO TẠO
HÀ NỘI
ĐỀ CHÍNH THỨC
Đề thi gồm 01 trang
Đề số 26
KỲ THI VÀO LỚP 10 CHUYÊN KHOA HỌC TỰ NHIÊN
NĂM HỌC 2010 – 2011
Môn: TOÁN (VÒNG 2)
Thời gian làm bài: 150 phút
(Không kể thời gian giao đề)
Website:
Liên hệ tài liệu word môn toán: TÀI LIỆU TOÁN HỌC
Câu I. 1) Giải phương trình
1
2
2 2
2
+
−
=
+
− x
x
x
x
2) Giải hệ phương trình





+
=
+
=
+
−
3
3
1
2
2
2
y
y
x
xy
y
x
Câu II. 1) Tìm chữ số tận cùng của số 2009
6
13
2009
6
13 +
+
2) Với a, b là những chữ số thực dương, tìm giá trị nhỏ nhất của biểu thức
)
5
4
(
)
5
4
( a
b
b
b
a
a
b
a
P
+
+
+
+
=
Câu III. Cho hình thoi ABCD. Gọi H là giao điểm của hai đường chéo AC và BD. Biết rằng bán
kính đường tròn ngoại tiếp tam giác ABC bằng a và bán kính đường tròn ngoại tiếp tam giác
ABD bằng b.
1) Chứng minh rằng
b
a
BH
AH
=
2) Tính diện tích hình thoi ABCD theo các bán kính a, b
Câu IV. Với a, b, c là những số thực dương, chứng minh rằng
5
14
8
3
14
8
3
14
8
3 2
2
2
2
2
2
2
2
2
c
b
a
ca
a
c
c
bc
c
b
b
ab
b
a
a +
+
≥
+
+
+
+
+
+
+
+
SỞ GIÁO DỤC VÀ ĐÀO TẠO
HÀ NỘI
ĐỀ CHÍNH THỨC
Đề thi gồm 01 trang
Đề số 27
KỲ THI VÀO LỚP 10 CHUYÊN KHOA HỌC TỰ NHIÊN
NĂM HỌC 2009 – 2010
Môn: TOÁN (VÒNG 1)
Thời gian làm bài: 120 phút
(Không kể thời gian giao đề)
Website:
Liên hệ tài liệu word môn toán: TÀI LIỆU TOÁN HỌC
Câu I. 1) Giải phương trình 35
36
84
1
6
35
14 2
+
+
+
=
+
+
+ x
x
x
x
2) Chứng minh rằng
1
4
)
1
2
(
4
1
2
...
3
4
3
1
4
1
2
2
4
4
4
+
=
−
+
−
+
+
+
+
+ n
n
n
n
Với mọi n nguyên dương
Câu II. 1) Tìm số nguyên dương n sao cho tất cả các số
n + 1, n + 5, n + 7, n + 13, n + 17, n + 25, n + 37 đều là nguyên tố
2) Mỗi lần cho phép thay thế cặp số (a,b) thuộc tập hợp
{ }
)
8
,
78
(
),
62
,
6
(
),
32
,
4
(
),
2
,
16
(
=
M bằng cặp số (a + c, b + d) trong đó cặp
số (c, d) cũng thuộc M.
Hỏi sau một số hữu hạn lần thay thế ta có thể nhận được tập hợp các cặp
số { }
)
912
,
2240
(
),
2176
,
1056
(
),
2104
,
844
(
),
702
,
2018
(
1 =
M hay không?
Câu III. Cho đường tròn (O) và (O’) cắt nhau tại hai điểm A và B. Trên đường thẳng AB ta lấy
một điểm M bất kỳ sao cho điểm A nằm trong đoạn BM ( )
A
M ≠ .Từ điểm M kẻ tới đường tròn
(O’) các tiếp tuyến MC và MD (C và D là các tiếp điểm, C nằm ngoài (O)). Đường thẳng AC cắt
lần thứ hai đường tròn (O) tại điểm P và đường thẳng AD cắt lần thứ hai đường tròn (O) tại Q.
Đường thẳng CD cắt PQ tại K.
1) Chứng minh rằng hai tam giác BCD và BPQ đồng dạng
2) Chứng minh rằng khi M thay đổi thì đường tròn ngoại tiếp tam giác KCP luôn đi qua điểm cố
định.
Câu IV. Giả sử x,y,z là những số thực thoả mãn điều kiện
2
,
,
0 ≤
≤ z
y
x và x+ y + z = 3
Tìm giá trị nhỏ nhất và lớn nhất của biểu thức :
( ) )
1
)(
1
(
1
12
4
4
4
z
y
x
z
y
x
M −
−
−
+
+
+
=
SỞ GIÁO DỤC VÀ ĐÀO TẠO
HÀ NỘI
ĐỀ CHÍNH THỨC
Đề thi gồm 01 trang
Đề số 28
KỲ THI VÀO LỚP 10 CHUYÊN KHOA HỌC TỰ NHIÊN
NĂM HỌC 2009 – 2010
Môn: TOÁN (VÒNG 2)
Thời gian làm bài: 150 phút
(Không kể thời gian giao đề)
Website:
Liên hệ tài liệu word môn toán: TÀI LIỆU TOÁN HỌC
Đề số 29- MÔN : TOÁN (Vòng 1)
Thời gian làm bài: 150 phút (không kể thời gian phát đề)
Câu 1.
a) Giải hệ phương trình:
2 2
3 3
2 1
8 7
 − =

− =

x y y x
x y
.
b) Cho 0 1
≤ ≤
x . Tìm giá trị lớn nhất của biểu thức: ( )
2 1
=
+ −
y x x .
Câu 2.
a) Tìm tất cả các số nguyên ,
x y thỏa mãn: 2 2
2 3 3 2 2 0
+ + + + + =
x y xy x y .
b) Tìm số nguyên dương , ,
a b c sao cho
( )( )( )
1 1 1
− − −
ab bc ca
abc
là một số nguyên.
Câu 3.
a) Tìm nghiệm nguyên của phương trình: 2 2 2 2
8 10
+ + =
x y x y xy .
b) Ký hiệu [ ]
x là phần nguyên của số x (số nguyên lớn nhất không vượt quá x ). Chứng minh rằng với
mọi số tự nhiên n , ta luôn có:
] [ ] [
3 3 3 3
72 1 9 9 1 72 7
 
+ = + + = +
 
n n n n
Câu 4. Cho tam giác ABC nội tiếp đường tròn ( )
O . Giả sử các tiếp tuyến với đường tròn ( )
O tại B và
C cắt nhau tại P nằm khác phía với A so với BC . Trên cung BC không chứa A lấy điểm ( )
,
≠
K K B C
. Đường thẳng PK cắt ( )
O tại điểm thứ hai Q khác A .
a) Chứng minh rằng các đường phân giác của các góc ,
∠ ∠
KBQ KCQ đi qua cùng một điểm trên đường
thẳng PQ.
b) Giả sử đường thẳng AK đi qua trung điểm M của cạnh BC . Chứng minh rằng 
AQ BC .
Câu 5.
Cho phương trình: 1 2
0 1 2 1 0
− −
−
+ + +…+ + =
n n n
n n
a x a x a x a x a thỏa mãn các hệ số 0 1 2
, , , ,
… n
a a a a chỉ nhận
một trong ba giá trị: 0 , hoặc 1 , hoặc 1
− và 0 0
≠
a .
Chứng minh rằng nếu 0
x là nghiệm của phương trình thì 0 2
<
x
ĐẠI HỌC QUỐC GIA HÀ NỘI
TRƯỜNG ĐẠI HỌC KHOA HỌC TỰ NHIÊN
ĐỀ THI TUYỂN SINH LỚP 10
HỆ THPT CHUYÊN NĂM 2008
Website:
Tài liệu sưu tầm và tổng hợp bản word đầy đủ liên hệ TÀI LIỆU TOÁN HỌC
32
Đề số 30- MÔN : TOÁN (Vòng 1)
Thời gian làm bài: 150 phút (không kể thời gian phát đề)
Câu 1. (3 điểm) Giải phương trình và hệ phương trình:
a) 2 2
4 1 2 2 1
− + = − + +
x x x x x
b)
( )
3 3
2
4
 + =

+ + + =

xy x y
x y x y
Câu 2. (3 điểm)
a) Giả sử 1 2
,
x x là hai nghiệm của phương trình 2
4 1 0.
− + =
x x Chứng minh rằng 5 5
1 2
+
x x là một số
nguyên.
b) Cho a, b là các số nguyên dương thỏa mãn a + 1 và b + 2007 đều chia hết cho 6.
Chứng minh rằng 4a + a + b chia hết cho 6.
Câu 3. (3 điểm) Cho M là trung điểm của cung nhỏ AB của đường tròn tâm O (AB không phải là
đường kính). C và D là 2 điểm phân biệt, thay đổi nằm giữa A và B. Các đường thẳng MC, MD cắt
(O) tương ứng tại E, F khác M.
a) Chứng minh các điểm C, D, E, F nằm trên một đường tròn.
b) Gọi 1
O và 2
O lần lượt là tâm các đường tròn ngoại tiếp các tam giác ACE và BDF.
Chứng minh rằng khi C và D thay đổi trên đoạn AB thì giao điểm của hai đường thẳng 1
AO
và 2
BO là một điểm cố định.
Câu 4. (1 điểm) Cho a, b, c là các số thực dương thỏa mãn abc = 1. Chứng minh rằng:
( ) ( ) ( )
2 2 2
1
1 1 1
+ + ≥
+ +
+ + + + + +
a b c
a b c
ab a bc b ca c
ĐẠI HỌC QUỐC GIA HÀ NỘI
TRƯỜNG ĐẠI HỌC KHOA HỌC TỰ NHIÊN
ĐỀ THI TUYỂN SINH LỚP 10
HỆ THPT CHUYÊN NĂM 2007
Website:
Tài liệu sưu tầm và tổng hợp bản word đầy đủ liên hệ TÀI LIỆU TOÁN HỌC
33
Đề số 31- MÔN : TOÁN (Vòng 2)
Thời gian làm bài: 150 phút (không kể thời gian phát đề)
Câu 1.
a) Giải hệ phương trình:
2 2
4 5
2 4 7
 + =

+ + =

x y
x y xy
.
b) Cho , ,
a b c là các số thực khác 0 . Tìm giá trị nhỏ nhất của biểu thức:
2 2
2
2 2 2
1 1
.
+
 
= + +
 
 
b c
P a
b c a
Câu 2.
a) Tìm cặp số nguyên ( )
,
x y thỏa mãn 2 2
5 17 2
+ = +
x y xy .
b) Tìm tất cả các số nguyên tố p sao cho 4
2
+
p là số nguyên tố.
Câu 3.
Cho hai đường thẳng 1
d và 2
d vuông góc tại O . Đường tròn ( )
1
O tiếp xúc với 1 2
,
d d lần lượt tại
,
A B . Đường tròn ( )
2
O tiếp xúc với 1 2
,
d d lần lượt tại ,
C D .
a) Chứng minh rằng B là trực tâm tam giác ACD .
b) Giả sử CB cắt ( )
1
O tại ,
E AD cắt ( )
2
O tại F . Chứng minh rằng ACEF là hình thang cân.
Câu 4.
Trong các tứ giác có ba cạnh đều bằng a cho trước. Tìm tứ giác diện tích lớn nhất.
Câu 5 .
Cho dãy số 0 1
, , , ,
… …
n
a a a được xác định bởi như sau: 0 0
=
a và
( )
1 2 3 1
+
= + + ∀ ∈
n n n
a a a n  .
Chứng minh rằng: ( )
2
1
(2 3) 2 3
4
 
= + − −
 
n
n
a .
ĐẠI HỌC QUỐC GIA HÀ NỘI
TRƯỜNG ĐẠI HỌC KHOA HỌC TỰ NHIÊN
ĐỀ THI TUYỂN SINH LỚP 10
HỆ THPT CHUYÊN NĂM 2007
Website:
Tài liệu sưu tầm và tổng hợp bản word đầy đủ liên hệ TÀI LIỆU TOÁN HỌC
34
Đề số 32- MÔN : TOÁN (Vòng 1)
Thời gian làm bài: 150 phút (không kể thời gian phát đề)
Câu I (2,0 điểm)
Giải hệ phương trình



=
+
+
=
+
+
+
4
)
xy
1
)(
y
x
(
4
y
x
xy
x2
Câu II (2,0 điểm)
Với những giá trị của x thỏa mãn điều kiện
2
1
x −
≥ , hãy tìm giá trị lớn nhất của biểu thức:
x
2
3
x
2
2
x
5
x
2
)
x
(
f 2
−
+
+
+
+
=
Câu III (2,0 điểm)
Tìm số tự nhiên gồm bốn chữ số thỏa mãn đồng thời hai tính chất:
(i) Khi chia số đó cho 100 ta được số dư là 6.
(ii) Khi chia số đó cho 51 ta được số dư là 17.
Câu IV (3,0 điểm)
Cho hình vuông ABCD có cạnh α
=
AB . Trên các cạnh AB, BC, CD, DA lấy lần lượt các
điểm M, N, P, Q sao cho: MN // AC, PQ // AC và
0
30
AMQ =
∠ .
1) Gọi A′ là điểm đối xứng với A qua đường thẳng MQ, C′ là điểm đối xứng với C qua
đường thẳng NP. Giả sử đường thẳng A
Q ′ cắt đoạn thẳng NP tại E, đường thẳng C
P ′ cắt
đoạn thẳng MQ tại F. chứng minh rằng năm điểm E, F, Q, D, P nằm trên cùng một đường
tròn.
2) Biết AC = 3MN, tính diện tích của hình thang MNPQ theo a.
Câu V (1,0 điểm)
Chứng minh rằng với mỗi số dương a cho trước, đa thức 2
ax
x
)
x
(
f 2
4
+
+
= luôn là tổng
bình phương của hai đa thức bậc hai.
ĐẠI HỌC QUỐC GIA HÀ NỘI
TRƯỜNG ĐẠI HỌC KHOA HỌC TỰ NHIÊN
ĐỀ THI TUYỂN SINH LỚP 10
HỆ THPT CHUYÊN NĂM 2006
Website:
Tài liệu sưu tầm và tổng hợp bản word đầy đủ liên hệ TÀI LIỆU TOÁN HỌC
35
Đề số 33- MÔN : TOÁN (Vòng 2)
Thời gian làm bài: 150 phút (không kể thời gian phát đề)
Câu 1.
Chứng minh rằng: 3 3
84 84
1 1
9 9
= + + −
A là một số nguyên.
Câu 2.
Giải hệ phương trình:
2 2
2 2
4 2 3
5
 − = − −

+ =

x y x y
x y
.
Câu 3.
a) Tìm nghiệm nguyên của phương trình: 2 2 2 2
8 10
+ + =
x y x y xy .
b) Ký hiệu [ ]
x là phần nguyên của số x (số nguyên lớn nhất không vượt quá x ). Chứng minh
rằng với mọi số tự nhiên n , ta luôn có:
] [ ] [
3 3 3 3
72 1 9 9 1 72 7
 
+ = + + = +
 
n n n n
Câu 4.
Cho tam giác ABC nội tiếp đường tròn ( )
O và I là điểm nằm trong tam giác ABC . Các đường
thẳng , ,
AI BI CI cắt ( )
O lần lượt tại , ,
′ ′ ′
A B C (khác , ,
A B C ). Dây cung ′ ′
B C cắt các cạnh
,
AB AC tương ứng tại các điểm ,
M N . Dây cung ′ ′
C A cắt các cạnh ,
AB BC tương ứng tại các
điểm ,
Q P . Dây cung ′ ′
A B cắt các cạnh ,
BC CA tương ứng tại các điểm ,
F E .
a) Giả sử , ,
= = =
AM AN BP BQ CE CF xảy ra đồng thời. Chứng minh rằng I là tâm đường tròn
nội tiếp tam giác ABC .
b) Giả sử = = = = =
AM AN BP BQ CE CF . Chứng minh rằng sáu điểm , , , , ,
M N P Q E F cùng
thuộc một đường tròn.
Câu 5.
Chứng minh rằng đa giác lồi 2n cạnh ( )
, 2
∈ ≥
n n
 luôn có ít nhất n đường chéo không không
song song với bất kỳ cạnh nào của đa giác đó.
ĐẠI HỌC QUỐC GIA HÀ NỘI
TRƯỜNG ĐẠI HỌC KHOA HỌC TỰ NHIÊN
ĐỀ THI TUYỂN SINH LỚP 10
HỆ THPT CHUYÊN NĂM 2006
Website:
Tài liệu sưu tầm và tổng hợp bản word đầy đủ liên hệ TÀI LIỆU TOÁN HỌC
36
Đề số 34- MÔN : TOÁN (Vòng 1)
Thời gian làm bài: 150 phút (không kể thời gian phát đề)
Câu 1: Giải hệ phương trình



=
+
=
+
+
2
y
x
3
xy
y
x
2
2
Câu 2: Giải phương trình
11
x
2
3
2
3
x
4
x =
−
+
+
+
Câu 3: Tìm nghiệm nguyên của phương trình
1740
)
y
x
(
51
xy
34
y
17
x 2
2
=
+
+
+
+
Câu 4:
Cho hai đường tròn (O), )
O
( ′ nằm ngoài nhau có tâm tương ứng là O và O′. Một tiếp
tuyến chung ngoài của hai đường tròn tiếp xúc với (O) tại A và )
O
( ′ tại B. Một tiếp tuyến chung
trong của hai đường tròn cắt AB tại I, tiếp xúc với (O) tại C và )
O
( ′ tại D. Biết rằng C nằm giữa I
và D.
1) Hai đường thẳng OC, B
O′ cắt nhau tại M. Chứng minh rằng OM > M
O′ .
2) Ký hiệu (S) là đường tròn đi qua A, C, B và )
S
( ′ là đường tròn đi qua A, D, B. Đường
thẳng CD cắt (S) tại E khác C và cắt )
S
( ′ tại F khác D. Chứng minh rằng AF vuông góc với BE
Câu 5:
Giả sử x, y, z là các số dương thay đổi và thỏa mãn điều kiện:
2
2
2
2
z
3
y
z
x
z
xy =
+
+ .
Hãy tìm giá trị lớn nhất của biểu thức:
( )
4
4
4
4
y
x
z
1
z
P
+
+
=
ĐẠI HỌC QUỐC GIA HÀ NỘI
TRƯỜNG ĐẠI HỌC KHOA HỌC TỰ NHIÊN
ĐỀ THI TUYỂN SINH LỚP 10
HỆ THPT CHUYÊN NĂM 2005
Website:
Tài liệu sưu tầm và tổng hợp bản word đầy đủ liên hệ TÀI LIỆU TOÁN HỌC
37
ĐẠI HỌC QUỐC GIA HÀ NỘI
TRƯỜNG ĐẠI HỌC KHOA HỌC TỰ NHIÊN
ĐỀ THI TUYỂN SINH LỚP 10
HỆ THPT CHUYÊN NĂM 2005
Đề số 35- MÔN : TOÁN (Vòng 2)
Thời gian làm bài: 150 phút (không kể thời gian phát đề)
Bài 1: Giải phương trình 2
2 2 4 2
x x x
− + + + − =
Bài 2: Giải hệ phương trình
3 3 2
4 4
1
4 4
x y xy
x y x y
 + − =

+ = +

Bài 3: Giả sử x, y là số không âm thỏa mãn 2 2
1
x y
+ =
a) Chứng minh rằng 1 2
x y
≤ + ≤
b)Tìm giá trị lớn nhất và nhỏ nhất của biểu thức 1 2 1 2
P x y
= + + +
Bài 4: Cho hình vuông ABCD và điểm P nằm trong tam giác ABC
a) Giả sử  135o
BPC = . Chứng minh rằng: 2 2 2
2PB PC PA
+ =
b) Các đường thẳng AP và CP cắt các cạnh BC và BA tại M, N. Gọi Q là điểm đối xứng với B
qua trung điểm của đoạn MN. Chứng minh rằng khi P thay đổi trong tam giác ABC, đường thẳng
PQ luôn đi qua D.
Bài 5:
a) Cho đa giác đều (H) có 14 đỉnh. Chứng minh rằng trong 6 đỉnh bất kỳ của (H) luôn có 4 đỉnh là
các đỉnh của 1 hình thang
b) Có bao nhiêu phân số tối giản 1
m
n
> (m, n là các số nguyên dương ) thỏa mãn . 13860.
m n =
Website:
Tài liệu sưu tầm và tổng hợp bản word đầy đủ liên hệ TÀI LIỆU TOÁN HỌC
38
ĐẠI HỌC QUỐC GIA HÀ NỘI
TRƯỜNG ĐẠI HỌC KHOA HỌC TỰ NHIÊN
ĐỀ THI TUYỂN SINH LỚP 10
HỆ THPT CHUYÊN NĂM 2004
Đề số 36- MÔN : TOÁN (Vòng 1)
Thời gian làm bài: 150 phút (không kể thời gian phát đề)
Câu 1:
1) Giải phương trình
1
x
1
1
x
1
x 2
−
+
=
−
+
+
2) Tìm nghiệm nguyên của hệ



=
−
+
+
=
−
+
−
−
8
y
x
y
x
7
x
2
y
2
xy
x
y
2
3
3
2
2
Câu 2: Cho các số thực dương a và b thỏa mãn
100 100 101 101 102 102
+ = + = +
a b a b a b
Hãy tính giá trị của biểu thức
2004
2004
b
a
P +
=
Câu 3: Cho tam giác ABC có AB = 3 cm, BC = 4 cm, CA = 5 cm. Đường cao, đường phân giác,
đường trung tuyến của tam giác kẻ từ đỉnh B chia tam giác thành bốn phần. Hãy tính diện tích mỗi
phần.
Câu 4: Cho tứ giác ABCD nội tiếp trong đường tròn có hai đường chéo AC và BD vuông góc với
nhau tại H (H không trùng với tâm của đường tròn). Gọi M và N lần lượt là chân các đường vuông
góc hạ từ H xuống các đường thẳng AB và BC; P và Q lần lượt là giao điểm của đường thẳng MH
và NH với các đường thẳng CD và DA. Chứng minh rằng đường thẳng PQ song song với đường
thẳng AC và bốn điểm M, N, P, Q nằm trên cùng một đường tròn.
Câu 5: Tìm giá trị nhỏ nhất của biểu thức
( ) ( )2
2
2
16
16
2
10
2
10
y
x
1
y
x
4
1
x
y
y
x
2
1
Q +
−
+
+








+
=
Website:
Tài liệu sưu tầm và tổng hợp bản word đầy đủ liên hệ TÀI LIỆU TOÁN HỌC
39
ĐẠI HỌC QUỐC GIA HÀ NỘI
TRƯỜNG ĐẠI HỌC KHOA HỌC TỰ NHIÊN
ĐỀ THI TUYỂN SINH LỚP 10
HỆ THPT CHUYÊN NĂM 2004
Đề số 37- MÔN : TOÁN (Vòng 2)
Thời gian làm bài: 150 phút (không kể thời gian phát đề)
Bài 1. Giải phương trình 3 1 2
x x
+ + − =
Bài 2. Giải hệ phương trình ( ) ( )
2 2 2 2
( ) 15( ) 3
x y x y x y x y
+ + = − − =
Bài 3. Tìm giá trị nhỏ nhất của biểu thức
( ) ( )
3 3 2 2
( 1)( 1)
x y x y
P
x y
+ − +
=
− −
trong đó, x, y là những số thực lớn hơn 1 .
Bài 4. Cho hình vuông ABCD và điểm M nằm trong hình vuông.
1. Tìm tất cả các vị trí của điểm M sao cho    
MAB MBC MCD MDA
= = =
2. Xét điểm M nằm trên đường chéo AC. Gọi N là chân đường vuông góc hạ từ điểm M xuống
A B và O là trung điểm của đoạn AM. Chứng minh rầng tỷ số
OB
CN
có giá trị không đổi khi M di
chuyển trên đường chéo A C.
3. Với giả thiết M nằm trên đường chéo AC, xét các đường tròn ( )
1
S và ( )
2
S có đường kính
tương ứng là AM và CN. Hai tiếp tuyến chung của ( )
1
S và ( )
2
S tiếp xúc với ( )
2
S tại P và Q .
Chứng minh rằng đường thẳng PQ tiếp xúc với ( )
1
S
Bài 5. Với số thực a , ta định nghĩa phần nguyên của số a là số nguyên lớn nhất không vượt quá a
và ký hiệu là [a]. Dãy các số 0 1 2
, , , , ,
n
x x x x
… …. được xác định bởi công thức
1
2 2
n
n n
x
   
+
= −
   
   
Hỏi trong 200 số { }
0 1 199
, , ,
x x x
… có bao nhiêu số khác 0? (Cho biết 1,41 2 1,42
< < ).
Website:
Tài liệu sưu tầm và tổng hợp bản word đầy đủ liên hệ TÀI LIỆU TOÁN HỌC
40
ĐẠI HỌC QUỐC GIA HÀ NỘI
TRƯỜNG ĐẠI HỌC KHOA HỌC TỰ NHIÊN
ĐỀ THI TUYỂN SINH LỚP 10
HỆ THPT CHUYÊN NĂM 2003
Đề số 38- MÔN : TOÁN (Vòng 1)
Thời gian làm bài: 150 phút (không kể thời gian phát đề)
Câu 1 (2 điểm): Giải phương trình:
2
( 5 2)(1 7 10) 3
x x x x
+ − + + + + =
Câu 2 (2 điểm): Giải hệ phương trình:





=
+
=
+
7
xy
6
y
5
y
x
3
x
2
2
3
2
3
Câu 3 (2 điểm): Tìm các số nguyên x, y thỏa mãn đẳng thức:
xy
y
2
x
1
y
x
x
y
2 2
2
2
+
+
=
+
+
+
Câu 4 (3 điểm) Cho nửa đường tròn (O) đường kính AB = 2R (R là một độ dài cho trước). M, N là
hai điểm trên nửa đường tròn (O) sao cho M thuộc cung AN và tổng các khoảng cách từ A, B đến
đường thẳng MN bằng 3 .
1. Tính độ dài đoạn MN theo R.
2. Gọi giao điểm của hai dây AN và BM là I, giao điểm của các đường thẳng AM và BN là K.
Chứng minh rằng bốn điểm M, N, I, K cùng nằm trên một đường tròn. Tính bán kính của đường
tròn đó theo R.
3. Tìm giá trị lớn nhất của diện tích tam giác KAB theo R khi M, N thay đổi nhưng vẫn thỏa
mãn giả thiết của bài toán.
Câu 5 (1 điểm): x, y, z là các số thực thỏa mãn điều kiện:
6
zx
yz
xy
z
y
x =
+
+
+
+
+
Chứng minh rằng: 3
z
y
x 2
2
2
≥
+
+
Website:
Tài liệu sưu tầm và tổng hợp bản word đầy đủ liên hệ TÀI LIỆU TOÁN HỌC
41
ĐẠI HỌC QUỐC GIA HÀ NỘI
TRƯỜNG ĐẠI HỌC KHOA HỌC TỰ NHIÊN
ĐỀ THI TUYỂN SINH LỚP 10
HỆ THPT CHUYÊN NĂM 2003
Đề số 39- MÔN : TOÁN (Vòng 2)
Thời gian làm bài: 150 phút (không kể thời gian phát đề)
Bài 1. Cho phương trình 4 2
2 4 0
x mx
+ + =
Tìm giá trị của tham số m để phương trình có 4 nghiệm phân biệt 1 2 3 4
, , ,
x x x x thoả mãn
4 4 4 4
1 2 3 4 32
x x x x
+ + + =
Bài 2. Giải hệ phương trình
2 2
2 2
2 5 2 0
4 0
x xy y x y
x y x y
 + − − + + =

+ + + − =

Bài 3. Tìm các số nguyên x, y thoả mãn đẳng thức 2 2 2 2
x xy y x y
+ + =
Bài 4. Cho đường tròn tâm O nội tiếp tam giác ABC tiếp xúc với các cạnh BC, CA, AB tương ứng
tại các điểm D, E, F. Đường tròn tâm O′
bàng tiếp trong góc 
BAC của tam giác ABC tiêp xúc với
cạnh BC và phần kéo dài của các cạnh AB, AC tương ứng tại các điểm $P, M, N$.
1. Chứng minh rằng: BP CD
= .
2. Trên đường thẳng MN ta lấy các điểm I và K sao cho / / , / /
CK AB BI AC . Chứng minh rằng
các tứ giác BICE và BKCF là các hình binh hành.
3. Gọi ( )
S là đường tròn đi qua ba điểm I, K, P. Chứng minh rằng ( )
S tiếp xúc với các đường
thẳng BC, BI, CK.
Bài 5. Số thực x thay đổi và thoả mãn điều kiện 2 2
(3 ) 5
x x
+ −  . Tìm giá trị nhỏ nhất của biểu
thức 4 4 2 2
(3 ) 6 (3 )
p x x x x
= + − + −
TUYỂN TẬP ĐỀ THI TOÁN VÀO LỚP 10 CHUYÊN KHOA HỌC TỰ NHIÊN HÀ NỘI NĂM 2022 CÓ LỜI GIẢI CHI TIẾT.pdf
TUYỂN TẬP ĐỀ THI TOÁN VÀO LỚP 10 CHUYÊN KHOA HỌC TỰ NHIÊN HÀ NỘI NĂM 2022 CÓ LỜI GIẢI CHI TIẾT.pdf
TUYỂN TẬP ĐỀ THI TOÁN VÀO LỚP 10 CHUYÊN KHOA HỌC TỰ NHIÊN HÀ NỘI NĂM 2022 CÓ LỜI GIẢI CHI TIẾT.pdf
TUYỂN TẬP ĐỀ THI TOÁN VÀO LỚP 10 CHUYÊN KHOA HỌC TỰ NHIÊN HÀ NỘI NĂM 2022 CÓ LỜI GIẢI CHI TIẾT.pdf
TUYỂN TẬP ĐỀ THI TOÁN VÀO LỚP 10 CHUYÊN KHOA HỌC TỰ NHIÊN HÀ NỘI NĂM 2022 CÓ LỜI GIẢI CHI TIẾT.pdf
TUYỂN TẬP ĐỀ THI TOÁN VÀO LỚP 10 CHUYÊN KHOA HỌC TỰ NHIÊN HÀ NỘI NĂM 2022 CÓ LỜI GIẢI CHI TIẾT.pdf
TUYỂN TẬP ĐỀ THI TOÁN VÀO LỚP 10 CHUYÊN KHOA HỌC TỰ NHIÊN HÀ NỘI NĂM 2022 CÓ LỜI GIẢI CHI TIẾT.pdf
TUYỂN TẬP ĐỀ THI TOÁN VÀO LỚP 10 CHUYÊN KHOA HỌC TỰ NHIÊN HÀ NỘI NĂM 2022 CÓ LỜI GIẢI CHI TIẾT.pdf
TUYỂN TẬP ĐỀ THI TOÁN VÀO LỚP 10 CHUYÊN KHOA HỌC TỰ NHIÊN HÀ NỘI NĂM 2022 CÓ LỜI GIẢI CHI TIẾT.pdf
TUYỂN TẬP ĐỀ THI TOÁN VÀO LỚP 10 CHUYÊN KHOA HỌC TỰ NHIÊN HÀ NỘI NĂM 2022 CÓ LỜI GIẢI CHI TIẾT.pdf
TUYỂN TẬP ĐỀ THI TOÁN VÀO LỚP 10 CHUYÊN KHOA HỌC TỰ NHIÊN HÀ NỘI NĂM 2022 CÓ LỜI GIẢI CHI TIẾT.pdf
TUYỂN TẬP ĐỀ THI TOÁN VÀO LỚP 10 CHUYÊN KHOA HỌC TỰ NHIÊN HÀ NỘI NĂM 2022 CÓ LỜI GIẢI CHI TIẾT.pdf
TUYỂN TẬP ĐỀ THI TOÁN VÀO LỚP 10 CHUYÊN KHOA HỌC TỰ NHIÊN HÀ NỘI NĂM 2022 CÓ LỜI GIẢI CHI TIẾT.pdf
TUYỂN TẬP ĐỀ THI TOÁN VÀO LỚP 10 CHUYÊN KHOA HỌC TỰ NHIÊN HÀ NỘI NĂM 2022 CÓ LỜI GIẢI CHI TIẾT.pdf
TUYỂN TẬP ĐỀ THI TOÁN VÀO LỚP 10 CHUYÊN KHOA HỌC TỰ NHIÊN HÀ NỘI NĂM 2022 CÓ LỜI GIẢI CHI TIẾT.pdf
TUYỂN TẬP ĐỀ THI TOÁN VÀO LỚP 10 CHUYÊN KHOA HỌC TỰ NHIÊN HÀ NỘI NĂM 2022 CÓ LỜI GIẢI CHI TIẾT.pdf
TUYỂN TẬP ĐỀ THI TOÁN VÀO LỚP 10 CHUYÊN KHOA HỌC TỰ NHIÊN HÀ NỘI NĂM 2022 CÓ LỜI GIẢI CHI TIẾT.pdf
TUYỂN TẬP ĐỀ THI TOÁN VÀO LỚP 10 CHUYÊN KHOA HỌC TỰ NHIÊN HÀ NỘI NĂM 2022 CÓ LỜI GIẢI CHI TIẾT.pdf
TUYỂN TẬP ĐỀ THI TOÁN VÀO LỚP 10 CHUYÊN KHOA HỌC TỰ NHIÊN HÀ NỘI NĂM 2022 CÓ LỜI GIẢI CHI TIẾT.pdf
TUYỂN TẬP ĐỀ THI TOÁN VÀO LỚP 10 CHUYÊN KHOA HỌC TỰ NHIÊN HÀ NỘI NĂM 2022 CÓ LỜI GIẢI CHI TIẾT.pdf
TUYỂN TẬP ĐỀ THI TOÁN VÀO LỚP 10 CHUYÊN KHOA HỌC TỰ NHIÊN HÀ NỘI NĂM 2022 CÓ LỜI GIẢI CHI TIẾT.pdf
TUYỂN TẬP ĐỀ THI TOÁN VÀO LỚP 10 CHUYÊN KHOA HỌC TỰ NHIÊN HÀ NỘI NĂM 2022 CÓ LỜI GIẢI CHI TIẾT.pdf
TUYỂN TẬP ĐỀ THI TOÁN VÀO LỚP 10 CHUYÊN KHOA HỌC TỰ NHIÊN HÀ NỘI NĂM 2022 CÓ LỜI GIẢI CHI TIẾT.pdf
TUYỂN TẬP ĐỀ THI TOÁN VÀO LỚP 10 CHUYÊN KHOA HỌC TỰ NHIÊN HÀ NỘI NĂM 2022 CÓ LỜI GIẢI CHI TIẾT.pdf
TUYỂN TẬP ĐỀ THI TOÁN VÀO LỚP 10 CHUYÊN KHOA HỌC TỰ NHIÊN HÀ NỘI NĂM 2022 CÓ LỜI GIẢI CHI TIẾT.pdf
TUYỂN TẬP ĐỀ THI TOÁN VÀO LỚP 10 CHUYÊN KHOA HỌC TỰ NHIÊN HÀ NỘI NĂM 2022 CÓ LỜI GIẢI CHI TIẾT.pdf
TUYỂN TẬP ĐỀ THI TOÁN VÀO LỚP 10 CHUYÊN KHOA HỌC TỰ NHIÊN HÀ NỘI NĂM 2022 CÓ LỜI GIẢI CHI TIẾT.pdf
TUYỂN TẬP ĐỀ THI TOÁN VÀO LỚP 10 CHUYÊN KHOA HỌC TỰ NHIÊN HÀ NỘI NĂM 2022 CÓ LỜI GIẢI CHI TIẾT.pdf
TUYỂN TẬP ĐỀ THI TOÁN VÀO LỚP 10 CHUYÊN KHOA HỌC TỰ NHIÊN HÀ NỘI NĂM 2022 CÓ LỜI GIẢI CHI TIẾT.pdf
TUYỂN TẬP ĐỀ THI TOÁN VÀO LỚP 10 CHUYÊN KHOA HỌC TỰ NHIÊN HÀ NỘI NĂM 2022 CÓ LỜI GIẢI CHI TIẾT.pdf
TUYỂN TẬP ĐỀ THI TOÁN VÀO LỚP 10 CHUYÊN KHOA HỌC TỰ NHIÊN HÀ NỘI NĂM 2022 CÓ LỜI GIẢI CHI TIẾT.pdf
TUYỂN TẬP ĐỀ THI TOÁN VÀO LỚP 10 CHUYÊN KHOA HỌC TỰ NHIÊN HÀ NỘI NĂM 2022 CÓ LỜI GIẢI CHI TIẾT.pdf
TUYỂN TẬP ĐỀ THI TOÁN VÀO LỚP 10 CHUYÊN KHOA HỌC TỰ NHIÊN HÀ NỘI NĂM 2022 CÓ LỜI GIẢI CHI TIẾT.pdf
TUYỂN TẬP ĐỀ THI TOÁN VÀO LỚP 10 CHUYÊN KHOA HỌC TỰ NHIÊN HÀ NỘI NĂM 2022 CÓ LỜI GIẢI CHI TIẾT.pdf
TUYỂN TẬP ĐỀ THI TOÁN VÀO LỚP 10 CHUYÊN KHOA HỌC TỰ NHIÊN HÀ NỘI NĂM 2022 CÓ LỜI GIẢI CHI TIẾT.pdf
TUYỂN TẬP ĐỀ THI TOÁN VÀO LỚP 10 CHUYÊN KHOA HỌC TỰ NHIÊN HÀ NỘI NĂM 2022 CÓ LỜI GIẢI CHI TIẾT.pdf
TUYỂN TẬP ĐỀ THI TOÁN VÀO LỚP 10 CHUYÊN KHOA HỌC TỰ NHIÊN HÀ NỘI NĂM 2022 CÓ LỜI GIẢI CHI TIẾT.pdf
TUYỂN TẬP ĐỀ THI TOÁN VÀO LỚP 10 CHUYÊN KHOA HỌC TỰ NHIÊN HÀ NỘI NĂM 2022 CÓ LỜI GIẢI CHI TIẾT.pdf
TUYỂN TẬP ĐỀ THI TOÁN VÀO LỚP 10 CHUYÊN KHOA HỌC TỰ NHIÊN HÀ NỘI NĂM 2022 CÓ LỜI GIẢI CHI TIẾT.pdf
TUYỂN TẬP ĐỀ THI TOÁN VÀO LỚP 10 CHUYÊN KHOA HỌC TỰ NHIÊN HÀ NỘI NĂM 2022 CÓ LỜI GIẢI CHI TIẾT.pdf
TUYỂN TẬP ĐỀ THI TOÁN VÀO LỚP 10 CHUYÊN KHOA HỌC TỰ NHIÊN HÀ NỘI NĂM 2022 CÓ LỜI GIẢI CHI TIẾT.pdf
TUYỂN TẬP ĐỀ THI TOÁN VÀO LỚP 10 CHUYÊN KHOA HỌC TỰ NHIÊN HÀ NỘI NĂM 2022 CÓ LỜI GIẢI CHI TIẾT.pdf
TUYỂN TẬP ĐỀ THI TOÁN VÀO LỚP 10 CHUYÊN KHOA HỌC TỰ NHIÊN HÀ NỘI NĂM 2022 CÓ LỜI GIẢI CHI TIẾT.pdf
TUYỂN TẬP ĐỀ THI TOÁN VÀO LỚP 10 CHUYÊN KHOA HỌC TỰ NHIÊN HÀ NỘI NĂM 2022 CÓ LỜI GIẢI CHI TIẾT.pdf
TUYỂN TẬP ĐỀ THI TOÁN VÀO LỚP 10 CHUYÊN KHOA HỌC TỰ NHIÊN HÀ NỘI NĂM 2022 CÓ LỜI GIẢI CHI TIẾT.pdf
TUYỂN TẬP ĐỀ THI TOÁN VÀO LỚP 10 CHUYÊN KHOA HỌC TỰ NHIÊN HÀ NỘI NĂM 2022 CÓ LỜI GIẢI CHI TIẾT.pdf
TUYỂN TẬP ĐỀ THI TOÁN VÀO LỚP 10 CHUYÊN KHOA HỌC TỰ NHIÊN HÀ NỘI NĂM 2022 CÓ LỜI GIẢI CHI TIẾT.pdf
TUYỂN TẬP ĐỀ THI TOÁN VÀO LỚP 10 CHUYÊN KHOA HỌC TỰ NHIÊN HÀ NỘI NĂM 2022 CÓ LỜI GIẢI CHI TIẾT.pdf
TUYỂN TẬP ĐỀ THI TOÁN VÀO LỚP 10 CHUYÊN KHOA HỌC TỰ NHIÊN HÀ NỘI NĂM 2022 CÓ LỜI GIẢI CHI TIẾT.pdf
TUYỂN TẬP ĐỀ THI TOÁN VÀO LỚP 10 CHUYÊN KHOA HỌC TỰ NHIÊN HÀ NỘI NĂM 2022 CÓ LỜI GIẢI CHI TIẾT.pdf
TUYỂN TẬP ĐỀ THI TOÁN VÀO LỚP 10 CHUYÊN KHOA HỌC TỰ NHIÊN HÀ NỘI NĂM 2022 CÓ LỜI GIẢI CHI TIẾT.pdf
TUYỂN TẬP ĐỀ THI TOÁN VÀO LỚP 10 CHUYÊN KHOA HỌC TỰ NHIÊN HÀ NỘI NĂM 2022 CÓ LỜI GIẢI CHI TIẾT.pdf
TUYỂN TẬP ĐỀ THI TOÁN VÀO LỚP 10 CHUYÊN KHOA HỌC TỰ NHIÊN HÀ NỘI NĂM 2022 CÓ LỜI GIẢI CHI TIẾT.pdf
TUYỂN TẬP ĐỀ THI TOÁN VÀO LỚP 10 CHUYÊN KHOA HỌC TỰ NHIÊN HÀ NỘI NĂM 2022 CÓ LỜI GIẢI CHI TIẾT.pdf
TUYỂN TẬP ĐỀ THI TOÁN VÀO LỚP 10 CHUYÊN KHOA HỌC TỰ NHIÊN HÀ NỘI NĂM 2022 CÓ LỜI GIẢI CHI TIẾT.pdf
TUYỂN TẬP ĐỀ THI TOÁN VÀO LỚP 10 CHUYÊN KHOA HỌC TỰ NHIÊN HÀ NỘI NĂM 2022 CÓ LỜI GIẢI CHI TIẾT.pdf
TUYỂN TẬP ĐỀ THI TOÁN VÀO LỚP 10 CHUYÊN KHOA HỌC TỰ NHIÊN HÀ NỘI NĂM 2022 CÓ LỜI GIẢI CHI TIẾT.pdf
TUYỂN TẬP ĐỀ THI TOÁN VÀO LỚP 10 CHUYÊN KHOA HỌC TỰ NHIÊN HÀ NỘI NĂM 2022 CÓ LỜI GIẢI CHI TIẾT.pdf
TUYỂN TẬP ĐỀ THI TOÁN VÀO LỚP 10 CHUYÊN KHOA HỌC TỰ NHIÊN HÀ NỘI NĂM 2022 CÓ LỜI GIẢI CHI TIẾT.pdf
TUYỂN TẬP ĐỀ THI TOÁN VÀO LỚP 10 CHUYÊN KHOA HỌC TỰ NHIÊN HÀ NỘI NĂM 2022 CÓ LỜI GIẢI CHI TIẾT.pdf
TUYỂN TẬP ĐỀ THI TOÁN VÀO LỚP 10 CHUYÊN KHOA HỌC TỰ NHIÊN HÀ NỘI NĂM 2022 CÓ LỜI GIẢI CHI TIẾT.pdf
TUYỂN TẬP ĐỀ THI TOÁN VÀO LỚP 10 CHUYÊN KHOA HỌC TỰ NHIÊN HÀ NỘI NĂM 2022 CÓ LỜI GIẢI CHI TIẾT.pdf
TUYỂN TẬP ĐỀ THI TOÁN VÀO LỚP 10 CHUYÊN KHOA HỌC TỰ NHIÊN HÀ NỘI NĂM 2022 CÓ LỜI GIẢI CHI TIẾT.pdf
TUYỂN TẬP ĐỀ THI TOÁN VÀO LỚP 10 CHUYÊN KHOA HỌC TỰ NHIÊN HÀ NỘI NĂM 2022 CÓ LỜI GIẢI CHI TIẾT.pdf
TUYỂN TẬP ĐỀ THI TOÁN VÀO LỚP 10 CHUYÊN KHOA HỌC TỰ NHIÊN HÀ NỘI NĂM 2022 CÓ LỜI GIẢI CHI TIẾT.pdf
TUYỂN TẬP ĐỀ THI TOÁN VÀO LỚP 10 CHUYÊN KHOA HỌC TỰ NHIÊN HÀ NỘI NĂM 2022 CÓ LỜI GIẢI CHI TIẾT.pdf
TUYỂN TẬP ĐỀ THI TOÁN VÀO LỚP 10 CHUYÊN KHOA HỌC TỰ NHIÊN HÀ NỘI NĂM 2022 CÓ LỜI GIẢI CHI TIẾT.pdf
TUYỂN TẬP ĐỀ THI TOÁN VÀO LỚP 10 CHUYÊN KHOA HỌC TỰ NHIÊN HÀ NỘI NĂM 2022 CÓ LỜI GIẢI CHI TIẾT.pdf
TUYỂN TẬP ĐỀ THI TOÁN VÀO LỚP 10 CHUYÊN KHOA HỌC TỰ NHIÊN HÀ NỘI NĂM 2022 CÓ LỜI GIẢI CHI TIẾT.pdf
TUYỂN TẬP ĐỀ THI TOÁN VÀO LỚP 10 CHUYÊN KHOA HỌC TỰ NHIÊN HÀ NỘI NĂM 2022 CÓ LỜI GIẢI CHI TIẾT.pdf
TUYỂN TẬP ĐỀ THI TOÁN VÀO LỚP 10 CHUYÊN KHOA HỌC TỰ NHIÊN HÀ NỘI NĂM 2022 CÓ LỜI GIẢI CHI TIẾT.pdf
TUYỂN TẬP ĐỀ THI TOÁN VÀO LỚP 10 CHUYÊN KHOA HỌC TỰ NHIÊN HÀ NỘI NĂM 2022 CÓ LỜI GIẢI CHI TIẾT.pdf
TUYỂN TẬP ĐỀ THI TOÁN VÀO LỚP 10 CHUYÊN KHOA HỌC TỰ NHIÊN HÀ NỘI NĂM 2022 CÓ LỜI GIẢI CHI TIẾT.pdf
TUYỂN TẬP ĐỀ THI TOÁN VÀO LỚP 10 CHUYÊN KHOA HỌC TỰ NHIÊN HÀ NỘI NĂM 2022 CÓ LỜI GIẢI CHI TIẾT.pdf
TUYỂN TẬP ĐỀ THI TOÁN VÀO LỚP 10 CHUYÊN KHOA HỌC TỰ NHIÊN HÀ NỘI NĂM 2022 CÓ LỜI GIẢI CHI TIẾT.pdf
TUYỂN TẬP ĐỀ THI TOÁN VÀO LỚP 10 CHUYÊN KHOA HỌC TỰ NHIÊN HÀ NỘI NĂM 2022 CÓ LỜI GIẢI CHI TIẾT.pdf
TUYỂN TẬP ĐỀ THI TOÁN VÀO LỚP 10 CHUYÊN KHOA HỌC TỰ NHIÊN HÀ NỘI NĂM 2022 CÓ LỜI GIẢI CHI TIẾT.pdf
TUYỂN TẬP ĐỀ THI TOÁN VÀO LỚP 10 CHUYÊN KHOA HỌC TỰ NHIÊN HÀ NỘI NĂM 2022 CÓ LỜI GIẢI CHI TIẾT.pdf
TUYỂN TẬP ĐỀ THI TOÁN VÀO LỚP 10 CHUYÊN KHOA HỌC TỰ NHIÊN HÀ NỘI NĂM 2022 CÓ LỜI GIẢI CHI TIẾT.pdf
TUYỂN TẬP ĐỀ THI TOÁN VÀO LỚP 10 CHUYÊN KHOA HỌC TỰ NHIÊN HÀ NỘI NĂM 2022 CÓ LỜI GIẢI CHI TIẾT.pdf
TUYỂN TẬP ĐỀ THI TOÁN VÀO LỚP 10 CHUYÊN KHOA HỌC TỰ NHIÊN HÀ NỘI NĂM 2022 CÓ LỜI GIẢI CHI TIẾT.pdf
TUYỂN TẬP ĐỀ THI TOÁN VÀO LỚP 10 CHUYÊN KHOA HỌC TỰ NHIÊN HÀ NỘI NĂM 2022 CÓ LỜI GIẢI CHI TIẾT.pdf
TUYỂN TẬP ĐỀ THI TOÁN VÀO LỚP 10 CHUYÊN KHOA HỌC TỰ NHIÊN HÀ NỘI NĂM 2022 CÓ LỜI GIẢI CHI TIẾT.pdf
TUYỂN TẬP ĐỀ THI TOÁN VÀO LỚP 10 CHUYÊN KHOA HỌC TỰ NHIÊN HÀ NỘI NĂM 2022 CÓ LỜI GIẢI CHI TIẾT.pdf
TUYỂN TẬP ĐỀ THI TOÁN VÀO LỚP 10 CHUYÊN KHOA HỌC TỰ NHIÊN HÀ NỘI NĂM 2022 CÓ LỜI GIẢI CHI TIẾT.pdf
TUYỂN TẬP ĐỀ THI TOÁN VÀO LỚP 10 CHUYÊN KHOA HỌC TỰ NHIÊN HÀ NỘI NĂM 2022 CÓ LỜI GIẢI CHI TIẾT.pdf
TUYỂN TẬP ĐỀ THI TOÁN VÀO LỚP 10 CHUYÊN KHOA HỌC TỰ NHIÊN HÀ NỘI NĂM 2022 CÓ LỜI GIẢI CHI TIẾT.pdf
TUYỂN TẬP ĐỀ THI TOÁN VÀO LỚP 10 CHUYÊN KHOA HỌC TỰ NHIÊN HÀ NỘI NĂM 2022 CÓ LỜI GIẢI CHI TIẾT.pdf
TUYỂN TẬP ĐỀ THI TOÁN VÀO LỚP 10 CHUYÊN KHOA HỌC TỰ NHIÊN HÀ NỘI NĂM 2022 CÓ LỜI GIẢI CHI TIẾT.pdf
TUYỂN TẬP ĐỀ THI TOÁN VÀO LỚP 10 CHUYÊN KHOA HỌC TỰ NHIÊN HÀ NỘI NĂM 2022 CÓ LỜI GIẢI CHI TIẾT.pdf
TUYỂN TẬP ĐỀ THI TOÁN VÀO LỚP 10 CHUYÊN KHOA HỌC TỰ NHIÊN HÀ NỘI NĂM 2022 CÓ LỜI GIẢI CHI TIẾT.pdf
TUYỂN TẬP ĐỀ THI TOÁN VÀO LỚP 10 CHUYÊN KHOA HỌC TỰ NHIÊN HÀ NỘI NĂM 2022 CÓ LỜI GIẢI CHI TIẾT.pdf
TUYỂN TẬP ĐỀ THI TOÁN VÀO LỚP 10 CHUYÊN KHOA HỌC TỰ NHIÊN HÀ NỘI NĂM 2022 CÓ LỜI GIẢI CHI TIẾT.pdf
TUYỂN TẬP ĐỀ THI TOÁN VÀO LỚP 10 CHUYÊN KHOA HỌC TỰ NHIÊN HÀ NỘI NĂM 2022 CÓ LỜI GIẢI CHI TIẾT.pdf
TUYỂN TẬP ĐỀ THI TOÁN VÀO LỚP 10 CHUYÊN KHOA HỌC TỰ NHIÊN HÀ NỘI NĂM 2022 CÓ LỜI GIẢI CHI TIẾT.pdf
TUYỂN TẬP ĐỀ THI TOÁN VÀO LỚP 10 CHUYÊN KHOA HỌC TỰ NHIÊN HÀ NỘI NĂM 2022 CÓ LỜI GIẢI CHI TIẾT.pdf
TUYỂN TẬP ĐỀ THI TOÁN VÀO LỚP 10 CHUYÊN KHOA HỌC TỰ NHIÊN HÀ NỘI NĂM 2022 CÓ LỜI GIẢI CHI TIẾT.pdf
TUYỂN TẬP ĐỀ THI TOÁN VÀO LỚP 10 CHUYÊN KHOA HỌC TỰ NHIÊN HÀ NỘI NĂM 2022 CÓ LỜI GIẢI CHI TIẾT.pdf
TUYỂN TẬP ĐỀ THI TOÁN VÀO LỚP 10 CHUYÊN KHOA HỌC TỰ NHIÊN HÀ NỘI NĂM 2022 CÓ LỜI GIẢI CHI TIẾT.pdf
TUYỂN TẬP ĐỀ THI TOÁN VÀO LỚP 10 CHUYÊN KHOA HỌC TỰ NHIÊN HÀ NỘI NĂM 2022 CÓ LỜI GIẢI CHI TIẾT.pdf
TUYỂN TẬP ĐỀ THI TOÁN VÀO LỚP 10 CHUYÊN KHOA HỌC TỰ NHIÊN HÀ NỘI NĂM 2022 CÓ LỜI GIẢI CHI TIẾT.pdf
TUYỂN TẬP ĐỀ THI TOÁN VÀO LỚP 10 CHUYÊN KHOA HỌC TỰ NHIÊN HÀ NỘI NĂM 2022 CÓ LỜI GIẢI CHI TIẾT.pdf
TUYỂN TẬP ĐỀ THI TOÁN VÀO LỚP 10 CHUYÊN KHOA HỌC TỰ NHIÊN HÀ NỘI NĂM 2022 CÓ LỜI GIẢI CHI TIẾT.pdf
TUYỂN TẬP ĐỀ THI TOÁN VÀO LỚP 10 CHUYÊN KHOA HỌC TỰ NHIÊN HÀ NỘI NĂM 2022 CÓ LỜI GIẢI CHI TIẾT.pdf
TUYỂN TẬP ĐỀ THI TOÁN VÀO LỚP 10 CHUYÊN KHOA HỌC TỰ NHIÊN HÀ NỘI NĂM 2022 CÓ LỜI GIẢI CHI TIẾT.pdf
TUYỂN TẬP ĐỀ THI TOÁN VÀO LỚP 10 CHUYÊN KHOA HỌC TỰ NHIÊN HÀ NỘI NĂM 2022 CÓ LỜI GIẢI CHI TIẾT.pdf
TUYỂN TẬP ĐỀ THI TOÁN VÀO LỚP 10 CHUYÊN KHOA HỌC TỰ NHIÊN HÀ NỘI NĂM 2022 CÓ LỜI GIẢI CHI TIẾT.pdf
TUYỂN TẬP ĐỀ THI TOÁN VÀO LỚP 10 CHUYÊN KHOA HỌC TỰ NHIÊN HÀ NỘI NĂM 2022 CÓ LỜI GIẢI CHI TIẾT.pdf
TUYỂN TẬP ĐỀ THI TOÁN VÀO LỚP 10 CHUYÊN KHOA HỌC TỰ NHIÊN HÀ NỘI NĂM 2022 CÓ LỜI GIẢI CHI TIẾT.pdf
TUYỂN TẬP ĐỀ THI TOÁN VÀO LỚP 10 CHUYÊN KHOA HỌC TỰ NHIÊN HÀ NỘI NĂM 2022 CÓ LỜI GIẢI CHI TIẾT.pdf
TUYỂN TẬP ĐỀ THI TOÁN VÀO LỚP 10 CHUYÊN KHOA HỌC TỰ NHIÊN HÀ NỘI NĂM 2022 CÓ LỜI GIẢI CHI TIẾT.pdf
TUYỂN TẬP ĐỀ THI TOÁN VÀO LỚP 10 CHUYÊN KHOA HỌC TỰ NHIÊN HÀ NỘI NĂM 2022 CÓ LỜI GIẢI CHI TIẾT.pdf
TUYỂN TẬP ĐỀ THI TOÁN VÀO LỚP 10 CHUYÊN KHOA HỌC TỰ NHIÊN HÀ NỘI NĂM 2022 CÓ LỜI GIẢI CHI TIẾT.pdf
TUYỂN TẬP ĐỀ THI TOÁN VÀO LỚP 10 CHUYÊN KHOA HỌC TỰ NHIÊN HÀ NỘI NĂM 2022 CÓ LỜI GIẢI CHI TIẾT.pdf
TUYỂN TẬP ĐỀ THI TOÁN VÀO LỚP 10 CHUYÊN KHOA HỌC TỰ NHIÊN HÀ NỘI NĂM 2022 CÓ LỜI GIẢI CHI TIẾT.pdf
TUYỂN TẬP ĐỀ THI TOÁN VÀO LỚP 10 CHUYÊN KHOA HỌC TỰ NHIÊN HÀ NỘI NĂM 2022 CÓ LỜI GIẢI CHI TIẾT.pdf
TUYỂN TẬP ĐỀ THI TOÁN VÀO LỚP 10 CHUYÊN KHOA HỌC TỰ NHIÊN HÀ NỘI NĂM 2022 CÓ LỜI GIẢI CHI TIẾT.pdf
TUYỂN TẬP ĐỀ THI TOÁN VÀO LỚP 10 CHUYÊN KHOA HỌC TỰ NHIÊN HÀ NỘI NĂM 2022 CÓ LỜI GIẢI CHI TIẾT.pdf
TUYỂN TẬP ĐỀ THI TOÁN VÀO LỚP 10 CHUYÊN KHOA HỌC TỰ NHIÊN HÀ NỘI NĂM 2022 CÓ LỜI GIẢI CHI TIẾT.pdf
TUYỂN TẬP ĐỀ THI TOÁN VÀO LỚP 10 CHUYÊN KHOA HỌC TỰ NHIÊN HÀ NỘI NĂM 2022 CÓ LỜI GIẢI CHI TIẾT.pdf
TUYỂN TẬP ĐỀ THI TOÁN VÀO LỚP 10 CHUYÊN KHOA HỌC TỰ NHIÊN HÀ NỘI NĂM 2022 CÓ LỜI GIẢI CHI TIẾT.pdf
TUYỂN TẬP ĐỀ THI TOÁN VÀO LỚP 10 CHUYÊN KHOA HỌC TỰ NHIÊN HÀ NỘI NĂM 2022 CÓ LỜI GIẢI CHI TIẾT.pdf
TUYỂN TẬP ĐỀ THI TOÁN VÀO LỚP 10 CHUYÊN KHOA HỌC TỰ NHIÊN HÀ NỘI NĂM 2022 CÓ LỜI GIẢI CHI TIẾT.pdf
TUYỂN TẬP ĐỀ THI TOÁN VÀO LỚP 10 CHUYÊN KHOA HỌC TỰ NHIÊN HÀ NỘI NĂM 2022 CÓ LỜI GIẢI CHI TIẾT.pdf
TUYỂN TẬP ĐỀ THI TOÁN VÀO LỚP 10 CHUYÊN KHOA HỌC TỰ NHIÊN HÀ NỘI NĂM 2022 CÓ LỜI GIẢI CHI TIẾT.pdf
TUYỂN TẬP ĐỀ THI TOÁN VÀO LỚP 10 CHUYÊN KHOA HỌC TỰ NHIÊN HÀ NỘI NĂM 2022 CÓ LỜI GIẢI CHI TIẾT.pdf
TUYỂN TẬP ĐỀ THI TOÁN VÀO LỚP 10 CHUYÊN KHOA HỌC TỰ NHIÊN HÀ NỘI NĂM 2022 CÓ LỜI GIẢI CHI TIẾT.pdf
TUYỂN TẬP ĐỀ THI TOÁN VÀO LỚP 10 CHUYÊN KHOA HỌC TỰ NHIÊN HÀ NỘI NĂM 2022 CÓ LỜI GIẢI CHI TIẾT.pdf
TUYỂN TẬP ĐỀ THI TOÁN VÀO LỚP 10 CHUYÊN KHOA HỌC TỰ NHIÊN HÀ NỘI NĂM 2022 CÓ LỜI GIẢI CHI TIẾT.pdf
TUYỂN TẬP ĐỀ THI TOÁN VÀO LỚP 10 CHUYÊN KHOA HỌC TỰ NHIÊN HÀ NỘI NĂM 2022 CÓ LỜI GIẢI CHI TIẾT.pdf
TUYỂN TẬP ĐỀ THI TOÁN VÀO LỚP 10 CHUYÊN KHOA HỌC TỰ NHIÊN HÀ NỘI NĂM 2022 CÓ LỜI GIẢI CHI TIẾT.pdf
TUYỂN TẬP ĐỀ THI TOÁN VÀO LỚP 10 CHUYÊN KHOA HỌC TỰ NHIÊN HÀ NỘI NĂM 2022 CÓ LỜI GIẢI CHI TIẾT.pdf
TUYỂN TẬP ĐỀ THI TOÁN VÀO LỚP 10 CHUYÊN KHOA HỌC TỰ NHIÊN HÀ NỘI NĂM 2022 CÓ LỜI GIẢI CHI TIẾT.pdf
TUYỂN TẬP ĐỀ THI TOÁN VÀO LỚP 10 CHUYÊN KHOA HỌC TỰ NHIÊN HÀ NỘI NĂM 2022 CÓ LỜI GIẢI CHI TIẾT.pdf
TUYỂN TẬP ĐỀ THI TOÁN VÀO LỚP 10 CHUYÊN KHOA HỌC TỰ NHIÊN HÀ NỘI NĂM 2022 CÓ LỜI GIẢI CHI TIẾT.pdf
TUYỂN TẬP ĐỀ THI TOÁN VÀO LỚP 10 CHUYÊN KHOA HỌC TỰ NHIÊN HÀ NỘI NĂM 2022 CÓ LỜI GIẢI CHI TIẾT.pdf
TUYỂN TẬP ĐỀ THI TOÁN VÀO LỚP 10 CHUYÊN KHOA HỌC TỰ NHIÊN HÀ NỘI NĂM 2022 CÓ LỜI GIẢI CHI TIẾT.pdf
TUYỂN TẬP ĐỀ THI TOÁN VÀO LỚP 10 CHUYÊN KHOA HỌC TỰ NHIÊN HÀ NỘI NĂM 2022 CÓ LỜI GIẢI CHI TIẾT.pdf
TUYỂN TẬP ĐỀ THI TOÁN VÀO LỚP 10 CHUYÊN KHOA HỌC TỰ NHIÊN HÀ NỘI NĂM 2022 CÓ LỜI GIẢI CHI TIẾT.pdf
TUYỂN TẬP ĐỀ THI TOÁN VÀO LỚP 10 CHUYÊN KHOA HỌC TỰ NHIÊN HÀ NỘI NĂM 2022 CÓ LỜI GIẢI CHI TIẾT.pdf

More Related Content

Similar to TUYỂN TẬP ĐỀ THI TOÁN VÀO LỚP 10 CHUYÊN KHOA HỌC TỰ NHIÊN HÀ NỘI NĂM 2022 CÓ LỜI GIẢI CHI TIẾT.pdf

TUYỂN TẬP 16 ĐỀ THI THỬ MỚI NHẤT CÁC TRƯỜNG HÀ NỘI 2021-2022.pdf
TUYỂN TẬP 16 ĐỀ THI THỬ MỚI NHẤT CÁC TRƯỜNG HÀ NỘI 2021-2022.pdfTUYỂN TẬP 16 ĐỀ THI THỬ MỚI NHẤT CÁC TRƯỜNG HÀ NỘI 2021-2022.pdf
TUYỂN TẬP 16 ĐỀ THI THỬ MỚI NHẤT CÁC TRƯỜNG HÀ NỘI 2021-2022.pdf
Blue.Sky Blue.Sky
 
BỘ ĐỀ CHÍNH THỨC TUYỂN SINH VÀO LỚP 10 THPT CHUYÊN TOÁN CÁC TỈNH NĂM HỌC 2023...
BỘ ĐỀ CHÍNH THỨC TUYỂN SINH VÀO LỚP 10 THPT CHUYÊN TOÁN CÁC TỈNH NĂM HỌC 2023...BỘ ĐỀ CHÍNH THỨC TUYỂN SINH VÀO LỚP 10 THPT CHUYÊN TOÁN CÁC TỈNH NĂM HỌC 2023...
BỘ ĐỀ CHÍNH THỨC TUYỂN SINH VÀO LỚP 10 THPT CHUYÊN TOÁN CÁC TỈNH NĂM HỌC 2023...
Nguyen Thanh Tu Collection
 
45 ĐỀ TUYỂN SINH VÀO LỚP 10 NĂM 2012
45 ĐỀ TUYỂN SINH VÀO LỚP 10 NĂM 201245 ĐỀ TUYỂN SINH VÀO LỚP 10 NĂM 2012
45 ĐỀ TUYỂN SINH VÀO LỚP 10 NĂM 2012
Blue.Sky Blue.Sky
 
Đề thi HSG Toán 9 Thái Bình năm 2011 - 2012
Đề thi HSG Toán 9 Thái Bình năm 2011 - 2012Đề thi HSG Toán 9 Thái Bình năm 2011 - 2012
Đề thi HSG Toán 9 Thái Bình năm 2011 - 2012
Bồi Dưỡng HSG Toán Lớp 3
 
Sach luyen de toan 2014 p1 (thay hung)
Sach luyen de toan 2014 p1 (thay hung)Sach luyen de toan 2014 p1 (thay hung)
Sach luyen de toan 2014 p1 (thay hung)
KyNang Toan
 
(Chuyendeonthi.wordpress.com) de thi toan on thi dh hay va dac sac phan 1
(Chuyendeonthi.wordpress.com) de thi toan on thi dh hay va dac sac phan 1(Chuyendeonthi.wordpress.com) de thi toan on thi dh hay va dac sac phan 1
(Chuyendeonthi.wordpress.com) de thi toan on thi dh hay va dac sac phan 1
Kings Kingsley
 
[Htq] toan 9 hsg tp hn dap an
[Htq] toan 9 hsg tp hn   dap an[Htq] toan 9 hsg tp hn   dap an
[Htq] toan 9 hsg tp hn dap an
Hồng Quang
 
BỘ ĐỀ CHÍNH THỨC + TÁCH ĐỀ + ĐỀ LUYỆN THI VÀO LỚP 10 CHUYÊN TOÁN CÁC TỈNH NĂM...
BỘ ĐỀ CHÍNH THỨC + TÁCH ĐỀ + ĐỀ LUYỆN THI VÀO LỚP 10 CHUYÊN TOÁN CÁC TỈNH NĂM...BỘ ĐỀ CHÍNH THỨC + TÁCH ĐỀ + ĐỀ LUYỆN THI VÀO LỚP 10 CHUYÊN TOÁN CÁC TỈNH NĂM...
BỘ ĐỀ CHÍNH THỨC + TÁCH ĐỀ + ĐỀ LUYỆN THI VÀO LỚP 10 CHUYÊN TOÁN CÁC TỈNH NĂM...
Nguyen Thanh Tu Collection
 
BỘ ĐỀ CHÍNH THỨC TUYỂN SINH VÀO LỚP 10 THPT MÔN TOÁN CÁC TỈNH NĂM HỌC 2023-20...
BỘ ĐỀ CHÍNH THỨC TUYỂN SINH VÀO LỚP 10 THPT MÔN TOÁN CÁC TỈNH NĂM HỌC 2023-20...BỘ ĐỀ CHÍNH THỨC TUYỂN SINH VÀO LỚP 10 THPT MÔN TOÁN CÁC TỈNH NĂM HỌC 2023-20...
BỘ ĐỀ CHÍNH THỨC TUYỂN SINH VÀO LỚP 10 THPT MÔN TOÁN CÁC TỈNH NĂM HỌC 2023-20...
Nguyen Thanh Tu Collection
 
65 Đề Thi Học Sinh Giỏi Toán 6 Có Đáp Án.doc
65 Đề Thi Học Sinh Giỏi Toán 6 Có Đáp Án.doc65 Đề Thi Học Sinh Giỏi Toán 6 Có Đáp Án.doc
65 Đề Thi Học Sinh Giỏi Toán 6 Có Đáp Án.doc
LanNguyen176907
 
Đề thi HSG Toán 9 Thanh Hóa năm 2016 - 2017
Đề thi HSG Toán 9 Thanh Hóa năm 2016 - 2017Đề thi HSG Toán 9 Thanh Hóa năm 2016 - 2017
Đề thi HSG Toán 9 Thanh Hóa năm 2016 - 2017
Bồi Dưỡng HSG Toán Lớp 3
 
BỘ ĐỀ CHÍNH THỨC KỲ THI TUYẾN SINH LỚP 10 THPT MÔN TOÁN CHUYÊN CÁC TỈNH NĂM H...
BỘ ĐỀ CHÍNH THỨC KỲ THI TUYẾN SINH LỚP 10 THPT MÔN TOÁN CHUYÊN CÁC TỈNH NĂM H...BỘ ĐỀ CHÍNH THỨC KỲ THI TUYẾN SINH LỚP 10 THPT MÔN TOÁN CHUYÊN CÁC TỈNH NĂM H...
BỘ ĐỀ CHÍNH THỨC KỲ THI TUYẾN SINH LỚP 10 THPT MÔN TOÁN CHUYÊN CÁC TỈNH NĂM H...
Nguyen Thanh Tu Collection
 
40 de-thi-toan-vao-10-chon-loc
40 de-thi-toan-vao-10-chon-loc40 de-thi-toan-vao-10-chon-loc
40 de-thi-toan-vao-10-chon-loc
Haianh Haianhngungok
 
De chuyen2009
De chuyen2009De chuyen2009
De chuyen2009Toan Isi
 
MA TRẬN + ĐẶC TẢ + ĐỀ KIỂM TRA GIỮA HỌC KÌ 2 MÔN TOÁN – LỚP 8 KẾT NỐI TRI THỨ...
MA TRẬN + ĐẶC TẢ + ĐỀ KIỂM TRA GIỮA HỌC KÌ 2 MÔN TOÁN – LỚP 8 KẾT NỐI TRI THỨ...MA TRẬN + ĐẶC TẢ + ĐỀ KIỂM TRA GIỮA HỌC KÌ 2 MÔN TOÁN – LỚP 8 KẾT NỐI TRI THỨ...
MA TRẬN + ĐẶC TẢ + ĐỀ KIỂM TRA GIỮA HỌC KÌ 2 MÔN TOÁN – LỚP 8 KẾT NỐI TRI THỨ...
Nguyen Thanh Tu Collection
 
Tuyen chon de thi vao 10 chuyen toan cac tinh
Tuyen chon de thi vao 10 chuyen toan cac tinhTuyen chon de thi vao 10 chuyen toan cac tinh
Tuyen chon de thi vao 10 chuyen toan cac tinh
Hoàng Quý
 
TUYỂN TẬP ĐỀ THI BỒI DƯỠNG HỌC SINH GIỎI TOÁN LỚP 8 NĂM HỌC 2022-2023 (30 ĐỀ ...
TUYỂN TẬP ĐỀ THI BỒI DƯỠNG HỌC SINH GIỎI TOÁN LỚP 8 NĂM HỌC 2022-2023 (30 ĐỀ ...TUYỂN TẬP ĐỀ THI BỒI DƯỠNG HỌC SINH GIỎI TOÁN LỚP 8 NĂM HỌC 2022-2023 (30 ĐỀ ...
TUYỂN TẬP ĐỀ THI BỒI DƯỠNG HỌC SINH GIỎI TOÁN LỚP 8 NĂM HỌC 2022-2023 (30 ĐỀ ...
Nguyen Thanh Tu Collection
 
De tsl10 toan phu tho chuyen 13-14
De tsl10 toan phu tho chuyen  13-14De tsl10 toan phu tho chuyen  13-14
De tsl10 toan phu tho chuyen 13-14Toan Isi
 
TUYỂN TẬP 23 CHUYÊN ĐỀ ĐẠI SỐ BỒI DƯỠNG HỌC SINH GIỎI TOÁN LỚP 9 - CÓ LỜI GIẢ...
TUYỂN TẬP 23 CHUYÊN ĐỀ ĐẠI SỐ BỒI DƯỠNG HỌC SINH GIỎI TOÁN LỚP 9 - CÓ LỜI GIẢ...TUYỂN TẬP 23 CHUYÊN ĐỀ ĐẠI SỐ BỒI DƯỠNG HỌC SINH GIỎI TOÁN LỚP 9 - CÓ LỜI GIẢ...
TUYỂN TẬP 23 CHUYÊN ĐỀ ĐẠI SỐ BỒI DƯỠNG HỌC SINH GIỎI TOÁN LỚP 9 - CÓ LỜI GIẢ...
Nguyen Thanh Tu Collection
 
Đề thi HSG Toán 9 Phú Thọ năm 2009 - 2010
Đề thi HSG Toán 9 Phú Thọ năm 2009 - 2010Đề thi HSG Toán 9 Phú Thọ năm 2009 - 2010
Đề thi HSG Toán 9 Phú Thọ năm 2009 - 2010
Bồi Dưỡng HSG Toán Lớp 3
 

Similar to TUYỂN TẬP ĐỀ THI TOÁN VÀO LỚP 10 CHUYÊN KHOA HỌC TỰ NHIÊN HÀ NỘI NĂM 2022 CÓ LỜI GIẢI CHI TIẾT.pdf (20)

TUYỂN TẬP 16 ĐỀ THI THỬ MỚI NHẤT CÁC TRƯỜNG HÀ NỘI 2021-2022.pdf
TUYỂN TẬP 16 ĐỀ THI THỬ MỚI NHẤT CÁC TRƯỜNG HÀ NỘI 2021-2022.pdfTUYỂN TẬP 16 ĐỀ THI THỬ MỚI NHẤT CÁC TRƯỜNG HÀ NỘI 2021-2022.pdf
TUYỂN TẬP 16 ĐỀ THI THỬ MỚI NHẤT CÁC TRƯỜNG HÀ NỘI 2021-2022.pdf
 
BỘ ĐỀ CHÍNH THỨC TUYỂN SINH VÀO LỚP 10 THPT CHUYÊN TOÁN CÁC TỈNH NĂM HỌC 2023...
BỘ ĐỀ CHÍNH THỨC TUYỂN SINH VÀO LỚP 10 THPT CHUYÊN TOÁN CÁC TỈNH NĂM HỌC 2023...BỘ ĐỀ CHÍNH THỨC TUYỂN SINH VÀO LỚP 10 THPT CHUYÊN TOÁN CÁC TỈNH NĂM HỌC 2023...
BỘ ĐỀ CHÍNH THỨC TUYỂN SINH VÀO LỚP 10 THPT CHUYÊN TOÁN CÁC TỈNH NĂM HỌC 2023...
 
45 ĐỀ TUYỂN SINH VÀO LỚP 10 NĂM 2012
45 ĐỀ TUYỂN SINH VÀO LỚP 10 NĂM 201245 ĐỀ TUYỂN SINH VÀO LỚP 10 NĂM 2012
45 ĐỀ TUYỂN SINH VÀO LỚP 10 NĂM 2012
 
Đề thi HSG Toán 9 Thái Bình năm 2011 - 2012
Đề thi HSG Toán 9 Thái Bình năm 2011 - 2012Đề thi HSG Toán 9 Thái Bình năm 2011 - 2012
Đề thi HSG Toán 9 Thái Bình năm 2011 - 2012
 
Sach luyen de toan 2014 p1 (thay hung)
Sach luyen de toan 2014 p1 (thay hung)Sach luyen de toan 2014 p1 (thay hung)
Sach luyen de toan 2014 p1 (thay hung)
 
(Chuyendeonthi.wordpress.com) de thi toan on thi dh hay va dac sac phan 1
(Chuyendeonthi.wordpress.com) de thi toan on thi dh hay va dac sac phan 1(Chuyendeonthi.wordpress.com) de thi toan on thi dh hay va dac sac phan 1
(Chuyendeonthi.wordpress.com) de thi toan on thi dh hay va dac sac phan 1
 
[Htq] toan 9 hsg tp hn dap an
[Htq] toan 9 hsg tp hn   dap an[Htq] toan 9 hsg tp hn   dap an
[Htq] toan 9 hsg tp hn dap an
 
BỘ ĐỀ CHÍNH THỨC + TÁCH ĐỀ + ĐỀ LUYỆN THI VÀO LỚP 10 CHUYÊN TOÁN CÁC TỈNH NĂM...
BỘ ĐỀ CHÍNH THỨC + TÁCH ĐỀ + ĐỀ LUYỆN THI VÀO LỚP 10 CHUYÊN TOÁN CÁC TỈNH NĂM...BỘ ĐỀ CHÍNH THỨC + TÁCH ĐỀ + ĐỀ LUYỆN THI VÀO LỚP 10 CHUYÊN TOÁN CÁC TỈNH NĂM...
BỘ ĐỀ CHÍNH THỨC + TÁCH ĐỀ + ĐỀ LUYỆN THI VÀO LỚP 10 CHUYÊN TOÁN CÁC TỈNH NĂM...
 
BỘ ĐỀ CHÍNH THỨC TUYỂN SINH VÀO LỚP 10 THPT MÔN TOÁN CÁC TỈNH NĂM HỌC 2023-20...
BỘ ĐỀ CHÍNH THỨC TUYỂN SINH VÀO LỚP 10 THPT MÔN TOÁN CÁC TỈNH NĂM HỌC 2023-20...BỘ ĐỀ CHÍNH THỨC TUYỂN SINH VÀO LỚP 10 THPT MÔN TOÁN CÁC TỈNH NĂM HỌC 2023-20...
BỘ ĐỀ CHÍNH THỨC TUYỂN SINH VÀO LỚP 10 THPT MÔN TOÁN CÁC TỈNH NĂM HỌC 2023-20...
 
65 Đề Thi Học Sinh Giỏi Toán 6 Có Đáp Án.doc
65 Đề Thi Học Sinh Giỏi Toán 6 Có Đáp Án.doc65 Đề Thi Học Sinh Giỏi Toán 6 Có Đáp Án.doc
65 Đề Thi Học Sinh Giỏi Toán 6 Có Đáp Án.doc
 
Đề thi HSG Toán 9 Thanh Hóa năm 2016 - 2017
Đề thi HSG Toán 9 Thanh Hóa năm 2016 - 2017Đề thi HSG Toán 9 Thanh Hóa năm 2016 - 2017
Đề thi HSG Toán 9 Thanh Hóa năm 2016 - 2017
 
BỘ ĐỀ CHÍNH THỨC KỲ THI TUYẾN SINH LỚP 10 THPT MÔN TOÁN CHUYÊN CÁC TỈNH NĂM H...
BỘ ĐỀ CHÍNH THỨC KỲ THI TUYẾN SINH LỚP 10 THPT MÔN TOÁN CHUYÊN CÁC TỈNH NĂM H...BỘ ĐỀ CHÍNH THỨC KỲ THI TUYẾN SINH LỚP 10 THPT MÔN TOÁN CHUYÊN CÁC TỈNH NĂM H...
BỘ ĐỀ CHÍNH THỨC KỲ THI TUYẾN SINH LỚP 10 THPT MÔN TOÁN CHUYÊN CÁC TỈNH NĂM H...
 
40 de-thi-toan-vao-10-chon-loc
40 de-thi-toan-vao-10-chon-loc40 de-thi-toan-vao-10-chon-loc
40 de-thi-toan-vao-10-chon-loc
 
De chuyen2009
De chuyen2009De chuyen2009
De chuyen2009
 
MA TRẬN + ĐẶC TẢ + ĐỀ KIỂM TRA GIỮA HỌC KÌ 2 MÔN TOÁN – LỚP 8 KẾT NỐI TRI THỨ...
MA TRẬN + ĐẶC TẢ + ĐỀ KIỂM TRA GIỮA HỌC KÌ 2 MÔN TOÁN – LỚP 8 KẾT NỐI TRI THỨ...MA TRẬN + ĐẶC TẢ + ĐỀ KIỂM TRA GIỮA HỌC KÌ 2 MÔN TOÁN – LỚP 8 KẾT NỐI TRI THỨ...
MA TRẬN + ĐẶC TẢ + ĐỀ KIỂM TRA GIỮA HỌC KÌ 2 MÔN TOÁN – LỚP 8 KẾT NỐI TRI THỨ...
 
Tuyen chon de thi vao 10 chuyen toan cac tinh
Tuyen chon de thi vao 10 chuyen toan cac tinhTuyen chon de thi vao 10 chuyen toan cac tinh
Tuyen chon de thi vao 10 chuyen toan cac tinh
 
TUYỂN TẬP ĐỀ THI BỒI DƯỠNG HỌC SINH GIỎI TOÁN LỚP 8 NĂM HỌC 2022-2023 (30 ĐỀ ...
TUYỂN TẬP ĐỀ THI BỒI DƯỠNG HỌC SINH GIỎI TOÁN LỚP 8 NĂM HỌC 2022-2023 (30 ĐỀ ...TUYỂN TẬP ĐỀ THI BỒI DƯỠNG HỌC SINH GIỎI TOÁN LỚP 8 NĂM HỌC 2022-2023 (30 ĐỀ ...
TUYỂN TẬP ĐỀ THI BỒI DƯỠNG HỌC SINH GIỎI TOÁN LỚP 8 NĂM HỌC 2022-2023 (30 ĐỀ ...
 
De tsl10 toan phu tho chuyen 13-14
De tsl10 toan phu tho chuyen  13-14De tsl10 toan phu tho chuyen  13-14
De tsl10 toan phu tho chuyen 13-14
 
TUYỂN TẬP 23 CHUYÊN ĐỀ ĐẠI SỐ BỒI DƯỠNG HỌC SINH GIỎI TOÁN LỚP 9 - CÓ LỜI GIẢ...
TUYỂN TẬP 23 CHUYÊN ĐỀ ĐẠI SỐ BỒI DƯỠNG HỌC SINH GIỎI TOÁN LỚP 9 - CÓ LỜI GIẢ...TUYỂN TẬP 23 CHUYÊN ĐỀ ĐẠI SỐ BỒI DƯỠNG HỌC SINH GIỎI TOÁN LỚP 9 - CÓ LỜI GIẢ...
TUYỂN TẬP 23 CHUYÊN ĐỀ ĐẠI SỐ BỒI DƯỠNG HỌC SINH GIỎI TOÁN LỚP 9 - CÓ LỜI GIẢ...
 
Đề thi HSG Toán 9 Phú Thọ năm 2009 - 2010
Đề thi HSG Toán 9 Phú Thọ năm 2009 - 2010Đề thi HSG Toán 9 Phú Thọ năm 2009 - 2010
Đề thi HSG Toán 9 Phú Thọ năm 2009 - 2010
 

More from Nguyen Thanh Tu Collection

BÀI TẬP BỔ TRỢ TIẾNG ANH LỚP 9 CẢ NĂM - GLOBAL SUCCESS - NĂM HỌC 2024-2025 - ...
BÀI TẬP BỔ TRỢ TIẾNG ANH LỚP 9 CẢ NĂM - GLOBAL SUCCESS - NĂM HỌC 2024-2025 - ...BÀI TẬP BỔ TRỢ TIẾNG ANH LỚP 9 CẢ NĂM - GLOBAL SUCCESS - NĂM HỌC 2024-2025 - ...
BÀI TẬP BỔ TRỢ TIẾNG ANH LỚP 9 CẢ NĂM - GLOBAL SUCCESS - NĂM HỌC 2024-2025 - ...
Nguyen Thanh Tu Collection
 
BÀI TẬP DẠY THÊM TIẾNG ANH LỚP 7 CẢ NĂM FRIENDS PLUS SÁCH CHÂN TRỜI SÁNG TẠO ...
BÀI TẬP DẠY THÊM TIẾNG ANH LỚP 7 CẢ NĂM FRIENDS PLUS SÁCH CHÂN TRỜI SÁNG TẠO ...BÀI TẬP DẠY THÊM TIẾNG ANH LỚP 7 CẢ NĂM FRIENDS PLUS SÁCH CHÂN TRỜI SÁNG TẠO ...
BÀI TẬP DẠY THÊM TIẾNG ANH LỚP 7 CẢ NĂM FRIENDS PLUS SÁCH CHÂN TRỜI SÁNG TẠO ...
Nguyen Thanh Tu Collection
 
CHUYÊN ĐỀ DẠY THÊM HÓA HỌC LỚP 10 - SÁCH MỚI - FORM BÀI TẬP 2025 (DÙNG CHUNG ...
CHUYÊN ĐỀ DẠY THÊM HÓA HỌC LỚP 10 - SÁCH MỚI - FORM BÀI TẬP 2025 (DÙNG CHUNG ...CHUYÊN ĐỀ DẠY THÊM HÓA HỌC LỚP 10 - SÁCH MỚI - FORM BÀI TẬP 2025 (DÙNG CHUNG ...
CHUYÊN ĐỀ DẠY THÊM HÓA HỌC LỚP 10 - SÁCH MỚI - FORM BÀI TẬP 2025 (DÙNG CHUNG ...
Nguyen Thanh Tu Collection
 
BÀI TẬP BỔ TRỢ TIẾNG ANH 8 CẢ NĂM - GLOBAL SUCCESS - NĂM HỌC 2023-2024 (CÓ FI...
BÀI TẬP BỔ TRỢ TIẾNG ANH 8 CẢ NĂM - GLOBAL SUCCESS - NĂM HỌC 2023-2024 (CÓ FI...BÀI TẬP BỔ TRỢ TIẾNG ANH 8 CẢ NĂM - GLOBAL SUCCESS - NĂM HỌC 2023-2024 (CÓ FI...
BÀI TẬP BỔ TRỢ TIẾNG ANH 8 CẢ NĂM - GLOBAL SUCCESS - NĂM HỌC 2023-2024 (CÓ FI...
Nguyen Thanh Tu Collection
 
BÀI TẬP BỔ TRỢ TIẾNG ANH I-LEARN SMART WORLD 9 CẢ NĂM CÓ TEST THEO UNIT NĂM H...
BÀI TẬP BỔ TRỢ TIẾNG ANH I-LEARN SMART WORLD 9 CẢ NĂM CÓ TEST THEO UNIT NĂM H...BÀI TẬP BỔ TRỢ TIẾNG ANH I-LEARN SMART WORLD 9 CẢ NĂM CÓ TEST THEO UNIT NĂM H...
BÀI TẬP BỔ TRỢ TIẾNG ANH I-LEARN SMART WORLD 9 CẢ NĂM CÓ TEST THEO UNIT NĂM H...
Nguyen Thanh Tu Collection
 
CHUYÊN ĐỀ BỒI DƯỠNG HỌC SINH GIỎI KHOA HỌC TỰ NHIÊN 9 CHƯƠNG TRÌNH MỚI - PHẦN...
CHUYÊN ĐỀ BỒI DƯỠNG HỌC SINH GIỎI KHOA HỌC TỰ NHIÊN 9 CHƯƠNG TRÌNH MỚI - PHẦN...CHUYÊN ĐỀ BỒI DƯỠNG HỌC SINH GIỎI KHOA HỌC TỰ NHIÊN 9 CHƯƠNG TRÌNH MỚI - PHẦN...
CHUYÊN ĐỀ BỒI DƯỠNG HỌC SINH GIỎI KHOA HỌC TỰ NHIÊN 9 CHƯƠNG TRÌNH MỚI - PHẦN...
Nguyen Thanh Tu Collection
 
BÀI TẬP BỔ TRỢ TIẾNG ANH GLOBAL SUCCESS LỚP 3 - CẢ NĂM (CÓ FILE NGHE VÀ ĐÁP Á...
BÀI TẬP BỔ TRỢ TIẾNG ANH GLOBAL SUCCESS LỚP 3 - CẢ NĂM (CÓ FILE NGHE VÀ ĐÁP Á...BÀI TẬP BỔ TRỢ TIẾNG ANH GLOBAL SUCCESS LỚP 3 - CẢ NĂM (CÓ FILE NGHE VÀ ĐÁP Á...
BÀI TẬP BỔ TRỢ TIẾNG ANH GLOBAL SUCCESS LỚP 3 - CẢ NĂM (CÓ FILE NGHE VÀ ĐÁP Á...
Nguyen Thanh Tu Collection
 
98 BÀI LUYỆN NGHE TUYỂN SINH VÀO LỚP 10 TIẾNG ANH DẠNG TRẮC NGHIỆM 4 CÂU TRẢ ...
98 BÀI LUYỆN NGHE TUYỂN SINH VÀO LỚP 10 TIẾNG ANH DẠNG TRẮC NGHIỆM 4 CÂU TRẢ ...98 BÀI LUYỆN NGHE TUYỂN SINH VÀO LỚP 10 TIẾNG ANH DẠNG TRẮC NGHIỆM 4 CÂU TRẢ ...
98 BÀI LUYỆN NGHE TUYỂN SINH VÀO LỚP 10 TIẾNG ANH DẠNG TRẮC NGHIỆM 4 CÂU TRẢ ...
Nguyen Thanh Tu Collection
 
GIÁO ÁN DẠY THÊM (KẾ HOẠCH BÀI BUỔI 2) - TIẾNG ANH 8 GLOBAL SUCCESS (2 CỘT) N...
GIÁO ÁN DẠY THÊM (KẾ HOẠCH BÀI BUỔI 2) - TIẾNG ANH 8 GLOBAL SUCCESS (2 CỘT) N...GIÁO ÁN DẠY THÊM (KẾ HOẠCH BÀI BUỔI 2) - TIẾNG ANH 8 GLOBAL SUCCESS (2 CỘT) N...
GIÁO ÁN DẠY THÊM (KẾ HOẠCH BÀI BUỔI 2) - TIẾNG ANH 8 GLOBAL SUCCESS (2 CỘT) N...
Nguyen Thanh Tu Collection
 
Nghiên cứu cơ chế và động học phản ứng giữa hợp chất Aniline (C6H5NH2) với gố...
Nghiên cứu cơ chế và động học phản ứng giữa hợp chất Aniline (C6H5NH2) với gố...Nghiên cứu cơ chế và động học phản ứng giữa hợp chất Aniline (C6H5NH2) với gố...
Nghiên cứu cơ chế và động học phản ứng giữa hợp chất Aniline (C6H5NH2) với gố...
Nguyen Thanh Tu Collection
 
50 ĐỀ LUYỆN THI IOE LỚP 9 - NĂM HỌC 2022-2023 (CÓ LINK HÌNH, FILE AUDIO VÀ ĐÁ...
50 ĐỀ LUYỆN THI IOE LỚP 9 - NĂM HỌC 2022-2023 (CÓ LINK HÌNH, FILE AUDIO VÀ ĐÁ...50 ĐỀ LUYỆN THI IOE LỚP 9 - NĂM HỌC 2022-2023 (CÓ LINK HÌNH, FILE AUDIO VÀ ĐÁ...
50 ĐỀ LUYỆN THI IOE LỚP 9 - NĂM HỌC 2022-2023 (CÓ LINK HÌNH, FILE AUDIO VÀ ĐÁ...
Nguyen Thanh Tu Collection
 
BÀI TẬP DẠY THÊM TOÁN LỚP 12 SÁCH MỚI THEO FORM THI MỚI BGD 2025 - CÁNH DIỀU ...
BÀI TẬP DẠY THÊM TOÁN LỚP 12 SÁCH MỚI THEO FORM THI MỚI BGD 2025 - CÁNH DIỀU ...BÀI TẬP DẠY THÊM TOÁN LỚP 12 SÁCH MỚI THEO FORM THI MỚI BGD 2025 - CÁNH DIỀU ...
BÀI TẬP DẠY THÊM TOÁN LỚP 12 SÁCH MỚI THEO FORM THI MỚI BGD 2025 - CÁNH DIỀU ...
Nguyen Thanh Tu Collection
 
TÀI LIỆU DẠY THÊM HÓA HỌC 12 - SÁCH MỚI (BẢN HS+GV) (FORM BÀI TẬP 2025 CHUNG ...
TÀI LIỆU DẠY THÊM HÓA HỌC 12 - SÁCH MỚI (BẢN HS+GV) (FORM BÀI TẬP 2025 CHUNG ...TÀI LIỆU DẠY THÊM HÓA HỌC 12 - SÁCH MỚI (BẢN HS+GV) (FORM BÀI TẬP 2025 CHUNG ...
TÀI LIỆU DẠY THÊM HÓA HỌC 12 - SÁCH MỚI (BẢN HS+GV) (FORM BÀI TẬP 2025 CHUNG ...
Nguyen Thanh Tu Collection
 
ĐỀ THI THỬ TUYỂN SINH VÀO LỚP 10 THPT MÔN TOÁN CÁC TỈNH NĂM HỌC 2023-2024 CÓ ...
ĐỀ THI THỬ TUYỂN SINH VÀO LỚP 10 THPT MÔN TOÁN CÁC TỈNH NĂM HỌC 2023-2024 CÓ ...ĐỀ THI THỬ TUYỂN SINH VÀO LỚP 10 THPT MÔN TOÁN CÁC TỈNH NĂM HỌC 2023-2024 CÓ ...
ĐỀ THI THỬ TUYỂN SINH VÀO LỚP 10 THPT MÔN TOÁN CÁC TỈNH NĂM HỌC 2023-2024 CÓ ...
Nguyen Thanh Tu Collection
 
BÀI TẬP DẠY THÊM TOÁN LỚP 12 SÁCH MỚI THEO FORM THI MỚI BGD 2025 - CHÂN TRỜI ...
BÀI TẬP DẠY THÊM TOÁN LỚP 12 SÁCH MỚI THEO FORM THI MỚI BGD 2025 - CHÂN TRỜI ...BÀI TẬP DẠY THÊM TOÁN LỚP 12 SÁCH MỚI THEO FORM THI MỚI BGD 2025 - CHÂN TRỜI ...
BÀI TẬP DẠY THÊM TOÁN LỚP 12 SÁCH MỚI THEO FORM THI MỚI BGD 2025 - CHÂN TRỜI ...
Nguyen Thanh Tu Collection
 
20 ĐỀ DỰ ĐOÁN - PHÁT TRIỂN ĐỀ MINH HỌA BGD KỲ THI TỐT NGHIỆP THPT NĂM 2024 MÔ...
20 ĐỀ DỰ ĐOÁN - PHÁT TRIỂN ĐỀ MINH HỌA BGD KỲ THI TỐT NGHIỆP THPT NĂM 2024 MÔ...20 ĐỀ DỰ ĐOÁN - PHÁT TRIỂN ĐỀ MINH HỌA BGD KỲ THI TỐT NGHIỆP THPT NĂM 2024 MÔ...
20 ĐỀ DỰ ĐOÁN - PHÁT TRIỂN ĐỀ MINH HỌA BGD KỲ THI TỐT NGHIỆP THPT NĂM 2024 MÔ...
Nguyen Thanh Tu Collection
 
BÀI TẬP DẠY THÊM HÓA HỌC LỚP 12 - CẢ NĂM - THEO FORM THI MỚI BGD 2025 (DÙNG C...
BÀI TẬP DẠY THÊM HÓA HỌC LỚP 12 - CẢ NĂM - THEO FORM THI MỚI BGD 2025 (DÙNG C...BÀI TẬP DẠY THÊM HÓA HỌC LỚP 12 - CẢ NĂM - THEO FORM THI MỚI BGD 2025 (DÙNG C...
BÀI TẬP DẠY THÊM HÓA HỌC LỚP 12 - CẢ NĂM - THEO FORM THI MỚI BGD 2025 (DÙNG C...
Nguyen Thanh Tu Collection
 
BÀI TẬP BỔ TRỢ 4 KỸ NĂNG TIẾNG ANH 11 CẢ NĂM - GLOBAL SUCCESS - NĂM HỌC 2023-...
BÀI TẬP BỔ TRỢ 4 KỸ NĂNG TIẾNG ANH 11 CẢ NĂM - GLOBAL SUCCESS - NĂM HỌC 2023-...BÀI TẬP BỔ TRỢ 4 KỸ NĂNG TIẾNG ANH 11 CẢ NĂM - GLOBAL SUCCESS - NĂM HỌC 2023-...
BÀI TẬP BỔ TRỢ 4 KỸ NĂNG TIẾNG ANH 11 CẢ NĂM - GLOBAL SUCCESS - NĂM HỌC 2023-...
Nguyen Thanh Tu Collection
 
15 ĐỀ THI THỬ TUYỂN SINH VÀO LỚP 10 MÔN TIẾNG ANH NĂM HỌC 2023 - 2024 SỞ GIÁO...
15 ĐỀ THI THỬ TUYỂN SINH VÀO LỚP 10 MÔN TIẾNG ANH NĂM HỌC 2023 - 2024 SỞ GIÁO...15 ĐỀ THI THỬ TUYỂN SINH VÀO LỚP 10 MÔN TIẾNG ANH NĂM HỌC 2023 - 2024 SỞ GIÁO...
15 ĐỀ THI THỬ TUYỂN SINH VÀO LỚP 10 MÔN TIẾNG ANH NĂM HỌC 2023 - 2024 SỞ GIÁO...
Nguyen Thanh Tu Collection
 
30 ĐỀ PHÁT TRIỂN THEO CẤU TRÚC ĐỀ MINH HỌA BGD NGÀY 22-3-2024 KỲ THI TỐT NGHI...
30 ĐỀ PHÁT TRIỂN THEO CẤU TRÚC ĐỀ MINH HỌA BGD NGÀY 22-3-2024 KỲ THI TỐT NGHI...30 ĐỀ PHÁT TRIỂN THEO CẤU TRÚC ĐỀ MINH HỌA BGD NGÀY 22-3-2024 KỲ THI TỐT NGHI...
30 ĐỀ PHÁT TRIỂN THEO CẤU TRÚC ĐỀ MINH HỌA BGD NGÀY 22-3-2024 KỲ THI TỐT NGHI...
Nguyen Thanh Tu Collection
 

More from Nguyen Thanh Tu Collection (20)

BÀI TẬP BỔ TRỢ TIẾNG ANH LỚP 9 CẢ NĂM - GLOBAL SUCCESS - NĂM HỌC 2024-2025 - ...
BÀI TẬP BỔ TRỢ TIẾNG ANH LỚP 9 CẢ NĂM - GLOBAL SUCCESS - NĂM HỌC 2024-2025 - ...BÀI TẬP BỔ TRỢ TIẾNG ANH LỚP 9 CẢ NĂM - GLOBAL SUCCESS - NĂM HỌC 2024-2025 - ...
BÀI TẬP BỔ TRỢ TIẾNG ANH LỚP 9 CẢ NĂM - GLOBAL SUCCESS - NĂM HỌC 2024-2025 - ...
 
BÀI TẬP DẠY THÊM TIẾNG ANH LỚP 7 CẢ NĂM FRIENDS PLUS SÁCH CHÂN TRỜI SÁNG TẠO ...
BÀI TẬP DẠY THÊM TIẾNG ANH LỚP 7 CẢ NĂM FRIENDS PLUS SÁCH CHÂN TRỜI SÁNG TẠO ...BÀI TẬP DẠY THÊM TIẾNG ANH LỚP 7 CẢ NĂM FRIENDS PLUS SÁCH CHÂN TRỜI SÁNG TẠO ...
BÀI TẬP DẠY THÊM TIẾNG ANH LỚP 7 CẢ NĂM FRIENDS PLUS SÁCH CHÂN TRỜI SÁNG TẠO ...
 
CHUYÊN ĐỀ DẠY THÊM HÓA HỌC LỚP 10 - SÁCH MỚI - FORM BÀI TẬP 2025 (DÙNG CHUNG ...
CHUYÊN ĐỀ DẠY THÊM HÓA HỌC LỚP 10 - SÁCH MỚI - FORM BÀI TẬP 2025 (DÙNG CHUNG ...CHUYÊN ĐỀ DẠY THÊM HÓA HỌC LỚP 10 - SÁCH MỚI - FORM BÀI TẬP 2025 (DÙNG CHUNG ...
CHUYÊN ĐỀ DẠY THÊM HÓA HỌC LỚP 10 - SÁCH MỚI - FORM BÀI TẬP 2025 (DÙNG CHUNG ...
 
BÀI TẬP BỔ TRỢ TIẾNG ANH 8 CẢ NĂM - GLOBAL SUCCESS - NĂM HỌC 2023-2024 (CÓ FI...
BÀI TẬP BỔ TRỢ TIẾNG ANH 8 CẢ NĂM - GLOBAL SUCCESS - NĂM HỌC 2023-2024 (CÓ FI...BÀI TẬP BỔ TRỢ TIẾNG ANH 8 CẢ NĂM - GLOBAL SUCCESS - NĂM HỌC 2023-2024 (CÓ FI...
BÀI TẬP BỔ TRỢ TIẾNG ANH 8 CẢ NĂM - GLOBAL SUCCESS - NĂM HỌC 2023-2024 (CÓ FI...
 
BÀI TẬP BỔ TRỢ TIẾNG ANH I-LEARN SMART WORLD 9 CẢ NĂM CÓ TEST THEO UNIT NĂM H...
BÀI TẬP BỔ TRỢ TIẾNG ANH I-LEARN SMART WORLD 9 CẢ NĂM CÓ TEST THEO UNIT NĂM H...BÀI TẬP BỔ TRỢ TIẾNG ANH I-LEARN SMART WORLD 9 CẢ NĂM CÓ TEST THEO UNIT NĂM H...
BÀI TẬP BỔ TRỢ TIẾNG ANH I-LEARN SMART WORLD 9 CẢ NĂM CÓ TEST THEO UNIT NĂM H...
 
CHUYÊN ĐỀ BỒI DƯỠNG HỌC SINH GIỎI KHOA HỌC TỰ NHIÊN 9 CHƯƠNG TRÌNH MỚI - PHẦN...
CHUYÊN ĐỀ BỒI DƯỠNG HỌC SINH GIỎI KHOA HỌC TỰ NHIÊN 9 CHƯƠNG TRÌNH MỚI - PHẦN...CHUYÊN ĐỀ BỒI DƯỠNG HỌC SINH GIỎI KHOA HỌC TỰ NHIÊN 9 CHƯƠNG TRÌNH MỚI - PHẦN...
CHUYÊN ĐỀ BỒI DƯỠNG HỌC SINH GIỎI KHOA HỌC TỰ NHIÊN 9 CHƯƠNG TRÌNH MỚI - PHẦN...
 
BÀI TẬP BỔ TRỢ TIẾNG ANH GLOBAL SUCCESS LỚP 3 - CẢ NĂM (CÓ FILE NGHE VÀ ĐÁP Á...
BÀI TẬP BỔ TRỢ TIẾNG ANH GLOBAL SUCCESS LỚP 3 - CẢ NĂM (CÓ FILE NGHE VÀ ĐÁP Á...BÀI TẬP BỔ TRỢ TIẾNG ANH GLOBAL SUCCESS LỚP 3 - CẢ NĂM (CÓ FILE NGHE VÀ ĐÁP Á...
BÀI TẬP BỔ TRỢ TIẾNG ANH GLOBAL SUCCESS LỚP 3 - CẢ NĂM (CÓ FILE NGHE VÀ ĐÁP Á...
 
98 BÀI LUYỆN NGHE TUYỂN SINH VÀO LỚP 10 TIẾNG ANH DẠNG TRẮC NGHIỆM 4 CÂU TRẢ ...
98 BÀI LUYỆN NGHE TUYỂN SINH VÀO LỚP 10 TIẾNG ANH DẠNG TRẮC NGHIỆM 4 CÂU TRẢ ...98 BÀI LUYỆN NGHE TUYỂN SINH VÀO LỚP 10 TIẾNG ANH DẠNG TRẮC NGHIỆM 4 CÂU TRẢ ...
98 BÀI LUYỆN NGHE TUYỂN SINH VÀO LỚP 10 TIẾNG ANH DẠNG TRẮC NGHIỆM 4 CÂU TRẢ ...
 
GIÁO ÁN DẠY THÊM (KẾ HOẠCH BÀI BUỔI 2) - TIẾNG ANH 8 GLOBAL SUCCESS (2 CỘT) N...
GIÁO ÁN DẠY THÊM (KẾ HOẠCH BÀI BUỔI 2) - TIẾNG ANH 8 GLOBAL SUCCESS (2 CỘT) N...GIÁO ÁN DẠY THÊM (KẾ HOẠCH BÀI BUỔI 2) - TIẾNG ANH 8 GLOBAL SUCCESS (2 CỘT) N...
GIÁO ÁN DẠY THÊM (KẾ HOẠCH BÀI BUỔI 2) - TIẾNG ANH 8 GLOBAL SUCCESS (2 CỘT) N...
 
Nghiên cứu cơ chế và động học phản ứng giữa hợp chất Aniline (C6H5NH2) với gố...
Nghiên cứu cơ chế và động học phản ứng giữa hợp chất Aniline (C6H5NH2) với gố...Nghiên cứu cơ chế và động học phản ứng giữa hợp chất Aniline (C6H5NH2) với gố...
Nghiên cứu cơ chế và động học phản ứng giữa hợp chất Aniline (C6H5NH2) với gố...
 
50 ĐỀ LUYỆN THI IOE LỚP 9 - NĂM HỌC 2022-2023 (CÓ LINK HÌNH, FILE AUDIO VÀ ĐÁ...
50 ĐỀ LUYỆN THI IOE LỚP 9 - NĂM HỌC 2022-2023 (CÓ LINK HÌNH, FILE AUDIO VÀ ĐÁ...50 ĐỀ LUYỆN THI IOE LỚP 9 - NĂM HỌC 2022-2023 (CÓ LINK HÌNH, FILE AUDIO VÀ ĐÁ...
50 ĐỀ LUYỆN THI IOE LỚP 9 - NĂM HỌC 2022-2023 (CÓ LINK HÌNH, FILE AUDIO VÀ ĐÁ...
 
BÀI TẬP DẠY THÊM TOÁN LỚP 12 SÁCH MỚI THEO FORM THI MỚI BGD 2025 - CÁNH DIỀU ...
BÀI TẬP DẠY THÊM TOÁN LỚP 12 SÁCH MỚI THEO FORM THI MỚI BGD 2025 - CÁNH DIỀU ...BÀI TẬP DẠY THÊM TOÁN LỚP 12 SÁCH MỚI THEO FORM THI MỚI BGD 2025 - CÁNH DIỀU ...
BÀI TẬP DẠY THÊM TOÁN LỚP 12 SÁCH MỚI THEO FORM THI MỚI BGD 2025 - CÁNH DIỀU ...
 
TÀI LIỆU DẠY THÊM HÓA HỌC 12 - SÁCH MỚI (BẢN HS+GV) (FORM BÀI TẬP 2025 CHUNG ...
TÀI LIỆU DẠY THÊM HÓA HỌC 12 - SÁCH MỚI (BẢN HS+GV) (FORM BÀI TẬP 2025 CHUNG ...TÀI LIỆU DẠY THÊM HÓA HỌC 12 - SÁCH MỚI (BẢN HS+GV) (FORM BÀI TẬP 2025 CHUNG ...
TÀI LIỆU DẠY THÊM HÓA HỌC 12 - SÁCH MỚI (BẢN HS+GV) (FORM BÀI TẬP 2025 CHUNG ...
 
ĐỀ THI THỬ TUYỂN SINH VÀO LỚP 10 THPT MÔN TOÁN CÁC TỈNH NĂM HỌC 2023-2024 CÓ ...
ĐỀ THI THỬ TUYỂN SINH VÀO LỚP 10 THPT MÔN TOÁN CÁC TỈNH NĂM HỌC 2023-2024 CÓ ...ĐỀ THI THỬ TUYỂN SINH VÀO LỚP 10 THPT MÔN TOÁN CÁC TỈNH NĂM HỌC 2023-2024 CÓ ...
ĐỀ THI THỬ TUYỂN SINH VÀO LỚP 10 THPT MÔN TOÁN CÁC TỈNH NĂM HỌC 2023-2024 CÓ ...
 
BÀI TẬP DẠY THÊM TOÁN LỚP 12 SÁCH MỚI THEO FORM THI MỚI BGD 2025 - CHÂN TRỜI ...
BÀI TẬP DẠY THÊM TOÁN LỚP 12 SÁCH MỚI THEO FORM THI MỚI BGD 2025 - CHÂN TRỜI ...BÀI TẬP DẠY THÊM TOÁN LỚP 12 SÁCH MỚI THEO FORM THI MỚI BGD 2025 - CHÂN TRỜI ...
BÀI TẬP DẠY THÊM TOÁN LỚP 12 SÁCH MỚI THEO FORM THI MỚI BGD 2025 - CHÂN TRỜI ...
 
20 ĐỀ DỰ ĐOÁN - PHÁT TRIỂN ĐỀ MINH HỌA BGD KỲ THI TỐT NGHIỆP THPT NĂM 2024 MÔ...
20 ĐỀ DỰ ĐOÁN - PHÁT TRIỂN ĐỀ MINH HỌA BGD KỲ THI TỐT NGHIỆP THPT NĂM 2024 MÔ...20 ĐỀ DỰ ĐOÁN - PHÁT TRIỂN ĐỀ MINH HỌA BGD KỲ THI TỐT NGHIỆP THPT NĂM 2024 MÔ...
20 ĐỀ DỰ ĐOÁN - PHÁT TRIỂN ĐỀ MINH HỌA BGD KỲ THI TỐT NGHIỆP THPT NĂM 2024 MÔ...
 
BÀI TẬP DẠY THÊM HÓA HỌC LỚP 12 - CẢ NĂM - THEO FORM THI MỚI BGD 2025 (DÙNG C...
BÀI TẬP DẠY THÊM HÓA HỌC LỚP 12 - CẢ NĂM - THEO FORM THI MỚI BGD 2025 (DÙNG C...BÀI TẬP DẠY THÊM HÓA HỌC LỚP 12 - CẢ NĂM - THEO FORM THI MỚI BGD 2025 (DÙNG C...
BÀI TẬP DẠY THÊM HÓA HỌC LỚP 12 - CẢ NĂM - THEO FORM THI MỚI BGD 2025 (DÙNG C...
 
BÀI TẬP BỔ TRỢ 4 KỸ NĂNG TIẾNG ANH 11 CẢ NĂM - GLOBAL SUCCESS - NĂM HỌC 2023-...
BÀI TẬP BỔ TRỢ 4 KỸ NĂNG TIẾNG ANH 11 CẢ NĂM - GLOBAL SUCCESS - NĂM HỌC 2023-...BÀI TẬP BỔ TRỢ 4 KỸ NĂNG TIẾNG ANH 11 CẢ NĂM - GLOBAL SUCCESS - NĂM HỌC 2023-...
BÀI TẬP BỔ TRỢ 4 KỸ NĂNG TIẾNG ANH 11 CẢ NĂM - GLOBAL SUCCESS - NĂM HỌC 2023-...
 
15 ĐỀ THI THỬ TUYỂN SINH VÀO LỚP 10 MÔN TIẾNG ANH NĂM HỌC 2023 - 2024 SỞ GIÁO...
15 ĐỀ THI THỬ TUYỂN SINH VÀO LỚP 10 MÔN TIẾNG ANH NĂM HỌC 2023 - 2024 SỞ GIÁO...15 ĐỀ THI THỬ TUYỂN SINH VÀO LỚP 10 MÔN TIẾNG ANH NĂM HỌC 2023 - 2024 SỞ GIÁO...
15 ĐỀ THI THỬ TUYỂN SINH VÀO LỚP 10 MÔN TIẾNG ANH NĂM HỌC 2023 - 2024 SỞ GIÁO...
 
30 ĐỀ PHÁT TRIỂN THEO CẤU TRÚC ĐỀ MINH HỌA BGD NGÀY 22-3-2024 KỲ THI TỐT NGHI...
30 ĐỀ PHÁT TRIỂN THEO CẤU TRÚC ĐỀ MINH HỌA BGD NGÀY 22-3-2024 KỲ THI TỐT NGHI...30 ĐỀ PHÁT TRIỂN THEO CẤU TRÚC ĐỀ MINH HỌA BGD NGÀY 22-3-2024 KỲ THI TỐT NGHI...
30 ĐỀ PHÁT TRIỂN THEO CẤU TRÚC ĐỀ MINH HỌA BGD NGÀY 22-3-2024 KỲ THI TỐT NGHI...
 

Recently uploaded

Smartbiz_He thong MES nganh may mac_2024june
Smartbiz_He thong MES nganh may mac_2024juneSmartbiz_He thong MES nganh may mac_2024june
Smartbiz_He thong MES nganh may mac_2024june
SmartBiz
 
THONG BAO nop ho so xet tuyen TS6 24-25.pdf
THONG BAO nop ho so xet tuyen TS6 24-25.pdfTHONG BAO nop ho so xet tuyen TS6 24-25.pdf
THONG BAO nop ho so xet tuyen TS6 24-25.pdf
QucHHunhnh
 
100 DẪN CHỨNG NGHỊ LUẬN XÃ HỘiI HAY.docx
100 DẪN CHỨNG NGHỊ LUẬN XÃ HỘiI HAY.docx100 DẪN CHỨNG NGHỊ LUẬN XÃ HỘiI HAY.docx
100 DẪN CHỨNG NGHỊ LUẬN XÃ HỘiI HAY.docx
khanhthy3000
 
Cau-Trắc-Nghiệm-TTHCM-Tham-Khảo-THI-CUỐI-KI.pdf
Cau-Trắc-Nghiệm-TTHCM-Tham-Khảo-THI-CUỐI-KI.pdfCau-Trắc-Nghiệm-TTHCM-Tham-Khảo-THI-CUỐI-KI.pdf
Cau-Trắc-Nghiệm-TTHCM-Tham-Khảo-THI-CUỐI-KI.pdf
HngMLTh
 
Halloween vocabulary for kids in primary school
Halloween vocabulary for kids in primary schoolHalloween vocabulary for kids in primary school
Halloween vocabulary for kids in primary school
AnhPhm265031
 
BÁO CÁO CUỐI KỲ PHÂN TÍCH THIẾT KẾ HƯỚNG ĐỐI TƯỢNG - NHÓM 7.docx
BÁO CÁO CUỐI KỲ PHÂN TÍCH THIẾT KẾ HƯỚNG ĐỐI TƯỢNG - NHÓM 7.docxBÁO CÁO CUỐI KỲ PHÂN TÍCH THIẾT KẾ HƯỚNG ĐỐI TƯỢNG - NHÓM 7.docx
BÁO CÁO CUỐI KỲ PHÂN TÍCH THIẾT KẾ HƯỚNG ĐỐI TƯỢNG - NHÓM 7.docx
HngL891608
 
insulin cho benh nhan nam vien co tang duong huyet
insulin cho benh nhan nam vien co tang duong huyetinsulin cho benh nhan nam vien co tang duong huyet
insulin cho benh nhan nam vien co tang duong huyet
lmhong80
 
FSSC 22000 version 6_Seminar_FINAL end.pptx
FSSC 22000 version 6_Seminar_FINAL end.pptxFSSC 22000 version 6_Seminar_FINAL end.pptx
FSSC 22000 version 6_Seminar_FINAL end.pptx
deviv80273
 
SLIDE BÀI GIẢNG MÔN THƯƠNG MẠI ĐIỆN TỬ.pdf
SLIDE BÀI GIẢNG MÔN THƯƠNG MẠI ĐIỆN TỬ.pdfSLIDE BÀI GIẢNG MÔN THƯƠNG MẠI ĐIỆN TỬ.pdf
SLIDE BÀI GIẢNG MÔN THƯƠNG MẠI ĐIỆN TỬ.pdf
UyenDang34
 
40 câu hỏi - đáp Bộ luật dân sự năm 2015 (1).doc
40 câu hỏi - đáp Bộ  luật dân sự năm  2015 (1).doc40 câu hỏi - đáp Bộ  luật dân sự năm  2015 (1).doc
40 câu hỏi - đáp Bộ luật dân sự năm 2015 (1).doc
NguynDimQunh33
 
LỊCH SỬ 12 - CHUYÊN ĐỀ 10 - TRẮC NGHIỆM.pptx
LỊCH SỬ 12 - CHUYÊN ĐỀ 10 - TRẮC NGHIỆM.pptxLỊCH SỬ 12 - CHUYÊN ĐỀ 10 - TRẮC NGHIỆM.pptx
LỊCH SỬ 12 - CHUYÊN ĐỀ 10 - TRẮC NGHIỆM.pptx
12D241NguynPhmMaiTra
 
Văn 7. Truyện ngụ ngôn Rùa và thỏ+ Viết PT nhân vật.docx
Văn 7. Truyện ngụ ngôn Rùa và thỏ+ Viết PT nhân vật.docxVăn 7. Truyện ngụ ngôn Rùa và thỏ+ Viết PT nhân vật.docx
Văn 7. Truyện ngụ ngôn Rùa và thỏ+ Viết PT nhân vật.docx
metamngoc123
 
trắc nhiệm ký sinh.docxddddddddddddddddd
trắc nhiệm ký sinh.docxdddddddddddddddddtrắc nhiệm ký sinh.docxddddddddddddddddd
trắc nhiệm ký sinh.docxddddddddddddddddd
my21xn0084
 
Biểu tượng trăng và bầu trời trong tác phẩm của Nguyễn Quang Thiều
Biểu tượng trăng và bầu trời trong tác phẩm của Nguyễn Quang ThiềuBiểu tượng trăng và bầu trời trong tác phẩm của Nguyễn Quang Thiều
Biểu tượng trăng và bầu trời trong tác phẩm của Nguyễn Quang Thiều
lamluanvan.net Viết thuê luận văn
 
Giải phẫu tim sau đại học- LÊ QUANG TUYỀN
Giải phẫu tim sau đại học- LÊ QUANG TUYỀNGiải phẫu tim sau đại học- LÊ QUANG TUYỀN
Giải phẫu tim sau đại học- LÊ QUANG TUYỀN
linh miu
 
PLĐC-chương 1 (1).ppt của trường ĐH Ngoại thương
PLĐC-chương 1 (1).ppt của trường  ĐH Ngoại thươngPLĐC-chương 1 (1).ppt của trường  ĐH Ngoại thương
PLĐC-chương 1 (1).ppt của trường ĐH Ngoại thương
hieutrinhvan27052005
 

Recently uploaded (16)

Smartbiz_He thong MES nganh may mac_2024june
Smartbiz_He thong MES nganh may mac_2024juneSmartbiz_He thong MES nganh may mac_2024june
Smartbiz_He thong MES nganh may mac_2024june
 
THONG BAO nop ho so xet tuyen TS6 24-25.pdf
THONG BAO nop ho so xet tuyen TS6 24-25.pdfTHONG BAO nop ho so xet tuyen TS6 24-25.pdf
THONG BAO nop ho so xet tuyen TS6 24-25.pdf
 
100 DẪN CHỨNG NGHỊ LUẬN XÃ HỘiI HAY.docx
100 DẪN CHỨNG NGHỊ LUẬN XÃ HỘiI HAY.docx100 DẪN CHỨNG NGHỊ LUẬN XÃ HỘiI HAY.docx
100 DẪN CHỨNG NGHỊ LUẬN XÃ HỘiI HAY.docx
 
Cau-Trắc-Nghiệm-TTHCM-Tham-Khảo-THI-CUỐI-KI.pdf
Cau-Trắc-Nghiệm-TTHCM-Tham-Khảo-THI-CUỐI-KI.pdfCau-Trắc-Nghiệm-TTHCM-Tham-Khảo-THI-CUỐI-KI.pdf
Cau-Trắc-Nghiệm-TTHCM-Tham-Khảo-THI-CUỐI-KI.pdf
 
Halloween vocabulary for kids in primary school
Halloween vocabulary for kids in primary schoolHalloween vocabulary for kids in primary school
Halloween vocabulary for kids in primary school
 
BÁO CÁO CUỐI KỲ PHÂN TÍCH THIẾT KẾ HƯỚNG ĐỐI TƯỢNG - NHÓM 7.docx
BÁO CÁO CUỐI KỲ PHÂN TÍCH THIẾT KẾ HƯỚNG ĐỐI TƯỢNG - NHÓM 7.docxBÁO CÁO CUỐI KỲ PHÂN TÍCH THIẾT KẾ HƯỚNG ĐỐI TƯỢNG - NHÓM 7.docx
BÁO CÁO CUỐI KỲ PHÂN TÍCH THIẾT KẾ HƯỚNG ĐỐI TƯỢNG - NHÓM 7.docx
 
insulin cho benh nhan nam vien co tang duong huyet
insulin cho benh nhan nam vien co tang duong huyetinsulin cho benh nhan nam vien co tang duong huyet
insulin cho benh nhan nam vien co tang duong huyet
 
FSSC 22000 version 6_Seminar_FINAL end.pptx
FSSC 22000 version 6_Seminar_FINAL end.pptxFSSC 22000 version 6_Seminar_FINAL end.pptx
FSSC 22000 version 6_Seminar_FINAL end.pptx
 
SLIDE BÀI GIẢNG MÔN THƯƠNG MẠI ĐIỆN TỬ.pdf
SLIDE BÀI GIẢNG MÔN THƯƠNG MẠI ĐIỆN TỬ.pdfSLIDE BÀI GIẢNG MÔN THƯƠNG MẠI ĐIỆN TỬ.pdf
SLIDE BÀI GIẢNG MÔN THƯƠNG MẠI ĐIỆN TỬ.pdf
 
40 câu hỏi - đáp Bộ luật dân sự năm 2015 (1).doc
40 câu hỏi - đáp Bộ  luật dân sự năm  2015 (1).doc40 câu hỏi - đáp Bộ  luật dân sự năm  2015 (1).doc
40 câu hỏi - đáp Bộ luật dân sự năm 2015 (1).doc
 
LỊCH SỬ 12 - CHUYÊN ĐỀ 10 - TRẮC NGHIỆM.pptx
LỊCH SỬ 12 - CHUYÊN ĐỀ 10 - TRẮC NGHIỆM.pptxLỊCH SỬ 12 - CHUYÊN ĐỀ 10 - TRẮC NGHIỆM.pptx
LỊCH SỬ 12 - CHUYÊN ĐỀ 10 - TRẮC NGHIỆM.pptx
 
Văn 7. Truyện ngụ ngôn Rùa và thỏ+ Viết PT nhân vật.docx
Văn 7. Truyện ngụ ngôn Rùa và thỏ+ Viết PT nhân vật.docxVăn 7. Truyện ngụ ngôn Rùa và thỏ+ Viết PT nhân vật.docx
Văn 7. Truyện ngụ ngôn Rùa và thỏ+ Viết PT nhân vật.docx
 
trắc nhiệm ký sinh.docxddddddddddddddddd
trắc nhiệm ký sinh.docxdddddddddddddddddtrắc nhiệm ký sinh.docxddddddddddddddddd
trắc nhiệm ký sinh.docxddddddddddddddddd
 
Biểu tượng trăng và bầu trời trong tác phẩm của Nguyễn Quang Thiều
Biểu tượng trăng và bầu trời trong tác phẩm của Nguyễn Quang ThiềuBiểu tượng trăng và bầu trời trong tác phẩm của Nguyễn Quang Thiều
Biểu tượng trăng và bầu trời trong tác phẩm của Nguyễn Quang Thiều
 
Giải phẫu tim sau đại học- LÊ QUANG TUYỀN
Giải phẫu tim sau đại học- LÊ QUANG TUYỀNGiải phẫu tim sau đại học- LÊ QUANG TUYỀN
Giải phẫu tim sau đại học- LÊ QUANG TUYỀN
 
PLĐC-chương 1 (1).ppt của trường ĐH Ngoại thương
PLĐC-chương 1 (1).ppt của trường  ĐH Ngoại thươngPLĐC-chương 1 (1).ppt của trường  ĐH Ngoại thương
PLĐC-chương 1 (1).ppt của trường ĐH Ngoại thương
 

TUYỂN TẬP ĐỀ THI TOÁN VÀO LỚP 10 CHUYÊN KHOA HỌC TỰ NHIÊN HÀ NỘI NĂM 2022 CÓ LỜI GIẢI CHI TIẾT.pdf

  • 1. Hỗ trợ trực tuyến Fb www.facebook.com/DayKemQuyNhon Mobi/Zalo 0905779594 Tài liệu chuẩn tham khảo Phát triển kênh bởi Ths Nguyễn Thanh Tú Đơn vị tài trợ / phát hành / chia sẻ học thuật : Nguyen Thanh Tu Group Đ Ề T H I T H Ử V À O L Ớ P 1 0 T H P T M Ô N T O Á N Ths Nguyễn Thanh Tú eBook Collection TUYỂN TẬP ĐỀ THI TOÁN VÀO LỚP 10 CHUYÊN KHOA HỌC TỰ NHIÊN HÀ NỘI NĂM 2022 CÓ LỜI GIẢI CHI TIẾT WORD VERSION | 2023 EDITION ORDER NOW / CHUYỂN GIAO QUA EMAIL TAILIEUCHUANTHAMKHAO@GMAIL.COM vectorstock.com/28062405 Website: Liên hệ tài liệu word môn toán: TÀI LIỆU TOÁN HỌC TUYỂN TẬP ĐỀ THI TOÁN VÀO LỚP 10 CHUYÊN KHOA HỌC TỰ NHIÊN HÀ NỘI LỜI NÓI ĐẦU Nhằm đáp ứng nhu cầu về của giáo viên toán THCS và học sinh luyện thi vào lớp 10 môn toán, website giới thiệu đến thầy cô và các em bộ đề thi vào lớp 10 chuyên Đại học khoa học tự nhiên Hà Nội. Đây là bộ đề thi mang tính chất thực tiễn cao, giúp các thầy cô và các em học sinh luyện thi vào lớp 10 có một tài liệu bám sát đề thi để đạt được thành tích cao, mang lại vinh dự cho bản thân, gia đình và nhà trường. Bộ đề gồm nhiều Câu toán hay được các thầy cô trên cả nước sưu tầm và sáng tác, ôn luyện qua sẽ giúp các em phát triển tư duy môn toán từ đó thêm yêu thích và học giỏi môn học này, tạo được nền tảng để có những kiến thức nền tốt đáp ứng cho việc tiếp nhận kiến thức ở các lớp, cấp học trên được nhẹ nhàng và hiệu quả hơn. Các vị phụ huynh và các thầy cô dạy toán có thể dùng có thể dùng tuyển tập đề toán này để giúp con em mình học tập. Hy vọng Tuyển tập đề thi toán vào lớp 10 chuyên Đại học khoa học tự nhiên này sẽ có thể giúp ích nhiều cho học sinh phát huy nội lực giải toán nói riêng và học toán nói chung. Mặc dù đã có sự đầu tư lớn về thời gian, trí tuệ song không thể tránh khỏi những hạn chế, sai sót. Mong được sự góp ý của các thầy, cô giáo và các em học! Chúc các thầy, cô giáo và các em học sinh thu được kết quả cao nhất từ bộ đề này!
  • 2. Website: Liên hệ tài liệu word môn toán: TÀI LIỆU TOÁN HỌC MỤC LỤC PHẦN 1: ĐỀ THI Đề số Đề thi Trang 1. Đề thi vào lớp 10 chuyên Đại học KHTN Hà Nội năm 2022 (vòng 1) 2. Đề thi vào lớp 10 chuyên Đại học KHTN Hà Nội năm 2022 (vòng 2) 3. Đề thi vào lớp 10 chuyên Đại học KHTN Hà Nội năm 2021 (vòng 1) 4. Đề thi vào lớp 10 chuyên Đại học KHTN Hà Nội năm 2021 (vòng 2) 5. Đề thi vào lớp 10 chuyên Đại học KHTN Hà Nội năm 2020 (vòng 1) 6. Đề thi vào lớp 10 chuyên Đại học KHTN Hà Nội năm 2020 (vòng 2) 7. Đề thi vào lớp 10 chuyên Đại học KHTN Hà Nội năm 2019 (vòng 1) 8. Đề thi vào lớp 10 chuyên Đại học KHTN Hà Nội năm 2019 (vòng 2) 9. Đề thi vào lớp 10 chuyên Đại học KHTN Hà Nội năm 2018 (vòng 1) 10. Đề thi vào lớp 10 chuyên Đại học KHTN Hà Nội năm 2018 (vòng 2) 11. Đề thi vào lớp 10 chuyên Đại học KHTN Hà Nội năm 2017 (vòng 1) 12. Đề thi vào lớp 10 chuyên Đại học KHTN Hà Nội năm 2017 (vòng 2) 13. Đề thi vào lớp 10 chuyên Đại học KHTN Hà Nội năm 2016 (vòng 1) 14. Đề thi vào lớp 10 chuyên Đại học KHTN Hà Nội năm 2016 (vòng 2) 15. Đề thi vào lớp 10 chuyên Đại học KHTN Hà Nội năm 2015 (vòng 1) 16. Đề thi vào lớp 10 chuyên Đại học KHTN Hà Nội năm 2015 (vòng 2) 17. Đề thi vào lớp 10 chuyên Đại học KHTN Hà Nội năm 2014 (vòng 1) 18. Đề thi vào lớp 10 chuyên Đại học KHTN Hà Nội năm 2014 (vòng 2) 19. Đề thi vào lớp 10 chuyên Đại học KHTN Hà Nội năm 2013 (vòng 1) 20. Đề thi vào lớp 10 chuyên Đại học KHTN Hà Nội năm 2013 (vòng 2) 21. Đề thi vào lớp 10 chuyên Đại học KHTN Hà Nội năm 2012 (vòng 1) 22. Đề thi vào lớp 10 chuyên Đại học KHTN Hà Nội năm 2012 (vòng 2) 23. Đề thi vào lớp 10 chuyên Đại học KHTN Hà Nội năm 2011 (vòng 1) 24. Đề thi vào lớp 10 chuyên Đại học KHTN Hà Nội năm 2011 (vòng 2) 25. Đề thi vào lớp 10 chuyên Đại học KHTN Hà Nội năm 2010 (vòng 1) 26. Đề thi vào lớp 10 chuyên Đại học KHTN Hà Nội năm 2010 (vòng 2) 27. Đề thi vào lớp 10 chuyên Đại học KHTN Hà Nội năm 2009 (vòng 1) 28. Đề thi vào lớp 10 chuyên Đại học KHTN Hà Nội năm 2009 (vòng 2) PHẦN 2: HƯỚNG DẪN GIẢI Website: Liên hệ tài liệu word toán zalo: TÀI LIỆU TOÁN HỌC 1 LỜI GIẢI ĐỀ THI TUYỂN SINH VÀO LỚP 10 NĂM HỌC: 2023 – 2024 Trường THPT Chuyên Khoa học tự nhiên Môn thi : Toán (vòng 1) Thời gian làm bài: 120 phút (không kể thời gian phát đề) Câu I. (3,5 điểm) 1. Giải phương trình 2 2 6 2023 3 5 2025 5. + + + + = + + + x x x x x 2. Giải hệ phương trình ( )( ) 3 3 2 2 6 3 2 12 2 6 15 19 6 12.  + + =  + + + + + =  x y x y x y x y y x x y Câu II. (2,5 điểm) 1. Giả sử n là số nguyên sao cho 3 3 1011 − n chia hết cho 1008 . Chứng minh rằng 1 − n chia hết cho 48 . 2. Với , , a b c là các số dương thỏa mãn điều kiện 1 + + = ab bc ca . Chứng minh rằng 2 2 2 1 1 1 1 1 1 4 1 1 1     + + + >     + + +     a b c Câu III. (3 điểm) Cho hai đường tròn ( ) O và ( ) ′ O cố định cắt nhau tại A và B sao cho O nằm ngoài ( ) ′ O và ′ O nằm ngoài ( ) O . Trên đường tròn ( ) O lấy điểm P di chuyển sao cho P nằm trong đường tròn ( ) ′ O . Đường thẳng AP cắt ( ) ′ O tại C khác A . 1. Chứng minh rằng hai tam giác OBP và ′ O BC đồng dạng. 2. Gọi Q là giao điểm của hai đường thẳng OP và ′ O C . Chứng minh rằng   90 + =  QBC ABP . 3. Lấy điểm D thuộc ( ) O sao cho AD vuông góc ′ O C . Chứng minh rằng trung điểm của đoạn thẳng DQ luôn nằm trên một đường tròn cố định khi P thay đổi. Câu IV. (1 điểm) Giả sử A là tập hợp con của tập hợp gồm 30 số tự nhiên đầu tiên { } 0,1,2,3, ,29 … sao cho với k nguyên bất kỳ, , ∈ a b A bất kỳ (có thể = a b ) thì 30 + + a b k không là tích của hai số nguyên liên tiếp. Chứng minh rằng số phần tử của tập hợp A nhỏ hơn hoặc bằng 10.
  • 3. Website: Liên hệ tài liệu word toán zalo: TÀI LIỆU TOÁN HỌC 2 2. PHẦN LỜI GIẢI Câu 1: (3,5 diểm) 1. Giải phương trình 2 2 6 2023 3 5 2025 5. + + + + = + + + x x x x x 2. Giải hệ phương trình ( )( ) 3 3 2 2 6 3 2 12 2 6 15 19 6 12.  + + =  + + + + + =  x y x y x y x y y x x y Lời giải. 1. Điều kiện xác định: 3 ≥ − x . Cách 1. Xét 2 > x thì vì ( ) ( ) 2 2 6 2023 5 2025 2 0 + + − + + = − > x x x x x và ( ) 3 5 + − = x 2 0 − > x nên suy ra 2 2 6 2023 3 5 2025 5. + + + + ≥ + + + x x x x x Tương tự nếu 3 2 − ≤ < x thì 2 2 6 2023 3 5 2025 5. + + + + ≤ + + + x x x x x Do đó, ta phải có 2 = x và nghiệm này thoả mãn điều kiện xác định. Vậy 2 = x là nghiệm duy nhất của phương trình Cách 2. Phương trình đã cho có thể viết lại thành 2 2 6 2023 5 5 2025 3. + + − = + + − + x x x x x Giả sử hai vế cùng dấu, bình phương hai vế và rút gọn, ta được ( ) ( )( ) 2 2 5 6 2023 5 2025 3 . + + = + + + x x x x x Bình phương một lần nữa, khai triển và rút gọn, ta được ( )( ) 2 2 5 2020 0. − + + = x x x Vì 2 5 2020 0 + + = x x vô nghiệm nên 2 = x và nghiệm này thoả mãn. Vậy, phương trình đã cho có nghiệm duy nhất 2 = x . 2. Biến đổi phương trình thứ hai thành ( )( ) 2 2 6 2 3 1 12. + + + + = x y x xy y Kết hợp với phương trình thứ nhất ta suy ra ( )( ) ( )( ) 2 2 6 3 2 6 2 3 1 . + + = + + + + x y x y x y x xy y Mà ta thấy 6 0 + ≠ x y nên chia cả hai vế phương trình trên cho 6 + x y ta được 2 2 2 3 1 3 2 . + + + = + x xy y x y Biến đổi phương trình trên ta được Website: Liên hệ tài liệu word toán zalo: TÀI LIỆU TOÁN HỌC 3 ( )( ) ( ) ( ) ( )( ) 2 2 0 2 3 3 2 1 2 2 1 2 1 1 = + + − − + = + + − + − + + = + − + − x xy y x y x y x y x y x y x y x y . nên suy ra 2 1 0 + − = x y hoặc 1 0 + − = x y . • Nếu 2 1 0 + − = x y thì ta có 1 2 = − y x . Thay vào phương trình thứ nhất ta được Giải phương trình ta thu được các nghiệm ( ) ( ) 28 45 , 0,1 , , 11 11 −   =     x y . • Nếu 1 0 + − = x y thì ta có 1 = − y x . Thay voà phương trình thứ nhất ta được ( )( ) 6 6 3 2 2 12. + − + − = x x x x Giải phương trình ta thu được các nghiệm ( ) ( ) 4 9 , 0,1 , , 5 5 −   =     x y . Vậy, hệ đã cho có ba nghiệm ( ) ( ) 28 45 4 9 , 0,1 , , , , 11 11 5 5 − −     =         x y . Câu 2: (2,5 điểm) 1. Giả sửn là số nguyễn sao cho 3 3 1011 − n chia hết cho 1008. Chứng minh rằng 1 − n chia hết cho 48 . 2. Với , , a b c là các số dương thỏa mãn điều kiện 1 + + = ab bc ca . Chứng minh rằng 2 2 2 1 1 1 1 1 1 4. 1 1 1     + + + >     + + +     a b c Lời giải. 1. Ta có các biến đổi sau 3 3 1008 3 1011 3 3 1008 − = − − n n ∣ khi và chỉ khi 3 1008 3 3 − = n ∣ ( ) 3 3 1 − n tương đương với ( )( ) 3 2 3 336 1 1 1 1 − = − + + = − + − n n n n n n n ∣ Vì 336 chia hết cho 16 mà ( ) 2 1 1 1 + += + + n n n n lẻ nên 1 − n chia hết cho 16 . Ngoài ra vì 336 chia hết cho 3 mà ( )( ) 3 1 1 − = − + n n n n n tích ba số nguyên liên tiếp chia hết cho 3 nên 1 − n chia hết cho 3 . Vì ( ) 3,16 1 = nên 1 − n chia hết cho 3 16 48 ⋅ =. Phép chứng minh hoàn tất. 2. Ta cần chứng minh 2 2 2 1 1 1 1 1 1 4. 1 1 1     + + + >     + + +     a b c Ta biến đổi tương đương bất đẳng thức (1) như sau.
  • 4. Website: Liên hệ tài liệu word toán zalo: TÀI LIỆU TOÁN HỌC 4 ( ) ( )( )( ) ( )( )( ) ( ) ( ) ( ) ( ) ( ) ( ) 2 2 2 2 2 2 2 2 2 2 2 2 2 2 2 2 2 2 2 2 2 2 2 2 2 2 1 4 1 1 1 2 2 2 4 1 1 1 3( ) 2 4 3( ) 2 ( ) 2 4 3( ) 2 1 2 4 3( ) 4 2.     + + + ⇔ >     + + +     ⇔ + + + > + + + ⇔ + + + < ⇔ + + + − + + < ⇔ + − + + < ⇔ − + + < a b c a b c a b c a b c abc a b b c c a abc ab bc ac abc a b c abc abc a b c abc abc a b c Theo bất đẳng thức − AM GM và giả thiết ta có 3 2 2 2 1 3 = + + ≥ ab bc ac a b c kéo theo 2 1 3( ) 2 9 ≤ < abc . Suy ra bất đẳng thức ( ) 2 đúng và bài toán được chứng minh. Câu 3: (3 điểm) Cho hai đường tròn ( ) O và ( ) ′ O cắt nhau tại A và B sao cho O nằm ngoài ( ) ′ O và ′ O nằm ngoài ( ) O . Trên đường tròn ( ) O lấy điểm P di chuyển sao cho P nằm trong đường tròn ( ) ′ O . Đường thẳng AP cắt ( ) ′ O tại C khác A . 1. Chứng minh rằng hai tam giác OBP và ′ O BC dồng dạng. 2. Gọi Q là giao điểm của hai đường thẳng OP và ′ O C . Chứng minh rằng   90 + =  QBC ABP 3. Lấy điểm D thuộc ( ) O sao cho AD vuông góc với ′ O C . Chứng minh rằng trung điểm của đoạn thẳng DQ luôn nằm trên một đường tròn cố định khi P thay đổi. Lời giải. 1. Ta có     2 2 ′ = = = BOP BAP BAC BO C . Kết hợp với 1 = ′ ′ = OB O B OP O C , ta thu được ∼ OBP ′ O BC . Website: Liên hệ tài liệu word toán zalo: TÀI LIỆU TOÁN HỌC 5 2. Vì ∼ ′   OBP O BC nên ta được   ′ = OPB O CB , từ đó kéo theo tứ giác BCQP nội tiếp. Do đó,    = = QBC QPC OPA . Hơn nữa, vì OAP cân tại O nên ta được   1 90 2 + =  AOP OPA . Như vậy,     1 90 2 + = + =  QBC ABP OPA AOP 3. Gọi M là trung điểm của DQ . Đặt ′ ∩ = AD O C E . Vì các tứ giác BCQP và ADBP nội tiếp nên ta được     = = = BQC BPC ADB EDB, vì thế tứ giác BDEQ nội tiếp đường tròn đường kính DQ (với tâm đường tròn là M ). Do đó, ta được     1 2 = = = BMO DMB DEB AEB Mặt khác, ta cũng có     1 180 180 2 = − = − =   BOM BOD BAD BAE Như vậy, ta được ∼   OBM ABE (g.g). Gọi , , F X Y lần lượt là trung điểm của , , ′ AB OO OB. Khi đó, MY và EF là các đường trung tuyến tương ứng của OBM và ABE , vì thế ta được ∼   OMY AEF . Kết hợp với việc tứ giác ′ AEO F nội tiếp (vì   = ′ ′ = AEO AFO ) 90 ,  XY là đường trung bình của ′ OBO và sử dụng tính đối xứng, ta được       = ′ = = ′ = ′ = OMY AEF AO F AO O BO O OXY Như vậy, tứ giác OMXY nội tiếp. Mặt khác, vì, , , ′ O B O là các điểm cố định nên , X Y cũng là các điểm cố định. Do vậy, điểm M luôn chạy trên đường tròn ( ) OXY cố định. Câu 4: (1 diểm) Giả sử tập A là tập con của tập hợp gồm 30 số tự nhiên đầu tiên { } 0,1,2, ,29 … sao cho với k nguyên bất kỳ và , ∈ a b A bất kỳ (có thể = a b ) thì 30 + + a b k không là tích của hai số nguyên liên tiếp. Chứng minh rằng số phần tử của tập hợp A nhỏ hơn hoặc bằng 10. Lời giải. Ta trình bày hai lời giải Cách 1. Trước hết ta loại các số mà bản thân nó không thể xuất hiện trong A , bao gồm: • Các số có dạng ( ) 1 : 0,1,3,6,10,15,24,28 2 + n n ; • Các số có dạng ( ) 1 30 :16,18,25 2 + + n n ; • Các số có dạng ( ) 1 30 :13 2 + − n n .
  • 5. Website: Liên hệ tài liệu word toán zalo: TÀI LIỆU TOÁN HỌC 6 Đối với các số còn lại, ta ghép cặp chúng như sau: ( ) ( ) ( ) ( ) ( ) ( ) ( ) ( ) 2,4 , 5,7 , 8,12 , 9,11 , 19,23 , 20,22 , 24,26 , 27,29 ,14,17. (gồm 8 cặp số và 2 số lẻ không trong cặp nào). Ta thấy các số không cùng một cặp không thể cùng thuộc A , cho nên A chỉ chứa đúng một số trong mỗi cặp. Do đó A chỉ có tối đa 10 phần tử. Cách 2. Với hai số nguyên liên tiếp , 1 + a a , ta có ( ) ( ) 1 0,2,6,12,20,26 mod30 + ≡ a a . Với ∈ a A, xét = b a và 0 = k ta có 2a không đồng dư với ( ) 0,2,6,12,20,26 mod30 nên a không đồng dư với ( ) 0,1,3,6,10,13,15,16,18,21,25,28 mod30 . Suy ra { } 2,4,5,7,8,9,11,12,14,17,19,20,22,23,24,26,27,29 ⊂ = A B và nếu phân hoạch B thành 10 tập { } { } { } { } { } { } { } { } { } { } 2,4 , 5,7 , 8,12 , 11,9 , 14,22 , 17,19 , 20 , 23,27 , 24,26 , 29 thì mỗi tập con này chứa nhiều nhất một phần tử của A . Do đó, A chứa tối đa 10 phần tử. Thực ra ta có thể chứng minh được số phần tử của A nhiều nhất bằng 10 , chỉ cần chọn { } 2,5,8,11,14,17,20,23,26,29 . = A Website: Liên hệ tài liệu word toán zalo: TÀI LIỆU TOÁN HỌC 1 LỜI GIẢI ĐỀ THI TUYỂN SINH VÀO LỚP 10 NĂM HỌC: 2023 – 2024 Trường THPT Chuyên Khoa học tự nhiên Môn thi : Toán (vòng 2) Câu 1 : (3,5 điểm): 1) Giải phương trình 2 2 2 1 2 4 6 4 5 + + + = − x x x x x 2) Giải hệ phương trình ( ) 3 3 30 30 120  + =   + = + + +   xy x y x y x y Câu 2 (2.5 điểm): 1) Tìm tất cả các cặp số nguyên dương (x;y) thỏa mãn ( )( ) ( ) 4 1 3 1 7 2 3 7 2 + + + = + + x y y x y y 2) Với x,y,z là những số thực dương, tìm giá trị nhỏ nhất của biểu thức: 14 6 14 6 14 6 2 2 2 2 2 2 3 3 3 − + − + − + = + + + + + + + + x x y y z z M x y zx zy y z xy xz z x yz yx Câu 3 (3 điểm). Cho tam giác ABC nhọn với AB < AC nội tiếp trong đường tròn (O) có tiếp tuyến tại A của (O) cắt BC ở T sao cho TB > BC Gọi P và E lần lượt là trung điểm của TA và TC. 1) Chứng minh rằng tứ giác APEB nội tiếp. 2) Gọi giao điểm thứ hai của AE với (O) là F. Láy G thuộc (O) sao cho FG song song với AC. Chửng minh rằng  . = ATG TAF 3) Gọi H là trực tâm của tam giác ABC,D là giao điểm của AH và BC. M là trung điểm BC. K đối xứng với A qua BC. N thuộc đường thẳng AM sao cho KN song song với HM. Lấy S thuộc BC sao cho NS ⊥ NK . Dựng R thuộc tia AK sao cho AR.AH = 2 . AD Q là điểm sao cho PQ ⊥ AS và SQ ⊥ AO . Chứng minh rằng điểm đối xứng của A qua QR thuộc đường tròn đường kinh DN. Câu 4 (1 điểm). Viết 100 số nguyên dương đầu tiên 1,2,...,100 vào một bảng ô vuông kích thước 10 10 × một cách tuỳ ý sao cho mỗi ô vuông được viết đúng một số. Chứng minh rằng tồn tại hai ô kề nhau (hai ô có cạnh chung) mà hai số được viết ở hai ô này có hiệu lớn hơn hoặc bằng 10.
  • 6. Website: Liên hệ tài liệu word toán zalo: TÀI LIỆU TOÁN HỌC 2 PHẦN LỜI GIẢI Câu 1: (3,0 điểm): 1) Điều kiện: 0 5 ≤ < x . Ta biến đổi phương trình thành 2 4 6 4 6 4 4 5 5 + + + + = + − + − x x x x x x x x Sử dụng hằng đẳng thức, ta thu được 2 2 ( 4 6) (2 5 ) + + = + − x x x x Suy ra 4 6 2 5 + + = + − x x x x (do từng vế đều không âm), hay 4 6 5 + = + − x x x Bình phương hai vế của phương trình này ta có ( ) 4 6 5 2 5 + = + − + − x x x x x Hay ( ) 4 1 2 5 + = − x x x . Tiếp tục hay Tiếp tục ta bình phương hai vế với điều kiện 4 1 0 + ≥ x (đã thoả mãn được ) ( ) 2 16 8 1 4 5 + + = − x x x x Giải phương trình trên ta thu được 1 1     2 10 = = x và x (đều thoả mãn điều kiện). Vậy, phương trình đã cho có đúng hai nghiệm 1 1 ,   2 10 = = x x 2) Đặt , =+ = S x y P xy . Ta có ( ) 3 3 3 3 ( ) 3 3 + = + − + = − x y x y xy x y S SP Khi đó hệ phương trình trở thành 3 3 30 3 30 120 =    − = + +   SP S SP S Thay SP = 30 vào phương trình thứ hai ta có 3 3 120 120 = + + S S hay ( ) 3 3 120 120 + = + + + S S S S Ta nhận thấy Nếu 3 120 > + S S thì 3 120 > + S S , suy ra ( ) 3 3 120 120 + > + + + S S S S loại. Nếu 3 120 < + S S thì 3 120 < + S S suy ra ( ) 3 3 120 120 + < + + + S S S S loại. Website: Liên hệ tài liệu word toán zalo: TÀI LIỆU TOÁN HỌC 3 Như vậy ta có 3 3 120,   120   = + − − S S hay S S Giải phương trình ta thu được S = 5 khi đó 30 6 = = P S . Vậy ta có 5 6 + =   =  x y xy Theo Vi-ét đảo thì x,y là hai nghiệm của phương trình 2 5 6 0 − + = x x Giải phương trình ta được ( ) ( ) ( ) , 2,3 , 3,2 . = x y Vậy hệ đã cho có hai nghiệm (x,y) là ( ) 2,3 và ( ) 3,2 Câu 2 : (3,0 điểm): 1) Cách 1. Ta có các biến đổi phương trình sau ( )( ) ( ) 4 1 3 1 7 2 3 7 2 + + + = + + x y y x y y ( ) 2 2 1 2 3 7 21 2 3 7 2 ⇔ − + + + + = + + x y y y x y y ( )( ) 2 1 3 7 1 2 21 ⇔ − + + − = x y y k y (1) Ta chứng minh UCLN ( ) 2 1;3 7 1 2 1 − + + − = x x y x .Thật vậy, nếu UCLN( 2 1;3 7 1 2 − + + − x x y x ) > 1 thì gọi p là ước nguyên tố chung của 2 1,3 7 1 2 − + + − x x y x . Suy ra p |3 7 + y y . chú ý là 3 7 + y y đều không chia hết cho 3, 7 nên 3,7. ≠ p Lại có p| 21y nên p ∈{3, 7} mâu thuẫn. Vậy UCLN( ) 2 1;3 7 1 2 1 − + + − = x x y x Ta xét hai trường hợp sau • Nếu x là số chẵn thì 2 1 − x chia hết cho 3 và 3 7 1 2 + + − x y x chia 3 dư 1. Khi đó, từ phương trình (1) ta có 2 1 3 3 7 1 2 7  − =  + + − =  x y x y x y Suy ra 2 3 1 = + x y ,Chi ý là 3 1,3 ≡ y (mod 8) nên 3 1 + y không chia hết cho 8. Từ đó 2   1 = = x và y . . Vậy (x, y) = (2, 1) • Nếu x là số lẻ thì 2 1 − x chia 3 dư 1 và 3 7 1 2 + + − x y x chia hết cho 3. Khi đó, từ phương trình (1) ta có 2 1 7 3 7 1 2 3  − =  + + − =  x y x y x y Suy ra 2 7 1 = + x y . Về phải chia 7 dư 1 nên về trái chia 7 dư 1. Từ đó 3 , = ∈ x k k N * và thay vào phương trình được
  • 7. Website: Liên hệ tài liệu word toán zalo: TÀI LIỆU TOÁN HỌC 4 ( )( ) 2 2 1 2 2 1 7 − + + = k k k y Vì UCLN ( ) { } 2 2 1;2 2 1 1,3 − + + ∈ k k k nên UCLN( ) 2 2 1;2 2 1 1. − + + = k k k Vì 2 2 2 1 1 + + > k k nên 2 1 1 − = k suy ra 1 = k và 7 7 = y nên 1 = y và 3 3 = = x k . Vậy ( ) ( ) , 3,1 . = x y Vậy tất cả các cặp số (x, y) thỏa mãn là (2, 1), (3, 1) . Cách 2. Phương trình đã cho có thể viết lại thành ( )( ) 2 7 1 2 3 1 0 − − − − = x y x y Tới đây giải giống hai trường hợp ở trên. 2) Ta có ( ) ( ) 14 6 14 6 6 6 6 3 3 3 4 3 5 7 3 5 4 5 − + = + − + ≥ − + = + x x x x x x x theo bất đẳng thức AM-GM. Lại có cũng theo bất đẳng thức AM-GM, thì ( ) ( ) ( ) ( ) 6 6 6 6 6 4 4 4 2 4 5 1 1 3 3 1 3 2 + = + + + + + + ≥ + + ≥ + x x x x x x x x x Suy ra 4 2 2 2 2 2 2 ≥ ∑ + ∑ + + + + x x M x y xz yz x y xz yz và áp dụng bất đẳng thức cộng mẫu cho về trái, ta có ( ) ( ) ( ) ( ) ( ) 2 2 2 2 2 2 2 2 2 2 2 2 2 2 2 2 2 2 2 2 2 2 2 2( ) 3 6 3 2 2 + + + + + + + + + + ≥ ≥ = + + + + + + + + + + x y z x y z x y y z z x xy yz zx M x y y z z x xy yz zx x y y z z x xy yz zx Dấu bằng xảy ra khi và chỉ khi x = y = z = 1 Giá trị nhỏ nhất của M là 3. Câu 3 : (3,0 điểm) G F E P T O A B C Website: Liên hệ tài liệu word toán zalo: TÀI LIỆU TOÁN HỌC 5 1) Vì AT là tiếp tuyến của nên ta được 2 . = TA TBTC . Như vậy, ta được 2 1 1 . . . 2 2 = = = TPTA TA TBTC TBTE và vì tứ giác APEB là tứ giác nội tiếp. 2) Vì EP là đường trung bình của ∆ TAC , ∆ FGC là hình thang cân và AT là tiếp tuyến của (O) nên ta thu được      = = = = AEP EAC FAC GCA TAG và    . = = = GAC FCA TAF PAE Như vậy, ta được ∆ AEP ~∆ ACG (g-g) và dẫn đến = AE AP AC AG . Lại chú ý 2 = AT AP và 2 = AC EP , ta thu được 2 2 = = = AE AE AP AT EP AC AG AG . Kết hợp với   = AEP TAG ta thu được   ~   ∆ ∆ AEP TAG (C.G.C) và vì thế  . = ATG TAF 3) ta xét bổ đề sau: ∆ABC, đường thẳng qua B vuông AC cắt AC,(ABC) tại F,D. E thuộc (ABC) thoả DE//AC. Đường thẳng qua E vuông EF cắt BF tại G, đg thẳng qua B vuông AG cắt đường thẳng qua C vuông AD tại H, L trung điểm AH. CMR AEL = 90 Giải: BH giao EG tại J khi đó J thuộc (BFE). Mặt khác gọi K trung điểm BC thì F,K,J thẳng do BFK = ABE = BEJ = BFJ. Mà do FK vuông AD vuông CH nên J là trung điểm BH => JL // AB nên L thuộc EG => đpcm Quay lại bài toán: Dựng hbh DKNG, DS' là đg kính của (ADG) khi đó KS'D = AS'D = AGD = DNK => S trùng S'. NG giao BC tại F, NE vuông AD tại E. Khi đó A,F,E,G cùng thuộc 1 đg tròn ( EF = DN = AG ) • Đường thẳng qua E vuông AF cắt đg thẳng qua T vuông AS tại J. Khi đó theo bổ đề, trung điểm Q' của AJ thuộc SG ( để ý T thuộc (AFGE) ). Mà Q'P vuông AS => Q' trùng Q. Hơn nữa biến đổi tỉ số cho ta R là trung điểm AE nên QR // JE vuông AF kết hợp thêm nếu cho JE cắt AF tại A' thì A' thuộc (EF) hay A' thuộc (DN) nên ta có đpcm. Câu 4: (1,0 điểm): Lời giải. Cách 1. Ta giải bài toán tổng quát: Điền các số 1.2.... 2 n với n > 1 vào các ô vuông của bảng cỡn nn Khi đó tồn tại hai ô vuông kề nhau (chung cạnh) chứa hai số x, y mà |x - y| ≥ n Kí hiệu , k k m M tương ứng là số nhỏ nhất và số lớn nhất của hàng thứ k với k = 1, 2 ,...,n. Chú ý là 1 2 , , m m ..., 1 2 , , , ., …… n n m M M M đôi một phân biệt. Đặt m= max ( 1 2 , , m m ..., n m ) và M= min ( 1 2 , , ., …… n M M M ) . Xét hai trường hợp • Nếu m < M thì ta có ≤ ≤ k k m m M với mọi k = 1, 2 ,...,n. Điều này suy ra với hàng k bất kỷ thì tồn tại hai số ≤ < k k a m b và với mỗi hàng k ta chọn cặp ( , k k a b ) thuộc hai ô kề nhau ở hàng k. Vì 1 2 , , .. … n b b b lớn hơn m và các số 1 2 , , .. … n b b b là đôi một phân biệt nên tồn tại k { } 1,2, ., ∈ … n . sao cho ≥ + k b n m và do đó ( ) . − ≥ + − = k k b a m n m n
  • 8. Website: Liên hệ tài liệu word toán zalo: TÀI LIỆU TOÁN HỌC 6 • Nếu m > M thì gọi i, j { ∈ 1, 2 ,...,n} là các chỉ số sao cho i m > M = j M Khi đó tại mỗi cột luôn tồn tại các số không vượt quá M (ví dụ như số hàng j) và các số lớn hơn M (ví dụ như số ở hàng i). Khi đó với cột k bất kỳ tồn tại , k k a b sao cho ≤ < k k a M b và , k k a b thuộc hai ô kề nhau của cột k. Tương tự như trường hợp đầu tiên thì ta cũng có tồn tại k { ∈ 1, 2 ,...,n}mà ≥ + k b M n . Suy ra ( ) . − ≥ + − = k k b a M n m n Trong mọi tình huống ta đều có điều phải chứng minh. Cách 2. Giả sử phản chứng, tồn tại cách điền để không tồn tại x,y như vậy. Ta lần lượt điền các số bắt đầu từ 1 vào bảng ô vuông, xét thời điểm đầu tiên mà cả n cột đều đã được điền số. Ta xét hai trường hợp sau: • Nếu tồn tại cột nào đó đã được phủ hoàn toàn, ta sẽ đổi vai trò của hàng và cột (xét thời điểm đầu tiên cả n hàng đều có số). Nếu khi đó lại tồn tại một hàng được phủ hoàn toàn, thì tức là hai thời điểm đang xét trùng nhau. Điều này là không thể vì ô vừa được thêm phải là ô cuối cùng của hàng và cột đó, xét thời điểm ngay trước đó cho ta tất cả các hàng đều đã được điền. • Nếu không tồn tại cột đã phủ hoàn toàn, do mỗi cột đều có ô đã điền, ta có thể chọn ra ở cột thứ i được cặp ô , i i i A B mà A được điền còn i B thì chưa. Cần có 1 − ≤ − i i B A n kéo theo ≤ i B max ( 1 2 , , .., … n A A A ) + n -1,∀i Mà các số từ 1 đến max( ) i A đều đã được điền nên ( ) max > i i B A , vô lý do n số i B phân biệt. Các trường hợp cho ta giả sử sai và ta có điều phải chứng minh. Website: Liên hệ tài liệu word môn toán: TÀI LIỆU TOÁN HỌC Câu I. (4 điểm) 1) Giải hệ phương trình ( ) 3 2 3 2 6 5 5 42 5 6 30 42 xy x y x x x y x y  + + + =   + + + =   2) Giải phương trình : ( ) ( )( ) ( ) 3 3 3 6 3 2 3 6 3 24 x x x x + + − + + − = Câu II. (2 điểm) 1) Tìm tất cả các cặp số nguyên ( ) , x y thỏa mãn đẳng thức : ( ) 2022 2 2 25 354 60 36 305 5 6 y x x y y x + + = + + − 2) Trên bàn có 8 hộp rỗng (trong các hộp không có viên bi nào). Người ta thực hiện các lần thêm bi vào các hộp theo quy tắc sau : mỗi lần ta chọn ra 4 hộp bất kỳ và bỏ vào hộp 1 viên, một hộp 2 viên, hai hộp còn lại mỗi hộp 3 viên. Hỏi số lần thêm bi ít nhất có thể để nhận được số bi ở 8 hộp trên là số tự nhiên liên tiếp ? Câu III. (3 điểm) Cho hình chữ nhật ( ) ABCD AB AD < nội tiếp đường tròn ( ) O . Trên cạnh AD lấy hai điểm E và F ( , E F không trùng với , ) A D sao cho E nằm giữa A và F, đồng thời 1 2 ABE DCF BOC ∠ + ∠ = ∠ 1) Chứng minh rằng BE và CF cắt nhau tại một điểm nằm trên đường tròn ( ) O 2) Đường thẳng qua O song song với BC cắt , BE CF theo thứ tự tại , . M N Chứng minh rằng 1 180 2 DAM ADN AOD ∠ + ∠ + ∠ = ° 3) Dựng hình chữ nhật MNPQ sao cho NQ song song với , BD đồng thời MP song song với . AC Chứng minh rằng đường tròn ngoại tiếp hình chữ nhật MNPQ tiếp xúc với đường tròn ( ) O Câu IV. (1 điểm) Cho , , a b c là những số thực dương. Chứng minh rằng : 2 6 2 4 3 32 3 2 a a b a c a b c a a b a c b c b c a b c + + + + + + + ≥ + + + + + + SỞ GIÁO DỤC VÀ ĐÀO TẠO HÀ NỘI ĐỀ CHÍNH THỨC Đề thi gồm 01 trang Đề số 1 KỲ THI VÀO LỚP 10 CHUYÊN KHOA HỌC TỰ NHIÊN NĂM HỌC 2022 – 2023 Môn: TOÁN (VÒNG 1) Thời gian làm bài: 120 phút (Không kể thời gian giao đề)
  • 9. Website: Liên hệ tài liệu word môn toán: TÀI LIỆU TOÁN HỌC Câu I. (3,5 điểm) 1) Với , , a b c là những số thực dương thỏa mãn điều kiện 1 1 1 1. a b c + + =Chứng minh rằng : ( )( )( ) 1 1 1 1 2 abc a bc b ca c ab a bc b ca c ab   + + =   + + + + + +   2) Giải hệ phương trình 2 2 2 3 6 3 2 1 2 2 6 x xy y x y x y  + + =   + + = + +   Câu II. (2,5 điểm) 1) Tìm tất cả các cặp số nguyên dương ( ) ; x y thỏa mãn đẳng thức : ( )( ) ( ) 3 3 3 2 3 4 5 5 x y x y xy x y x y xy + + + = + + + 2) Với , , a b c là những số thực dương thỏa mãn các điều kiện sau ( )( ) 2 2 2 2 3 3 3; 2 10; 2 2 14 1 1 4 2 2 2 2 c b a b a b a c a b ab a b a b  ≤ < ≤ + ≤ + + ≤   + + + ≤ + + +   . Tìm giá trị lớn nhất của biểu thức: 4 4 2 2 4 2 4 P a b b c = + + + Câu III. (3 điểm) Cho tam giác ABC nhọn, không cân, nội tiếp đường tròn (O). Điểm P nằm trong tam giác ABC . Gọi , E F lần lượt là hình chiếu vuông góc của P trên các cạnh , CA CB . Giả sử tứ giác BCEF nội tiếp trong đường tròn ( ) K 1) Chứng minh rằng AP vuông góc với BC 2) Chứng minh rằng 2 AP OK = 3) Đường thẳng qua P vuông góc với AP cắt đường tròn tại hai điểm , Q R . Chứng minh rằng đường tròn tâm A bán kính AP tiếp xúc với đường tròn ngoại tiếp KQR ∆ Câu IV. (1 điểm) Cho các điểm 1 2 30 , ,....., A A A theo thứ tự nằm trên một đường thẳng sao cho độ dài các đoạn 1 k k A A + bằng k (đơn vị dài), với 1,2,....,29 k = . Ta tô màu mỗi đoạn thẳng 1 2 29 30 ,....., A A A A bởi 1 trong 3 màu (mỗi đoạn được tô bởi đúng 1 màu). Chứng minh rằng với mọi cách tô màu,, ta luôn chọn được hai số nguyên dương 1 29 j i ≤ ≤ ≤ sao cho hai đoạn 1 i i A A+ và 1 j j A A + được tô cùng màu và i j − là bình phương của số nguyên dương. SỞ GIÁO DỤC VÀ ĐÀO TẠO HÀ NỘI ĐỀ CHÍNH THỨC Đề thi gồm 01 trang Đề số 2 KỲ THI VÀO LỚP 10 CHUYÊN KHOA HỌC TỰ NHIÊN NĂM HỌC 2022 – 2023 Môn: TOÁN (VÒNG 2) Thời gian làm bài: 120 phút (Không kể thời gian giao đề) Website: Liên hệ tài liệu word môn toán: TÀI LIỆU TOÁN HỌC Bài 1. Giải phương trình ( )( ) 13 5 18 8 61 3 5 8 x x x x x − + + = + + − + . Bài 2. Giải hệ phương trình: ( ) ( ) ( ) 4 4 2 2 4 6 1 1 2 x y x y x x y x y  + + =   + =−   . Bài 3. Tìm số nguyên dương n nhỏ nhất, biết rằng khi chia n cho 7, 9, 11, 13 được các số dư tương ứng là 3, 4, 5, 6 . Bài 4. Cho tam giác nhọn ABC có điểm P nằm trong tam giác ( P không nằm trên các cạnh). Gọi , , J K L lần lượt là tâm đường tròn nội tiếp các tam giác PBC , PCA , PAB . 1. Chứng minh rằng    450 BJC CKA ALB + + = °. 2. Giả sử PB PC = và PC PA < . Gọi , , X Y Z lần lượt là hình chiếu vuông góc của , , J K L trên các cạnh , , BC CA AB . Dựng hình bình hành XYWZ . Chứng minh W nằm trên phân giác  BAC . Bài 5. Cho tập { } 1; 2; 3;...;2021 A = . Tìm số nguyên dương k lớn nhất ( ) 2 k > sao cho ta có thể chọn được k số phân biệt từ tập A mà tổng của hai số phân biệt bất kỳ trong k số được chọn không chia hết cho hiệu của chúng.  HẾT  SỞ GIÁO DỤC VÀ ĐÀO TẠO HÀ NỘI ĐỀ CHÍNH THỨC Đề thi gồm 01 trang Đề số 3 KỲ THI VÀO LỚP 10 CHUYÊN KHOA HỌC TỰ NHIÊN NĂM HỌC 2021 – 2022 Môn: TOÁN (VÒNG 1) Thời gian làm bài: 120 phút (Không kể thời gian giao đề)
  • 10. Website: Liên hệ tài liệu word môn toán: TÀI LIỆU TOÁN HỌC Câu 1. 1) Với , , a b c là các số thực thỏa mãn 0 a b c + + ≠ và ( )( )( ) 1 a b b c c a + + + = . Chứng minh rằng 2 2 2 1 ( ) ( ) 1 ( ) 1 ( ) a b abc ab a b c a a b c abc b a b c abc a b c + + + + + = ⋅ + + + + + + + + + + 2) Giải hệ phương trình: ( ) 2 2 2 2 4 4 2 11 3 2 31 9 18 13 x y xy x y xy x y x y xy  + + + =   + + = + +   . Câu 2. 1) Tìm , x y nguyên dương thỏa mãn 3 29 2 . x y + = 2) Với , , a b c là các số thực dương thỏa mãn điều kiện ( ) 2 9 a b c ab bc ca + + + + + = . Tìm giá trị lớn nhất của biểu thức 2 2 2 1 1 1 10 21 10 21 10 21 a b c M a a b b c c + + + = + + ⋅ + + + + + + Câu 3. Cho hình thoi ABCD có  BAD nhọn có đường tròn nội tiếp ( ) O . Các điểm , M N lần lượt thuộc các cạnh , CB CD sao cho MN tiếp xúc ( ) O tại P và tam giác CMN không cân. MN lần lượt cắt , AB AD tại , E F . Gọi , K L lần lượt là trực tâm các , BME DNF ∆ ∆ . 1) Chứng minh OP đi qua trung điểm I của KL . 2) Gọi H là trực tâm tam giác CMN . Chứng minh 1 2 2 OI EF CH MN − = − . 3) Gọi , EK FL lần lượt cắt BD tại , S T . NS cắt MT tại Q . Đường tròn nội tiếp tam giác CMN tiếp xúc MN với tại G . Chứng minh PQ song song với . GH Câu 4. Giả sử 1 2 2021 , ,....., a a a là những số thực thỏa mãn 2021 1 2 2 2 2 1 2 2021 ...... 0 1 1 1 a a a a a a + + + = + + + . Chứng minh rằng tồn tại số nguyên ( ) 1 2021 k k ≤ ≤ sao cho 1 2 2 2 2 1 2 2 2 1 ...... 1 1 1 8 k k ka a a k a a a + + + + ≤ ⋅ + + + ……………………….HẾT……………………… SỞ GIÁO DỤC VÀ ĐÀO TẠO HÀ NỘI ĐỀ CHÍNH THỨC Đề thi gồm 01 trang Đề số 4 KỲ THI VÀO LỚP 10 CHUYÊN KHOA HỌC TỰ NHIÊN NĂM HỌC 2021 – 2022 Môn: TOÁN (VÒNG 2) Thời gian làm bài: 120 phút (Không kể thời gian giao đề) Website: Liên hệ tài liệu word môn toán: TÀI LIỆU TOÁN HỌC Câu I. (4 điểm) 1) Giải hệ phương trình : ( ) 2 2 3 2 7 9 70 x y xy x xy x y  + + =   = + −   2) Giải phương trình: ( )( ) 11 5 8 2 1 24 3 5 2 1 x x x x − + − = + − − Câu II. (2 điểm) 1) Tìm , x y nguyên dương thỏa mãn 2 2 2 2 16 99 9 36 13 26 x y xy x y x y − + = + + + 2) Với , a b là những số thực dương thỏa mãn 2 2 2 2 3 5 ;8 12 2 3 5 10 a b a b a b ab ≤ + ≤ + ≤ + + + Chứng minh rằng: 2 2 3 8 10 21 a b ab + + ≤ Câu III. (3 điểm) Cho tam giác ABC có  BAC là góc nhỏ nhất trong ba góc của tam giác và nội tiếp đường tròn (O). Điểm D thuộc cạnh BC sao cho AD là phân giác . BAC Lấy các điểm , M N thuộc (O) sao cho đường thẳng , CM BN cùng song song với đường thẳng AD 1) Chứng minh rằng AM AN = 2) Gọi giao điểm của đường thẳng MN với các đường thẳng , AC AB lần lượt là , . E F Chứng minh rằng bốn điểm , , , B C E F cùng thuộc một đường tròn 3) Gọi , P Q theo thứ tự là trung điểm của các đoạn thẳng , . AM AN Chứng minh rằng các đường thẳng , , EQ FP ADđồng quy. Câu IV. (1 điểm) Với , , a b clà những số thực dương thỏa mãn 3. a b c + + =Chứng minh rằng: ( ) ( ) ( ) ( ) ( ) ( ) 2 2 2 2 2 2 4 2 2 2 a a bc b b ca c c ab b ab c c bc a a ca b + + + + + ≥ + + + SỞ GIÁO DỤC VÀ ĐÀO TẠO HÀ NỘI ĐỀ CHÍNH THỨC Đề thi gồm 01 trang Đề số 5 KỲ THI VÀO LỚP 10 CHUYÊN KHOA HỌC TỰ NHIÊN NĂM HỌC 2020 – 2021 Môn: TOÁN (VÒNG 1) Thời gian làm bài: 120 phút (Không kể thời gian giao đề)
  • 11. Website: Liên hệ tài liệu word môn toán: TÀI LIỆU TOÁN HỌC Câu I. (4 điểm) 1) Giải hệ phương trình: ( )( ) 2 3 3 ( )( 1) 4 5 12 13 243 x y x y xy x y x y y + + =    + + + + + + + =   2) Giải phương trình: 7 7 7 ( 12) (2 12) (24 3 ) 0 x x x − + − + − = Câu II. (2 điểm) 1) Tìm tất cả các số nguyên dương a, b, c sao cho cả ba số 2 2 2 4 5 ;4 5 ;4 5 a b b c c a + + + đều là bình phương của số nguyên dương. 2) Từ một bộ bốn số thực ( , , , ) a b c d ta xây dựng bộ số mới ( , , , ) a b b c c d d a + + + + và liên tiếp xây dựng các bộ số mới theo quy tắc trên. Chứng minh rằng nếu ở hai thời điểm khác nhau, ta thu được cùng một bộ số (có thể khác thứ tự) thì bộ số ban đầu phải có dạng ( , , , ) a a a a − − Câu III. (3 điểm) Cho tam giác ABC cân tại A với 90o BAC ∠ < . Gọi P là giao điểm của BE với trung trực của BC. Gọi K là hình chiếu vuông góc của P lên AB. Gọi Q là hình chiếu vuông góc của E lên AP. Gọi giao điểm của EQ và PK là F . 1) Chứng minh rằng bốn điểm A, E, P, F cùng thuộc một đường tròn. 2) Gọi giao điểm của KQ và PE là L. Chứng minh LA vuông góc LE. 3) Gọi giao điểm của FL và AB là S . Gọi giao điểm của KE và AL là T . Lấy R là điểm đối xứng với A qua L . Chứng minh rằng đường tròn ngoại tiếp tam giác AST và đường tròn ngoại tiếp tam giác BPR tiếp xúc với nhau. Câu IV. (1 điểm) Với a, b, c là những số thực dương thỏa mãn 3 a b c + + =. Chứng minh rằng 2 1 1 1 4 3 1 1 3 a b c a b c abc bc ca ab     + + − + ≥ + + +         SỞ GIÁO DỤC VÀ ĐÀO TẠO HÀ NỘI ĐỀ CHÍNH THỨC Đề thi gồm 01 trang Đề số 6 KỲ THI VÀO LỚP 10 CHUYÊN KHOA HỌC TỰ NHIÊN NĂM HỌC 2020 – 2021 Môn: TOÁN (VÒNG 1) Thời gian làm bài: 120 phút (Không kể thời gian giao đề) Website: Liên hệ tài liệu word môn toán: TÀI LIỆU TOÁN HỌC Bài 1. a, Giải phương trình: 2 2 26 5 2 26 5 3 30 30 x x x x + + += + + b, Giải hệ phương trình: 2 2 2 2 ( 2 )(2 3 4 ) 27 x y x y y xy  + =   + + + =   Bài 2. a, Tìm tất cả các cặp số nguyên thỏa mãn: 2 2 ( 1)( ) 3 1 x x y xy x − + + = − b, Với x,y là các số thực thay đổi thỏa mãn 1≤ y ≤ 2 và xy + 2 ≥ 2y, tìm giá trị nhỏ nhất của biểu thức 2 2 4 1 x M y + = + Bài 3. Cho hình vuông ABCD, đường tròn (O) nội tiếp hình vuông tiếp xúc với các cạnh AB, AD tại hai điểm E,F. Gọi G là giao điểm các đường thẳng CE và BF. a, Chứng minh rằng năm điểm A,F,O,G,E cùng nằm trên một đường tròn b, Gọi giao điểm của đường thẳng FB và đường tròn là M(M ≠ F). CMR M là trung điểm của đoạn thẳng BG. c, CMR trực tâm của tam giác GAF nằm trên đường tròn (O) Bài 4. Cho x, y, z là các số thực dương thỏa mãn xy + yz + xz = 1. Chứng minh rằng: 3 2 2 2 2 2 2 1 1 1 2 3 1 1 1 1 1 1     + + ≥ + +   + + + + + +   x y z x y z x y z SỞ GIÁO DỤC VÀ ĐÀO TẠO HÀ NỘI ĐỀ CHÍNH THỨC Đề thi gồm 01 trang Đề số 7 KỲ THI VÀO LỚP 10 CHUYÊN KHOA HỌC TỰ NHIÊN NĂM HỌC 2019 – 2020 Môn: TOÁN (VÒNG 1) Thời gian làm bài: 150 phút (Không kể thời gian giao đề)
  • 12. Website: Liên hệ tài liệu word môn toán: TÀI LIỆU TOÁN HỌC Bài 1. a. Giải hệ phương trình: ( )( ) 2 2 2 3 4 8 . 2 8 x y xy x y x xy  + + =   + + + =   b. Giải phương trình: ( ) 2 2 27 27 2 2 5 2 2 5 x x x x x x + + + = + − + − + Bài 2. a. Chứng minh rằng với mọi số nguyên dương n , ta luôn có ( ) ( ) ( ) 7 7 7 7 7 7 27 5 10 10 27 5 5 10 27 n n n       + + + + + + + +       chia hết cho 42 . b. Với , x y là các số thực dương thay đổi thỏa mãn điều kiện 2 2 4 4 17 5 5 1 x y xy x y + + + + ≥ . Tìm giá trị nhỏ nhất của biểu thức: 2 2 17 17 16 P x y xy = + + . Bài 3. Cho tam giác ABC cân tại A, có đường tròn nội tiếp ( ) I . Các điểm , E F theo thứ tự thuộc các cạnh , CA AB ( E khác C và A; F khác B và A) sao cho EF tiếp xúc với đường tròn ( ) I tại điểm P . Gọi , K L lần lượt là hình chiếu vuông góc của , E F trên BC . Giả sử FK cắt EL tại điểm J . Gọi H là hình chiếu vuông góc của J trên BC . a) Chứng minh rằng HJ là phân giác của góc EHF . b) Kí hiệu 1 2 , S S lần lượt là diện tích của các tứ giác BFJL và CEJK . Chứng minh rằng 2 1 2 2 S BF S CE = . c) Gọi D là trung điểm của cạnh BC . Chứng minh rằng ba điểm , , P J D thẳng hàng. Bài 4. Cho M là tập tất cả 4039 số nguyên liên tiếp từ 2019 − đến 2019 . Chứng minh rằng trong 2021 số đôi một phân biệt được chọn bất kì từ M luôn tồn tại ba số phân biệt có tổng bằng 0 . SỞ GIÁO DỤC VÀ ĐÀO TẠO HÀ NỘI ĐỀ CHÍNH THỨC Đề thi gồm 01 trang Đề số 8 KỲ THI VÀO LỚP 10 CHUYÊN KHOA HỌC TỰ NHIÊN NĂM HỌC 2019 – 2020 Môn: TOÁN (VÒNG 2) Thời gian làm bài: 150 phút (Không kể thời gian giao đề) Website: Liên hệ tài liệu word môn toán: TÀI LIỆU TOÁN HỌC Bài 1. a) Giải phương trình 2 3 x x 2 x 1 2 x 1 − + + = + . b) Giải hệ phương trình 2 2 2 xy y 1 y x 2y 2xy 4 x  + = +   + + =+   . Bài 2. a) Tìm tất cả các cặp số nguyên ( ) x; y thỏa mãn ( )( ) 2 x y 3x 2y 2x y 1 + + = + − . b) Với a, b là các số thực dương thay đổi thỏa mãn b a 2b 2 3 + = + . Tìm giá trị nhỏ nhất của biểu thức a b M a 2b b 2a = + + + . Bài 3. Cho tam giác ABC có đường tròn nội tiếp ( ) I tiếp xúc với các cạnh BC, CA, AB lần lượt tại các điểm D, E, F. Gọi K là hình chiếu vuông góc của B trên đường thẳng DE và M là trung điểm của đoạn thẳng DF. a) Chứng minh rằng hai tam giác BKM và DEF đồng dạng với nhau. b) Gọi L là hình chiếu của vuông góc của C trên đường thẳng DF và N là trung điểm của đoạn thẳng DE. Chứng minh rằng hai đường thẳng MK và NL song song với nhau. c) Gọi J, X lần lượt là trung điểm của các đoạn thẳng KL và ID. Chứng minh rằng đường thẳng JX vuông góc với đường thẳng EF. Bài 4. Trên mặt phẳng cho hai điểm P và Q phân biệt. Xét 10 đường thẳng nằm trong mặt phẳng trên thỏa mãn các tính chất sau: i) Không có hai đường thẳng nào song song hoặc trùng nhau. ii) Mỗi đường thẳng đi qua P hoặc Q, không có đường thẳng nào đi qua cả P và Q. Hỏi 10 đường thẳng trên có thể chia mặt phẳng thành tối đa bao nhiêu miền? Hãy giải thích. SỞ GIÁO DỤC VÀ ĐÀO TẠO HÀ NỘI ĐỀ CHÍNH THỨC Đề thi gồm 01 trang Đề số 9 KỲ THI VÀO LỚP 10 CHUYÊN KHOA HỌC TỰ NHIÊN NĂM HỌC 2018 – 2019 Môn: TOÁN (VÒNG 1) Thời gian làm bài: 120 phút (Không kể thời gian giao đề)
  • 13. Website: Liên hệ tài liệu word môn toán: TÀI LIỆU TOÁN HỌC Câu 1. a) Giải hệ phương trình : ( )( ) 3 3 3 3 ( ) 2 7 1 1 31 xy x y x y x y x y + =    + + + + + =   b) Giải phương trình: ( ) 9 3 3 2 7 5 3 2 x x x x + − = + − Câu 2. a) Cho , x y là các số nguyên sao cho 2 2 2 2 ; 2 x xy y xy y x − − − − đều chia hết cho 5. Chứng minh 2 2 2 2 x y x y + + + cũng chia hết cho 5 b) Cho 1 2 50 , ,......, a a a là các số nguyên thỏa mãn: 1 2 50 1 ...... 50 a a a ≤ ≤ ≤ ≤ , 1 2 50 ..... 100 a a a + + + = . Chứng minh rằng từ các số đã cho có thể chọn được một vài số có tổng là 50 Câu 3. Cho ngũ giác lồi ABCDE nội tiếp (O) có / / CD BE . Hai đường chéo CE và BD cắt nhau tại P. Điểm M thuộc BE sao cho   MAB PAE = . Điểm K thuộc AC sao cho MK song song AD, điểm L thuộc đường thẳng AD sao cho ML // AC. Đường tròn ngoại tiếp tam giác KBC cắt BD, CE tại Q và S (Q khác B, S khác C) a) Chứng minh 3 điểm K, M, Q thẳng hàng b) Đường tròn ngoại tiếp tam giác LDE cắt BD, CE tai T và R (T khác D, R khác E). Chứng minh M, S, Q, R,T cùng thuộc một đường tròn c) Chứng minh đường tròn ngoại tiếp tam giác PQR tiếp xúc (O) Câu 4. Cho , , a b c là các số thực dương. Chứng minh rằng 1 1 2 ab bc a b b c a b b c    + + ≤      + + + +     SỞ GIÁO DỤC VÀ ĐÀO TẠO HÀ NỘI ĐỀ CHÍNH THỨC Đề thi gồm 01 trang Đề số 10 KỲ THI VÀO LỚP 10 CHUYÊN KHOA HỌC TỰ NHIÊN NĂM HỌC 2018 – 2019 Môn: TOÁN (VÒNG 2) Thời gian làm bài: 150 phút (Không kể thời gian giao đề) Website: Liên hệ tài liệu word môn toán: TÀI LIỆU TOÁN HỌC Câu 1. (3.5 điểm) a) Giải hệ phương trình 2 2 2 3 1 2 x y xy x x y y               b) Giải phương trình      2 2 1 1 1 1 2 1 x x x x x         Câu 2 (2.5 điểm) a) Chứng minh rằng không tồn tại các số nguyên x, y thỏa mãn đẳng thức 2 2 12 26 15 4617 x xy y    b) Với a, b là các số thực dương. Tìm giá trị lớn nhát của biểu thức   2 2 1 1 1 M a b ab a b b a                   Câu 3 (3.0 điểm) Cho hình thoi ABCD có  0 90 BAD  . Đường tròn tâm I nội tiếp tam giác ABD tiếp xúc với BD và BA lần lượt tại J và L. Trên đường thẳng IJ lấy điểm K sao cho BK song song ID. a) Chứng minh rằng   CBK ABI  . b) Chứng minh rằng KC KB  . c) Chứng minh rằng bốn điểm C, K, I ,L cùng nằm trên một đường tròn. Câu 4. (1.0 điểm) Tìm tập hợp số nguyên dương n sao cho tồn tại một cách sắp xếp các số 1;2;3;...;n thành 1 2 3 ; ; ;...; n a a a a mà khi chia các số 1 1 2 1 2 3 1 2 ; ; ;...; ...an a a a a a a a a cho n ta được các số dư đôi một khác nhau. SỞ GIÁO DỤC VÀ ĐÀO TẠO HÀ NỘI ĐỀ CHÍNH THỨC Đề thi gồm 01 trang Đề số 11 KỲ THI VÀO LỚP 10 CHUYÊN KHOA HỌC TỰ NHIÊN NĂM HỌC 2017 – 2018 Môn: TOÁN (VÒNG 1) Thời gian làm bài: 120 phút (Không kể thời gian giao đề)
  • 14. Website: Liên hệ tài liệu word môn toán: TÀI LIỆU TOÁN HỌC Câu 1 (3.5 điểm). 1. Giải hệ phương trình 2 2 3 3 x y x y x y xy                2. Với a, b là các số thực dương thỏa mãn 1 ab a b    . Chứng minh rằng:    2 2 2 2 1 1 1 2 1 1 a b ab a b a b        Câu 2 (2.5 điểm). 1. Giả sử p và q là các số nguyên tố thỏa mãn đẳng thức     2 1 1 p p q q    . a) Chứng minh rằng tồn tại số nguyên dương k sao cho 2 1 , 1 p kq q kp     . b) Tìm tất cả các số nguyên tố p, q thỏa mãn đẳng thức     2 1 1 p p q q    . 2. Với a, b, c là các số thực dương thỏa mãn 2 ab bc ca abc     . Tìm giá trị lớn nhất của biểu thức 2 2 2 1 1 1 2 2 2 2 2 2 a b c M a a b b c c             Câu 3 (3.0 điểm). Cho tam giác ABC nhọn với AB AC  . Gọi E, F lần lượt là trung điểm của CA, AB. Đường trung trực của EF cắt BC tại D. Giả sử P nằm trong góc  EAF và nằm ngoài tam giác AEF sao cho   EF PEC D  và   PEB DFE  . Đường thẳng PA cắt đường tròn ngoại tiếp tam giác PEF tại Q khác P. a) Chứng minh rằng    EQF BAC EDF   . b) Tiếp tuyến tại P của đường tròn ngoại tiếp tam giác PEF cắt CA, AB lần lượt tại M, N. Chứng minh rằng bốn điểm C, M, B, N cùng nằ trên một đường tròn. Gọi đường tròn này là đường tròn   K . c) Chứng minh rằng đường tròn   K tiếp xúc với đường tròn ngại tiếp tam giác AEF. Câu 4 (1.0 điểm). Cho n là số nguyên dương với 5 n  . Xét đa giác lồi n cạnh. Người ta muốn kẻ một số đường chéo của đa giác mà các đường chéo này chia đa giác thành đúng k miền, mõi miền là mọt ngũ giác lồi (hai miền bất kì không có điểm chung trong). a) Chứng minh rằng ta có thể thực hiện được với 2018, 672 n k   b) Với 2017, 672 n k   ta có thể thực hiện được không? Hãy giải thích. SỞ GIÁO DỤC VÀ ĐÀO TẠO HÀ NỘI ĐỀ CHÍNH THỨC Đề thi gồm 01 trang Đề số 12 KỲ THI VÀO LỚP 10 CHUYÊN KHOA HỌC TỰ NHIÊN NĂM HỌC 2017 – 2018 Môn: TOÁN (VÒNG 2) Thời gian làm bài: 150 phút (Không kể thời gian giao đề) Website: Liên hệ tài liệu word môn toán: TÀI LIỆU TOÁN HỌC Câu 1 (3.5 điểm). a) Giải hệ phương trình      3 3 2 2 x + y + xy x + y = 4 xy + 1 x + y = 4          b) Giải phương trình 8 3 7 2 5 5 x x x      Câu 2 (2.5 điểm) a)Tìm tất cả các giá tri của m sao cho tồn tại cặp số nguyên   ; x y thỏa mãn hệ phương trình :   2 2 2 2 3 2 6 mxy m m x y m               b) Với x, y là những số thực thỏa mãn các điều kiện 0 2;2 2 x y x y xy      . Tìm giá trị lớn nhất của biểu thức     2 2 2 2 P x x 1 y y 1     Câu 3 (3.0 điểm). Cho tam giác ABC nhọn nội tiếp đường tròn   O với AB AC  . Phân giác của góc  BAC cắt BC tại D và cắt đường tròn   O tại E khác A. M là trung điểm của đoạn thẳng AD. Đường thẳng BM cắt đường tròn   O tại P khác B. Giả sử các đường thẳng EP và AC cắt nhau tại N. a) chứng minh rằng tứ giác APNM nội tiếp và N là trung điểm của đoạn thẳng AC. b) Giả sử đường tròn   K ngoại tiếp tam giác EMN cắt đường thẳng AC tại Q khác N. Chứng minh rằng B và Q đối xứng nhau qua AE. c) Giả sử đường tròn   K cắt đường thẳng BM tại M. Chứng minh rằng RA vuông góc RC. Câu 4 (1.0 điểm). Số nguyên a được gọi là số “đẹp” nếu với mọi cách sắp xếp theo thứ tự tùy ý của 100 số 1, 2, 3,…, 100 luôn tồn tại 10 số hạng liên tiếp có tổng lớn hơn hoặc bằng a. Tìm số “đẹp” lớn nhất SỞ GIÁO DỤC VÀ ĐÀO TẠO HÀ NỘI ĐỀ CHÍNH THỨC Đề thi gồm 01 trang Đề số 13 KỲ THI VÀO LỚP 10 CHUYÊN KHOA HỌC TỰ NHIÊN NĂM HỌC 2016 – 2017 Môn: TOÁN (VÒNG 1) Thời gian làm bài: 120 phút (Không kể thời gian giao đề)
  • 15. Website: Liên hệ tài liệu word môn toán: TÀI LIỆU TOÁN HỌC Câu 1(3.5 điểm). a) Giải hệ phương trình 2 2 2 2 4 5 4 8 5 10 1 x y x xy x y                b) giải phương trình 3 2 2 64 4 5 6 5 5 6 6 x x x x x x       Câu 2 (2.5 điểm). a) Với x, y là những số nguyên thỏa mãn đẳng thức 2 2 1 1 2 3 x y    . Chứng minh 2 2 40 x y   . b) Tìm tất cả các cặp số nguyên   ; x y thỏa mãn đẳng thức sau 4 2 3 2 x x y   . Câu 3 (3.0 điểm) Cho hình vuông ABCD nội tiếp đường tròn   O . P là điểm thuộc cung nhỏ AD của đường tròn   O và P khác A, D. Các đường thẳng PB, PC lần lượt cắt AD tại AD tại M, N. Đường trung trực của AM cắt đường thẳng AC, PB lần lượt tại E, K. Đường trung trực DN cắt các đường thẳng BD, PC lần lượt tại F, L. a) Chứng minh rằng ba điểm K, O, L thẳng hàng. b) Chứng minh đường thẳng PO đi qua trung điểm của EF c) Giả sử đường thảng EK cắt đường thẳng FL và AC cắt nhau tại T. Đường thẳng ST cắt các đường thẳng PB, PC lần lượt tại U và V. Chứng minh rằng bốn điểm K, L, V, U cùng thuộc một đương tròn. Câu 4(1.0 điểm) Chứng minh rằng với mọi số tự nhiên n 3  luôn tồn tại cách xếp bộ n số 1,2,3,...,n thành bộ số 1 2 3 , , ,..., n x x x x sao cho 2 i k j x x x   với mọi bộ chỉ số   ; ; i j k mà 1 i j k n     . SỞ GIÁO DỤC VÀ ĐÀO TẠO HÀ NỘI ĐỀ CHÍNH THỨC Đề thi gồm 01 trang Đề số 14 KỲ THI VÀO LỚP 10 CHUYÊN KHOA HỌC TỰ NHIÊN NĂM HỌC 2016 – 2017 Môn: TOÁN (VÒNG 2) Thời gian làm bài: 150 phút (Không kể thời gian giao đề) Website: Liên hệ tài liệu word môn toán: TÀI LIỆU TOÁN HỌC Câu 1. (3,0 điểm). 1). Giả sử là hai số thực phân biệt thỏa mãn . a). Chứng minh rằng . b). Chứng minh rằng . 2). Giải hệ phương trình . Câu 2. (3,0 điểm). 1). Tìm các số nguyên không nhỏ hơn 2 sao cho chia hết cho . 2). Với là những số thực thỏa mãn đẳng thức Tìm giá trị lớn nhất và nhỏ nhất của biểu thức . Câu 3. (3,0 điểm). Cho tam giác nhọn không cân có tâm đường tròn nội tiếp là điểm . Đường thẳng cắt tại . Gọi lần lượt là các điểm đối xứng của qua . 1). Chứng minh rằng song song với . 2). Gọi lần lượt là trung điểm các đoạn thẳng . Đường tròn ngoại tiếp tam giác cắt đường tròn ngoại tiếp tam giác tại khác . Chứng minh rằng bốn điểm cùng nằm trên một đường tròn. 3). Chứng minh rằng ba điểm thẳng hàng. Câu 4. (1,0 điểm). 1). Cho bảng ô vuông 2015 x2015. Kí hiệu ô là ô ở hang thứ , cột thứ . Ta viết các số nguyên dương từ 1 đến 2015 vào các ô của bảng theo quy tắc sau: i). Số 1 được viết vào ô (1,1) ii). Nếu số được viết vào ô ( ) thì số được viết vào ô . iii). Nếu số được viết vào ô thì số được viết vào ô (xem hình 1). Khi đó số 2015 được viết vào ô . Hãy xác định và 2). Giả sử là các số thực dương thỏa mãn Chứng minh rằng . ; a b 2 2 3 3 2 a a b b     3 a b   3 3 45 a b   2 2 2 2 3 5 4 5 x y xy x y xy              ; x y 1 xy    1 1 x y   ; x y 2 2 2 1 0. x y y    3 1 xy P y   ABC  I AI BC D ; E F D ; IC IB EF BC ; ; M N J ; ; DE DF EF AEM  AFN P A ; ; ; M N P J ; ; A J P ( ; ) i j i j k ( ; ) i j 1 i  1 k  ( 1; 1) i j   k (1; ) j 1 k  ( 1; 1) j    ; m n m . n ; ; a b c 4. ab bc ac abc       2 2 2 2 a b c a b c ab bc ac         SỞ GIÁO DỤC VÀ ĐÀO TẠO HÀ NỘI ĐỀ CHÍNH THỨC Đề thi gồm 01 trang Đề số 15 KỲ THI VÀO LỚP 10 CHUYÊN KHOA HỌC TỰ NHIÊN NĂM HỌC 2015 – 2016 Môn: TOÁN (VÒNG 1) Thời gian làm bài: 120 phút (Không kể thời gian giao đề)
  • 16. Website: Liên hệ tài liệu word môn toán: TÀI LIỆU TOÁN HỌC Câu 1. (3,0 điểm). 1) Với là các số thỏa mãn . Chứng minh rằng . 2) Giải hệ phương trình . Câu 2. (3,0 điểm). 1) Tìm số tự nhiên để và là số chính phương (số chính phương là bình phương của một số nguyên) 2) Tìm nguyên thỏa mãn đẳng thức . 3) Giả sử là các số thực lớn hơn 2. Tìm giá trị nhỏ nhất của biểu thức: . Câu 3. (3,0 điểm). Cho tam giác nhọn không cân với . Gọi là trung điểm của đoạn thẳng . Gọi là hình chiếu vuông góc của trên đoạn . Trên tia đối của tia lấy điểm sao cho . 1) Chứng minh rằng . 2) Gọi là điểm đối xứng với qua . Đường thẳng cắt tại .Chunwgs minh rằng bốn điểm cùng thuộc một đường tròn,gọi đường tròn này là . 3) Đường tròn ngoại tiếp tam giác cắt tại và . Chứng minh rằng song song với . Câu 4. (1,0 điểm). Ký hiệu là tập hợp gồm diểm phân biệt trên một mặt phẳng. Giả sử tất cả các điểm của không cùng nằm trên một đường thẳng. Chứng minh rằng có ít nhất đường thẳng phân biệt mà mỗi đường thẳng đi qua ít nhất hai điểm của . ; ; a b c         3 3 3 3 3 3 3 24 3 3 3 a b c a b c b c a c a b                 2 2 2 1 a b b c c a       3 3 2 2 2 5 27 7 26 27 9 x y xy x y y x x x                 n 5 n 30 n   ; x y 1 3 x y x y      ; ; x y z 4 4 4 y x z P y z z x x y          ABC  AB AC  M BC H B AM AM N 2 AN MH  BN AC  Q A N AC BQ D ; ; ; B D N C ( ) O AQD  ( ) O G D NG BC S 2015 S 2015 S SỞ GIÁO DỤC VÀ ĐÀO TẠO HÀ NỘI ĐỀ CHÍNH THỨC Đề thi gồm 01 trang Đề số 16 KỲ THI VÀO LỚP 10 CHUYÊN KHOA HỌC TỰ NHIÊN NĂM HỌC 2015 – 2016 Môn: TOÁN (VÒNG 2) Thời gian làm bài: 150 phút (Không kể thời gian giao đề) Website: Liên hệ tài liệu word môn toán: TÀI LIỆU TOÁN HỌC Câu 1. 1) Giải phương trình ( )( ) 2 1 1 2 2 1 8. + + − + − = x x x 2) Giải hệ phương trình 2 2 2 2 1 2 4  − + =   + + =   x xy y x xy y Câu 2. 1) Giả sử x, y, z là ba số dương thỏa mãn điều kiện x + y + z = xyz. Chứng minh rằng: ( ) ( )( )( ) 2 2 2 5 4 3 2 3 . 1 1 1 + + + + = + + + + + + xyz x y z x y z x y y z z x x y z 2) Tìm nghiệm nguyên của phương trình: ( ) 2 2 3 . + + + + + x y x y x y xy Câu 3. Cho tam giác ABC nhọn với AB < BC. Gọi D là điểm thuộc cạnh BC sao cho AD là phân giác của ∠BAC . Đường thẳng qua C song song với AD cắt trung trực của AB tại F. 1) Chứng minh tam giác ABF đồng dạng với tam giác ACE. 2) Chứng minh rằng các đường thẳng BE, CF, AD đồng quy tại một điểm, gọi điểm đó là G. 3) Đường thẳng qua G song song với AE cắt đường thẳng BF tại Q. Đường thẳng QE cắt đường tròn ngoại tiếp tam giác GEC tại P khác E. Chứng minh rằng các điểm A, P, G, Q, F cùng thuộc một đường tròn. Câu 4. Giả sử a, b, c là các số thực dương thỏa mãn đẳng thức ab + bc + ca = 1 . Chứng minh rằng ( ) 4 2 4 2 4 2 5 2 . 9 + + ≤ + + + abc a b c a b b c c a SỞ GIÁO DỤC VÀ ĐÀO TẠO HÀ NỘI ĐỀ CHÍNH THỨC Đề thi gồm 01 trang Đề số 17 KỲ THI VÀO LỚP 10 CHUYÊN KHOA HỌC TỰ NHIÊN NĂM HỌC 2014 – 2015 Môn: TOÁN (VÒNG 1) Thời gian làm bài: 120 phút (Không kể thời gian giao đề)
  • 17. Website: Liên hệ tài liệu word môn toán: TÀI LIỆU TOÁN HỌC Câu 1. 1. Giả sử x, y là những số thực dương thỏa mãn: 2 4 4 2 2 4 4 8 4 2 4 8 4 y y y y x y x y x y x y + + + = + + + − Chứng minh rằng: 5y = 4x 2. Giải hệ phương trình: 2 2 2 2 2 3 12 6 12 6 x y xy x x y y y x  − + =   + = + +   Câu 2. 1. Cho x, y là những số nguyên lớn hơn 1 sao cho 4x2 y2 – 7x + 7y là số chính phương. Chứng minh rằng: x = y. 2. Giả sử x, y là những số thực không âm thỏa mãn: x3 + y3 + xy = x2 + y2 Tìm giá trị lớn nhất và nhỏ nhất của biểu thức: 1 2 2 1 x x P y y + + = + + + Câu 3. Cho tam giác ABC nội tiếp đường tròn (O) và điểm P nằm trong tam giác thỏa mãn PB = PC. D là điểm thuộc cạnh BC (D khác B và D khác C) sao cho P nằm trong đường tròn ngoại tiếp tam giác DAC và đường tròn ngoại tiếp tam giác DAC. Đường thẳng PB cắt đường tròn ngoại tiếp tam giác DAB tại E khác B. Đường thẳng PC cắt đường tròn ngoại tiếp tam giác DAC tại F khác C. 1. Chứng minh rằng bốn điểm A, E, B, F cùng thuộc một đường tròn. 2. Giả sử đường thẳng AD cắt đường tròn (O) tại Q khác A, đường thẳng AF cắt đường thẳng QC tại L. Chứng minh rằng tam giác ABE đồng dạng với tam giác CLF. 3. Gọi K là giao điểm của đường thẳng AE và đường thẳng QB. Chứng minh rằng:     QKL PAB QLK PAC + = + . Câu 4. Cho tập hợp A gồm 31 phần tử và dãy gồm m tập hợp của A thỏa mãn đồng thời các điều kiện sau: i) Mỗi tập hợp thuộc dãy có ít nhất hai phần tử. ii) Nếu hai tập hợp thuộc dãy có chung nhau ít nhất hai phần tử thì số phần tử của hai tập hợp này khác nhau. Chứng minh rằng: m ≤ 900 SỞ GIÁO DỤC VÀ ĐÀO TẠO HÀ NỘI ĐỀ CHÍNH THỨC Đề thi gồm 01 trang Đề số 18 KỲ THI VÀO LỚP 10 CHUYÊN KHOA HỌC TỰ NHIÊN NĂM HỌC 2014 – 2015 Môn: TOÁN (VÒNG 2) Thời gian làm bài: 150 phút (Không kể thời gian giao đề) Website: Liên hệ tài liệu word môn toán: TÀI LIỆU TOÁN HỌC Câu 1. (3,0 điểm). 1). Giải phương trình . 2). Giải hệ phương trình . Câu 2. (3,0 điểm). 1). Cho các số thực thỏa mãn . Chứng minh rằng . 2). Có bao nhiêu số nguyên dương có chữ số sao cho chia hết cho ? Câu 3. (3,0 điểm). Cho tam giác nhọn nội tiếp đường tròn với Đường phân giác của góc cắt tại điểm khác Gọi là trung điểm của và là điểm đối xứng với qua tâm . Giả sử đường tròn ngoại tiếp tam giác cắt đoạn thẳng tại điểm khác 1). Chứng minh rằng tam giác và tam giác đồng dạng. 2). Chứng minh rằng vuông góc với Câu 4. (1,0 điểm). Giả sử là các số thực dương thỏa mãn điều kiện Tìm giá trị nhỏ nhất của biểu thức . 3 1 2 3 x x     1 1 9 2 1 3 1 1 4 2 x y x y x xy y xy                                    ; ; 0 a b c      8 a b b c c a abc              3 4 a b c ab bc ca a b b c c a a b b c b c c a c a a b                5 abcde   10 abc d e   101 ABC  ( ) O . AB AC   BAC ( ) O D . A M AD E D O ABM  AC F . A BDM  BCF  EF . AC ; ; ; a b c d 1. abc bcd cda dab       3 3 3 3 4 9 P a b c d     SỞ GIÁO DỤC VÀ ĐÀO TẠO HÀ NỘI ĐỀ CHÍNH THỨC Đề thi gồm 01 trang Đề số 19 KỲ THI VÀO LỚP 10 CHUYÊN KHOA HỌC TỰ NHIÊN NĂM HỌC 2013 – 2014 Môn: TOÁN (VÒNG 1) Thời gian làm bài: 120 phút (Không kể thời gian giao đề)
  • 18. Website: Liên hệ tài liệu word môn toán: TÀI LIỆU TOÁN HỌC Câu 1. (3,0 điểm). 1). Giải hệ phương trình . 2). Giải phương trình . Câu 2. (3,0 điểm). 1). Tìm các cặp số nguyên thỏa mãn . 2). Với là các số thực dương thỏa mãn tìm giá trị nhỏ nhất của biểu thức . Câu 3. (3,0 điểm). Cho tam giác nhọn nội tiếp đường tròn có trực tâm . Gọi là điểm nằm trên đường tròn ngoại tiếp tam giác ( khác và ) và nằm trong tam giác . cắt tại khác cắt tại khác . cắt tại cắt tại . Đường tròn ngoại tiếp tam giác và đường tròn ngoại tiếp tam giác cắt nhau tại khác 1). Chứng minh rằng ba điểm thẳng hàng. 2). Giả sử là phân giác góc Chứng minh rằng khi đó đi qua trung điểm của Câu 4. (1,0 điểm). Giả sử dãy số thực có thứ tự thỏa mãn các điều kiện và Chứng minh rằng 3 3 1 7 7 x y y x xy xy y x                2 3 1 3 1 1 x x x x          ; x y 2 2 5 8 20412 x y     ; x y 1, x y   2 2 1 1 1 P x y x y                ABC  ( ) O H P HBC  P , B C H ABC  PB ( ) O M , B PC ( ) O N C BM AC , E CN AB F AME  ANF  Q . A ; ; M N Q AP . MAN PQ . BC 1 2 192 x x x     1 2 192 0 x x x      1 2 192 2013. x x x      192 1 2013 . 96 x x   SỞ GIÁO DỤC VÀ ĐÀO TẠO HÀ NỘI ĐỀ CHÍNH THỨC Đề thi gồm 01 trang Đề số 20 KỲ THI VÀO LỚP 10 CHUYÊN KHOA HỌC TỰ NHIÊN NĂM HỌC 2013 – 2014 Môn: TOÁN (VÒNG 2) Thời gian làm bài: 150 phút (Không kể thời gian giao đề) Website: Liên hệ tài liệu word môn toán: TÀI LIỆU TOÁN HỌC Câu 1. (3,0 điểm). 1). Giải phương trình: . 2). Giải hệ phương trình . Câu 2. (3,0 điểm). 1). Tìm tất cả các cặp số nguyên thỏa mãn đẳng thức . 2). Giả sử là các số thực dương thỏa mãn điều kiện . Tìm giá trị nhỏ nhất của biểu thức . Câu 3. (3,0 điểm). Cho tam giác nhọn nội tiếp đường tròn tâm . Gọi là một điểm trên cung nhỏ ( khác và không đi qua ). Giả sử là một điểm thuộc đoạn thẳng sao cho đường tròn đường kính cắt cung nhỏ tại điểm khác . 1). Gọi là điểm đối xứng với điểm qua . Chứng minh rằng ba điểm thẳng hàng. 2). Đường tròn đường kính cắt tại điểm khác . Chứng minh rằng là tâm đường tròn nội tiếp tam giác . Câu 4. (1,0 điểm). Giả sử là các số thực dương thỏa mãn: ; ; . Tìm giá trị nhỏ nhất của biểu thức: .    9 2012 6 2012 9 6 x x x x        2 2 2 4 2 4 x y y x y xy                ; x y      1 5 2 x y xy x y x y          ; x y    1 1 4 x y    2 2 y x P y x   ABC  O M  BC M ; B C AM O P AM MP  BC N M D M O ; ; N P D MP MD Q M P AQN  ; ; a b c 3 a b c    1 c b   a b c         2 1 1 1 1 ab a b c ab a b c Q         SỞ GIÁO DỤC VÀ ĐÀO TẠO HÀ NỘI ĐỀ CHÍNH THỨC Đề thi gồm 01 trang Đề số 21 KỲ THI VÀO LỚP 10 CHUYÊN KHOA HỌC TỰ NHIÊN NĂM HỌC 2012 – 2013 Môn: TOÁN (VÒNG 1) Thời gian làm bài: 120 phút (Không kể thời gian giao đề)
  • 19. Website: Liên hệ tài liệu word môn toán: TÀI LIỆU TOÁN HỌC Câu 1. (3,0 điểm). 1). Giải hệ phương trình . 2). Giải phương trình . Câu 2. (3,0 điểm). 1). Tìm hai chữ số cuối cùng của số . 2). Tìm giá trị lớn nhất của hàm số , với . Câu 3. (3,0 điểm). Cho tam giác nhọn ( ) nội tiếp đường tròn . Giả sử là hai điểm thuộc cung nhỏ sao cho song song với và tia nằm giữa hai tia . Gọi là hình chiếu vuông góc của điểm trên và là hình chiếu vuông góc của điểm trên . 1). Giả sử cắt tại điểm . Chứng minh nằm trên đường tròn . 2). Gọi giao điểm của và là khác . Giả sử cắt tại . Chứng minh rằng bốn điểm cùng thuộc một đường tròn. Câu 4. (1,0 điểm). Với mỗi số nguyên lớn hơn hoặc bằng 2 cố định, xét các tập số thực đôi một khác nhau . Kí hiệu là số các giá trị khác nhau của tổng ( ). Tìm giá trị lớn nhất và giá trị nhỏ nhất của .     3 3 2 9 3 6 26 2 xy x y xy x y x y                   4 2 4 2 2 x x x      106 2012 41 57 A   2 3 2 1 5 4 y x x x     1 5 2 2 x   ABC  AB AC  ( ) O ; M N  BC MN BC AN ; AM AB P C AN Q M AB CP QM T T ( ) O NQ ( ) O R N AM PQ S ; ; ; A R Q S n n   1 2 ; ;...; n X x x x    C X i j x x  1 i j n      C X SỞ GIÁO DỤC VÀ ĐÀO TẠO HÀ NỘI ĐỀ CHÍNH THỨC Đề thi gồm 01 trang Đề số 22 KỲ THI VÀO LỚP 10 CHUYÊN KHOA HỌC TỰ NHIÊN NĂM HỌC 2012 – 2013 Môn: TOÁN (VÒNG 2) Thời gian làm bài: 150 phút (Không kể thời gian giao đề) Website: Liên hệ tài liệu word môn toán: TÀI LIỆU TOÁN HỌC Câu 1 (3,0 điểm). 1). Giải hệ phương trình . 2). Giải phương trình . Câu 2. (3,0 điểm). 1). Chứng minh rằng không tồn tại các bộ ba số nguyên thỏa mãn đẳng thức: . 2). Tìm tất cả các cặp số nguyên thỏa mãn đẳng thức . Câu 3. (3,0 điểm). Cho hình bình hành với . Đường phân giác của góc cắt đường tròn ngoại tiếp tam giác tại khác . Kẻ đường thẳng đi qua và vuông góc với . Đường thẳng lần lượt cắt các đường thẳng tại . 1). Chứng minh rằng . 2). Chứng minh rằng là tâm đường tròn ngoại tiếp tam giác . 3). Gọi giao điểm của và là , chứng minh rằng . Câu 4. (1,0 điểm). Với là những số thực dương, tìm giá trị nhỏ nhất của biểu thức .   2 2 1 3 ( 2) 1 x y x y y x y x                    2 3 7 2 1 x x x x       ; ; x y z 4 4 4 7 5 x y z      ; x y     4 4 3 1 1 x x y     ABCD  90 BAD    BCD BCD  O C d A CO d ; CB CD ; E F OBE ODC   O CEF  OC BD I . . . . IB BE EI ID DF FI  ; x y   3 3 3 3 3 3 4 8 y x x y y x y P      SỞ GIÁO DỤC VÀ ĐÀO TẠO HÀ NỘI ĐỀ CHÍNH THỨC Đề thi gồm 01 trang Đề số 23 KỲ THI VÀO LỚP 10 CHUYÊN KHOA HỌC TỰ NHIÊN NĂM HỌC 2011 – 2012 Môn: TOÁN (VÒNG 1) Thời gian làm bài: 120 phút (Không kể thời gian giao đề)
  • 20. Website: Liên hệ tài liệu word môn toán: TÀI LIỆU TOÁN HỌC Câu 1. (3,0 điểm). 1). Giải phương trình : . 2). Giải hệ phương trình: . Câu 2. (3,0 điểm). 1). Với mỗi số thực ta gọi phần nguyên của là số nguyên lớn nhất không vượt quá và ký hiệu là . Chứng minh rằng với mọi số nguyên dương , biểu thức không biểu diễn được dưới dạng lập phương của một số nguyên dương. 2). Với là các số thực dương thỏa mãn đẳng thức , Tìm giá trị nhỏ nhất của biểu thức : . Câu 3. (3,0 điểm). Cho hình thang với song song . Các góc và là các góc nhọn. Hai đường chéo và cắt nhau tại . là điểm bất kỳ trên đoạn thẳng ( không trùng với ). Giả sử đường tròn ngoại tiếp tam giác cắt đoạn thẳng tại khác và đường tròn ngoại tiếp tam giác cắt đoạn thẳng tại khác 1). Chứng minh rằng năm điểm cùng nằm trên một đường tròn. Gọi đường tròn này là . 2). Giả sử các đường thẳng và cắt nhau tại , chứng minh rằng cũng nằm trên đường tròn . 3). Trong trường hợp thẳng hàng, chứng minh rằng . Câu 4. (1,0 điểm). Giả sử là một tập con của tập các số tự nhiên . Tập có phần tử nhỏ nhất là 1, phần tử lớn nhất là 100 và mỗi thuộc ( ) luôn tồn tại cũng thuộc sao cho ( có thể bằng ). Hãy tìm một tập có số phần tử nhỏ nhất.    1 1 3 1 x x x         2 2 2 2 2 2 1 2 4 x y xy x y x y x y               a a a a     n 2 3 1 1 27 3 n n              ; ; x y z 5 xy yz zx          2 2 2 3 3 2 6 5 6 5 6 5 x y z P x y z         ABCD BC AD  BAD  CDA AC BD I P BC P ; B C BIP  PA M P CIP  PD N P ; ; ; ; A M I N D ( ) K BM CN Q Q ( ) K ; ; P I Q PB BD PC CA  A  A x A 1 x  ; a b A x a b   a b A SỞ GIÁO DỤC VÀ ĐÀO TẠO HÀ NỘI ĐỀ CHÍNH THỨC Đề thi gồm 01 trang Đề số 24 KỲ THI VÀO LỚP 10 CHUYÊN KHOA HỌC TỰ NHIÊN NĂM HỌC 2011 – 2012 Môn: TOÁN (VÒNG 2) Thời gian làm bài: 150 phút (Không kể thời gian giao đề) Website: Liên hệ tài liệu word môn toán: TÀI LIỆU TOÁN HỌC Câu I. 1) Giải hệ phương trình      = + = + + . 2 23 12 8 3 2 2 2 2 y x xy y x 2) Giải phương trình . 1 8 3 1 2 4 3 1 2 3 2 + + = + − + + x x x x Câu II. 1) Tìm tất cả các số nguyên không âm (x, y) thoả mãn đẳng thức ( )( ) ( )( ) . 25 1 2 4 1 1 2 2 = + + + + + + xy y x xy y x 2) Với mỗi số thực a, ta gọi phần nguyên của số a là số nguyên lớn nhất không vượt quá a và ký hiệu là [a]. Chứng minh rằng với mọi n nguyên dương ta luôn có. ( ) n n n n n =       + + + + + 1 1 ... 3 . 2 7 2 . 1 3 2 Câu III. Cho đường tròn (O) với đường kính AB = 2R. Trên đường thẳng tiếp xúc với đương tròn (O) tại A ta lấy điểm C sao cho góc 0 30 = ACB . Gọi H là giao điểm thứ hai của đường thăng BC với đường tròn (O). 1) Tính độ dài đương thẳng AC, BC và khoảng cách từ A đến đương thẳng BC theo R. 2) Với mỗi điểm M trên đoạn thẳng AC, đường thẳng BM cắt đường tròn (O tại điểm N (khác B). Chứng minh rằng bốn điểm C, M, N, H nằm trên cùng một đường tròn và tâm đường tròn đó luôn chạy trên một đường thẳng cố định khi M thay đổi trên đoạn thẳng AC. Câu IV. Với a,b là các số thực thoả mãn đẳng thức 4 9 ) 1 )( 1 ( = + + b a , hãy tìm giá trị nhỏ nhất của biểu thức 4 4 1 1 b a P + + + = . SỞ GIÁO DỤC VÀ ĐÀO TẠO HÀ NỘI ĐỀ CHÍNH THỨC Đề thi gồm 01 trang Đề số 25 KỲ THI VÀO LỚP 10 CHUYÊN KHOA HỌC TỰ NHIÊN NĂM HỌC 2010 – 2011 Môn: TOÁN (VÒNG 1) Thời gian làm bài: 120 phút (Không kể thời gian giao đề)
  • 21. Website: Liên hệ tài liệu word môn toán: TÀI LIỆU TOÁN HỌC Câu I. 1) Giải phương trình x 3 3x 1 4 + + + = 2) Giải hệ phương trình ( )( ) 2 2 5x 2y 2xy 26 3x 2x y x y 11.  + + =   + + − =   Câu II. 1) Tìm tất cả các số nguyên dương n để 2 n 391 + là số chính phương. 2) Giả sử x, y, z là những số thực dương thỏa mãn điều kiện x + y + z = 1. Chứng minh rằng: 2 2 xy z 2x 2y 1. 1 xy + + + ≥ + Câu III. Cho tam giác ABC có ba góc nhọn và M là điểm nằm trong tam giác. Kí hiệu H là hình chiếu của M trên cạnh BC và P, Q, E, F lần lượt là hình chiếu của H trên các đường thẳng MB, MC, AB, AC. Giả sử bốn điểm P, Q, E, F thẳng hàng. 1) Chứng minh rằng M là trực tâm của tam giác ABC. 2) Chứng minh rằng BEFC là tứ giác nội tiếp. Câu IV. Trong dãy số gồm 2010 số thực khác 0 được sắp xếp theo thứ tự 1 2 3 2010 a ,a ,a ,...,a ta đánh dấu tất cả các số dương và tất cả các số mà tổng của nó với một số liên tiếp liền ngay sau nó là một số dương. Chứng minh rằng nếu trong dãy số đã cho có ít nhất một số dương thì tổng của tất cả các số được đánh dấu là một số dương. SỞ GIÁO DỤC VÀ ĐÀO TẠO HÀ NỘI ĐỀ CHÍNH THỨC Đề thi gồm 01 trang Đề số 26 KỲ THI VÀO LỚP 10 CHUYÊN KHOA HỌC TỰ NHIÊN NĂM HỌC 2010 – 2011 Môn: TOÁN (VÒNG 2) Thời gian làm bài: 150 phút (Không kể thời gian giao đề) Website: Liên hệ tài liệu word môn toán: TÀI LIỆU TOÁN HỌC Câu I. 1) Giải phương trình 1 2 2 2 2 + − = + − x x x x 2) Giải hệ phương trình      + = + = + − 3 3 1 2 2 2 y y x xy y x Câu II. 1) Tìm chữ số tận cùng của số 2009 6 13 2009 6 13 + + 2) Với a, b là những chữ số thực dương, tìm giá trị nhỏ nhất của biểu thức ) 5 4 ( ) 5 4 ( a b b b a a b a P + + + + = Câu III. Cho hình thoi ABCD. Gọi H là giao điểm của hai đường chéo AC và BD. Biết rằng bán kính đường tròn ngoại tiếp tam giác ABC bằng a và bán kính đường tròn ngoại tiếp tam giác ABD bằng b. 1) Chứng minh rằng b a BH AH = 2) Tính diện tích hình thoi ABCD theo các bán kính a, b Câu IV. Với a, b, c là những số thực dương, chứng minh rằng 5 14 8 3 14 8 3 14 8 3 2 2 2 2 2 2 2 2 2 c b a ca a c c bc c b b ab b a a + + ≥ + + + + + + + + SỞ GIÁO DỤC VÀ ĐÀO TẠO HÀ NỘI ĐỀ CHÍNH THỨC Đề thi gồm 01 trang Đề số 27 KỲ THI VÀO LỚP 10 CHUYÊN KHOA HỌC TỰ NHIÊN NĂM HỌC 2009 – 2010 Môn: TOÁN (VÒNG 1) Thời gian làm bài: 120 phút (Không kể thời gian giao đề)
  • 22. Website: Liên hệ tài liệu word môn toán: TÀI LIỆU TOÁN HỌC Câu I. 1) Giải phương trình 35 36 84 1 6 35 14 2 + + + = + + + x x x x 2) Chứng minh rằng 1 4 ) 1 2 ( 4 1 2 ... 3 4 3 1 4 1 2 2 4 4 4 + = − + − + + + + + n n n n Với mọi n nguyên dương Câu II. 1) Tìm số nguyên dương n sao cho tất cả các số n + 1, n + 5, n + 7, n + 13, n + 17, n + 25, n + 37 đều là nguyên tố 2) Mỗi lần cho phép thay thế cặp số (a,b) thuộc tập hợp { } ) 8 , 78 ( ), 62 , 6 ( ), 32 , 4 ( ), 2 , 16 ( = M bằng cặp số (a + c, b + d) trong đó cặp số (c, d) cũng thuộc M. Hỏi sau một số hữu hạn lần thay thế ta có thể nhận được tập hợp các cặp số { } ) 912 , 2240 ( ), 2176 , 1056 ( ), 2104 , 844 ( ), 702 , 2018 ( 1 = M hay không? Câu III. Cho đường tròn (O) và (O’) cắt nhau tại hai điểm A và B. Trên đường thẳng AB ta lấy một điểm M bất kỳ sao cho điểm A nằm trong đoạn BM ( ) A M ≠ .Từ điểm M kẻ tới đường tròn (O’) các tiếp tuyến MC và MD (C và D là các tiếp điểm, C nằm ngoài (O)). Đường thẳng AC cắt lần thứ hai đường tròn (O) tại điểm P và đường thẳng AD cắt lần thứ hai đường tròn (O) tại Q. Đường thẳng CD cắt PQ tại K. 1) Chứng minh rằng hai tam giác BCD và BPQ đồng dạng 2) Chứng minh rằng khi M thay đổi thì đường tròn ngoại tiếp tam giác KCP luôn đi qua điểm cố định. Câu IV. Giả sử x,y,z là những số thực thoả mãn điều kiện 2 , , 0 ≤ ≤ z y x và x+ y + z = 3 Tìm giá trị nhỏ nhất và lớn nhất của biểu thức : ( ) ) 1 )( 1 ( 1 12 4 4 4 z y x z y x M − − − + + + = SỞ GIÁO DỤC VÀ ĐÀO TẠO HÀ NỘI ĐỀ CHÍNH THỨC Đề thi gồm 01 trang Đề số 28 KỲ THI VÀO LỚP 10 CHUYÊN KHOA HỌC TỰ NHIÊN NĂM HỌC 2009 – 2010 Môn: TOÁN (VÒNG 2) Thời gian làm bài: 150 phút (Không kể thời gian giao đề) Website: Liên hệ tài liệu word môn toán: TÀI LIỆU TOÁN HỌC Đề số 29- MÔN : TOÁN (Vòng 1) Thời gian làm bài: 150 phút (không kể thời gian phát đề) Câu 1. a) Giải hệ phương trình: 2 2 3 3 2 1 8 7  − =  − =  x y y x x y . b) Cho 0 1 ≤ ≤ x . Tìm giá trị lớn nhất của biểu thức: ( ) 2 1 = + − y x x . Câu 2. a) Tìm tất cả các số nguyên , x y thỏa mãn: 2 2 2 3 3 2 2 0 + + + + + = x y xy x y . b) Tìm số nguyên dương , , a b c sao cho ( )( )( ) 1 1 1 − − − ab bc ca abc là một số nguyên. Câu 3. a) Tìm nghiệm nguyên của phương trình: 2 2 2 2 8 10 + + = x y x y xy . b) Ký hiệu [ ] x là phần nguyên của số x (số nguyên lớn nhất không vượt quá x ). Chứng minh rằng với mọi số tự nhiên n , ta luôn có: ] [ ] [ 3 3 3 3 72 1 9 9 1 72 7   + = + + = +   n n n n Câu 4. Cho tam giác ABC nội tiếp đường tròn ( ) O . Giả sử các tiếp tuyến với đường tròn ( ) O tại B và C cắt nhau tại P nằm khác phía với A so với BC . Trên cung BC không chứa A lấy điểm ( ) , ≠ K K B C . Đường thẳng PK cắt ( ) O tại điểm thứ hai Q khác A . a) Chứng minh rằng các đường phân giác của các góc , ∠ ∠ KBQ KCQ đi qua cùng một điểm trên đường thẳng PQ. b) Giả sử đường thẳng AK đi qua trung điểm M của cạnh BC . Chứng minh rằng  AQ BC . Câu 5. Cho phương trình: 1 2 0 1 2 1 0 − − − + + +…+ + = n n n n n a x a x a x a x a thỏa mãn các hệ số 0 1 2 , , , , … n a a a a chỉ nhận một trong ba giá trị: 0 , hoặc 1 , hoặc 1 − và 0 0 ≠ a . Chứng minh rằng nếu 0 x là nghiệm của phương trình thì 0 2 < x ĐẠI HỌC QUỐC GIA HÀ NỘI TRƯỜNG ĐẠI HỌC KHOA HỌC TỰ NHIÊN ĐỀ THI TUYỂN SINH LỚP 10 HỆ THPT CHUYÊN NĂM 2008
  • 23. Website: Tài liệu sưu tầm và tổng hợp bản word đầy đủ liên hệ TÀI LIỆU TOÁN HỌC 32 Đề số 30- MÔN : TOÁN (Vòng 1) Thời gian làm bài: 150 phút (không kể thời gian phát đề) Câu 1. (3 điểm) Giải phương trình và hệ phương trình: a) 2 2 4 1 2 2 1 − + = − + + x x x x x b) ( ) 3 3 2 4  + =  + + + =  xy x y x y x y Câu 2. (3 điểm) a) Giả sử 1 2 , x x là hai nghiệm của phương trình 2 4 1 0. − + = x x Chứng minh rằng 5 5 1 2 + x x là một số nguyên. b) Cho a, b là các số nguyên dương thỏa mãn a + 1 và b + 2007 đều chia hết cho 6. Chứng minh rằng 4a + a + b chia hết cho 6. Câu 3. (3 điểm) Cho M là trung điểm của cung nhỏ AB của đường tròn tâm O (AB không phải là đường kính). C và D là 2 điểm phân biệt, thay đổi nằm giữa A và B. Các đường thẳng MC, MD cắt (O) tương ứng tại E, F khác M. a) Chứng minh các điểm C, D, E, F nằm trên một đường tròn. b) Gọi 1 O và 2 O lần lượt là tâm các đường tròn ngoại tiếp các tam giác ACE và BDF. Chứng minh rằng khi C và D thay đổi trên đoạn AB thì giao điểm của hai đường thẳng 1 AO và 2 BO là một điểm cố định. Câu 4. (1 điểm) Cho a, b, c là các số thực dương thỏa mãn abc = 1. Chứng minh rằng: ( ) ( ) ( ) 2 2 2 1 1 1 1 + + ≥ + + + + + + + + a b c a b c ab a bc b ca c ĐẠI HỌC QUỐC GIA HÀ NỘI TRƯỜNG ĐẠI HỌC KHOA HỌC TỰ NHIÊN ĐỀ THI TUYỂN SINH LỚP 10 HỆ THPT CHUYÊN NĂM 2007 Website: Tài liệu sưu tầm và tổng hợp bản word đầy đủ liên hệ TÀI LIỆU TOÁN HỌC 33 Đề số 31- MÔN : TOÁN (Vòng 2) Thời gian làm bài: 150 phút (không kể thời gian phát đề) Câu 1. a) Giải hệ phương trình: 2 2 4 5 2 4 7  + =  + + =  x y x y xy . b) Cho , , a b c là các số thực khác 0 . Tìm giá trị nhỏ nhất của biểu thức: 2 2 2 2 2 2 1 1 . +   = + +     b c P a b c a Câu 2. a) Tìm cặp số nguyên ( ) , x y thỏa mãn 2 2 5 17 2 + = + x y xy . b) Tìm tất cả các số nguyên tố p sao cho 4 2 + p là số nguyên tố. Câu 3. Cho hai đường thẳng 1 d và 2 d vuông góc tại O . Đường tròn ( ) 1 O tiếp xúc với 1 2 , d d lần lượt tại , A B . Đường tròn ( ) 2 O tiếp xúc với 1 2 , d d lần lượt tại , C D . a) Chứng minh rằng B là trực tâm tam giác ACD . b) Giả sử CB cắt ( ) 1 O tại , E AD cắt ( ) 2 O tại F . Chứng minh rằng ACEF là hình thang cân. Câu 4. Trong các tứ giác có ba cạnh đều bằng a cho trước. Tìm tứ giác diện tích lớn nhất. Câu 5 . Cho dãy số 0 1 , , , , … … n a a a được xác định bởi như sau: 0 0 = a và ( ) 1 2 3 1 + = + + ∀ ∈ n n n a a a n  . Chứng minh rằng: ( ) 2 1 (2 3) 2 3 4   = + − −   n n a . ĐẠI HỌC QUỐC GIA HÀ NỘI TRƯỜNG ĐẠI HỌC KHOA HỌC TỰ NHIÊN ĐỀ THI TUYỂN SINH LỚP 10 HỆ THPT CHUYÊN NĂM 2007
  • 24. Website: Tài liệu sưu tầm và tổng hợp bản word đầy đủ liên hệ TÀI LIỆU TOÁN HỌC 34 Đề số 32- MÔN : TOÁN (Vòng 1) Thời gian làm bài: 150 phút (không kể thời gian phát đề) Câu I (2,0 điểm) Giải hệ phương trình    = + + = + + + 4 ) xy 1 )( y x ( 4 y x xy x2 Câu II (2,0 điểm) Với những giá trị của x thỏa mãn điều kiện 2 1 x − ≥ , hãy tìm giá trị lớn nhất của biểu thức: x 2 3 x 2 2 x 5 x 2 ) x ( f 2 − + + + + = Câu III (2,0 điểm) Tìm số tự nhiên gồm bốn chữ số thỏa mãn đồng thời hai tính chất: (i) Khi chia số đó cho 100 ta được số dư là 6. (ii) Khi chia số đó cho 51 ta được số dư là 17. Câu IV (3,0 điểm) Cho hình vuông ABCD có cạnh α = AB . Trên các cạnh AB, BC, CD, DA lấy lần lượt các điểm M, N, P, Q sao cho: MN // AC, PQ // AC và 0 30 AMQ = ∠ . 1) Gọi A′ là điểm đối xứng với A qua đường thẳng MQ, C′ là điểm đối xứng với C qua đường thẳng NP. Giả sử đường thẳng A Q ′ cắt đoạn thẳng NP tại E, đường thẳng C P ′ cắt đoạn thẳng MQ tại F. chứng minh rằng năm điểm E, F, Q, D, P nằm trên cùng một đường tròn. 2) Biết AC = 3MN, tính diện tích của hình thang MNPQ theo a. Câu V (1,0 điểm) Chứng minh rằng với mỗi số dương a cho trước, đa thức 2 ax x ) x ( f 2 4 + + = luôn là tổng bình phương của hai đa thức bậc hai. ĐẠI HỌC QUỐC GIA HÀ NỘI TRƯỜNG ĐẠI HỌC KHOA HỌC TỰ NHIÊN ĐỀ THI TUYỂN SINH LỚP 10 HỆ THPT CHUYÊN NĂM 2006 Website: Tài liệu sưu tầm và tổng hợp bản word đầy đủ liên hệ TÀI LIỆU TOÁN HỌC 35 Đề số 33- MÔN : TOÁN (Vòng 2) Thời gian làm bài: 150 phút (không kể thời gian phát đề) Câu 1. Chứng minh rằng: 3 3 84 84 1 1 9 9 = + + − A là một số nguyên. Câu 2. Giải hệ phương trình: 2 2 2 2 4 2 3 5  − = − −  + =  x y x y x y . Câu 3. a) Tìm nghiệm nguyên của phương trình: 2 2 2 2 8 10 + + = x y x y xy . b) Ký hiệu [ ] x là phần nguyên của số x (số nguyên lớn nhất không vượt quá x ). Chứng minh rằng với mọi số tự nhiên n , ta luôn có: ] [ ] [ 3 3 3 3 72 1 9 9 1 72 7   + = + + = +   n n n n Câu 4. Cho tam giác ABC nội tiếp đường tròn ( ) O và I là điểm nằm trong tam giác ABC . Các đường thẳng , , AI BI CI cắt ( ) O lần lượt tại , , ′ ′ ′ A B C (khác , , A B C ). Dây cung ′ ′ B C cắt các cạnh , AB AC tương ứng tại các điểm , M N . Dây cung ′ ′ C A cắt các cạnh , AB BC tương ứng tại các điểm , Q P . Dây cung ′ ′ A B cắt các cạnh , BC CA tương ứng tại các điểm , F E . a) Giả sử , , = = = AM AN BP BQ CE CF xảy ra đồng thời. Chứng minh rằng I là tâm đường tròn nội tiếp tam giác ABC . b) Giả sử = = = = = AM AN BP BQ CE CF . Chứng minh rằng sáu điểm , , , , , M N P Q E F cùng thuộc một đường tròn. Câu 5. Chứng minh rằng đa giác lồi 2n cạnh ( ) , 2 ∈ ≥ n n  luôn có ít nhất n đường chéo không không song song với bất kỳ cạnh nào của đa giác đó. ĐẠI HỌC QUỐC GIA HÀ NỘI TRƯỜNG ĐẠI HỌC KHOA HỌC TỰ NHIÊN ĐỀ THI TUYỂN SINH LỚP 10 HỆ THPT CHUYÊN NĂM 2006
  • 25. Website: Tài liệu sưu tầm và tổng hợp bản word đầy đủ liên hệ TÀI LIỆU TOÁN HỌC 36 Đề số 34- MÔN : TOÁN (Vòng 1) Thời gian làm bài: 150 phút (không kể thời gian phát đề) Câu 1: Giải hệ phương trình    = + = + + 2 y x 3 xy y x 2 2 Câu 2: Giải phương trình 11 x 2 3 2 3 x 4 x = − + + + Câu 3: Tìm nghiệm nguyên của phương trình 1740 ) y x ( 51 xy 34 y 17 x 2 2 = + + + + Câu 4: Cho hai đường tròn (O), ) O ( ′ nằm ngoài nhau có tâm tương ứng là O và O′. Một tiếp tuyến chung ngoài của hai đường tròn tiếp xúc với (O) tại A và ) O ( ′ tại B. Một tiếp tuyến chung trong của hai đường tròn cắt AB tại I, tiếp xúc với (O) tại C và ) O ( ′ tại D. Biết rằng C nằm giữa I và D. 1) Hai đường thẳng OC, B O′ cắt nhau tại M. Chứng minh rằng OM > M O′ . 2) Ký hiệu (S) là đường tròn đi qua A, C, B và ) S ( ′ là đường tròn đi qua A, D, B. Đường thẳng CD cắt (S) tại E khác C và cắt ) S ( ′ tại F khác D. Chứng minh rằng AF vuông góc với BE Câu 5: Giả sử x, y, z là các số dương thay đổi và thỏa mãn điều kiện: 2 2 2 2 z 3 y z x z xy = + + . Hãy tìm giá trị lớn nhất của biểu thức: ( ) 4 4 4 4 y x z 1 z P + + = ĐẠI HỌC QUỐC GIA HÀ NỘI TRƯỜNG ĐẠI HỌC KHOA HỌC TỰ NHIÊN ĐỀ THI TUYỂN SINH LỚP 10 HỆ THPT CHUYÊN NĂM 2005 Website: Tài liệu sưu tầm và tổng hợp bản word đầy đủ liên hệ TÀI LIỆU TOÁN HỌC 37 ĐẠI HỌC QUỐC GIA HÀ NỘI TRƯỜNG ĐẠI HỌC KHOA HỌC TỰ NHIÊN ĐỀ THI TUYỂN SINH LỚP 10 HỆ THPT CHUYÊN NĂM 2005 Đề số 35- MÔN : TOÁN (Vòng 2) Thời gian làm bài: 150 phút (không kể thời gian phát đề) Bài 1: Giải phương trình 2 2 2 4 2 x x x − + + + − = Bài 2: Giải hệ phương trình 3 3 2 4 4 1 4 4 x y xy x y x y  + − =  + = +  Bài 3: Giả sử x, y là số không âm thỏa mãn 2 2 1 x y + = a) Chứng minh rằng 1 2 x y ≤ + ≤ b)Tìm giá trị lớn nhất và nhỏ nhất của biểu thức 1 2 1 2 P x y = + + + Bài 4: Cho hình vuông ABCD và điểm P nằm trong tam giác ABC a) Giả sử  135o BPC = . Chứng minh rằng: 2 2 2 2PB PC PA + = b) Các đường thẳng AP và CP cắt các cạnh BC và BA tại M, N. Gọi Q là điểm đối xứng với B qua trung điểm của đoạn MN. Chứng minh rằng khi P thay đổi trong tam giác ABC, đường thẳng PQ luôn đi qua D. Bài 5: a) Cho đa giác đều (H) có 14 đỉnh. Chứng minh rằng trong 6 đỉnh bất kỳ của (H) luôn có 4 đỉnh là các đỉnh của 1 hình thang b) Có bao nhiêu phân số tối giản 1 m n > (m, n là các số nguyên dương ) thỏa mãn . 13860. m n =
  • 26. Website: Tài liệu sưu tầm và tổng hợp bản word đầy đủ liên hệ TÀI LIỆU TOÁN HỌC 38 ĐẠI HỌC QUỐC GIA HÀ NỘI TRƯỜNG ĐẠI HỌC KHOA HỌC TỰ NHIÊN ĐỀ THI TUYỂN SINH LỚP 10 HỆ THPT CHUYÊN NĂM 2004 Đề số 36- MÔN : TOÁN (Vòng 1) Thời gian làm bài: 150 phút (không kể thời gian phát đề) Câu 1: 1) Giải phương trình 1 x 1 1 x 1 x 2 − + = − + + 2) Tìm nghiệm nguyên của hệ    = − + + = − + − − 8 y x y x 7 x 2 y 2 xy x y 2 3 3 2 2 Câu 2: Cho các số thực dương a và b thỏa mãn 100 100 101 101 102 102 + = + = + a b a b a b Hãy tính giá trị của biểu thức 2004 2004 b a P + = Câu 3: Cho tam giác ABC có AB = 3 cm, BC = 4 cm, CA = 5 cm. Đường cao, đường phân giác, đường trung tuyến của tam giác kẻ từ đỉnh B chia tam giác thành bốn phần. Hãy tính diện tích mỗi phần. Câu 4: Cho tứ giác ABCD nội tiếp trong đường tròn có hai đường chéo AC và BD vuông góc với nhau tại H (H không trùng với tâm của đường tròn). Gọi M và N lần lượt là chân các đường vuông góc hạ từ H xuống các đường thẳng AB và BC; P và Q lần lượt là giao điểm của đường thẳng MH và NH với các đường thẳng CD và DA. Chứng minh rằng đường thẳng PQ song song với đường thẳng AC và bốn điểm M, N, P, Q nằm trên cùng một đường tròn. Câu 5: Tìm giá trị nhỏ nhất của biểu thức ( ) ( )2 2 2 16 16 2 10 2 10 y x 1 y x 4 1 x y y x 2 1 Q + − + +         + = Website: Tài liệu sưu tầm và tổng hợp bản word đầy đủ liên hệ TÀI LIỆU TOÁN HỌC 39 ĐẠI HỌC QUỐC GIA HÀ NỘI TRƯỜNG ĐẠI HỌC KHOA HỌC TỰ NHIÊN ĐỀ THI TUYỂN SINH LỚP 10 HỆ THPT CHUYÊN NĂM 2004 Đề số 37- MÔN : TOÁN (Vòng 2) Thời gian làm bài: 150 phút (không kể thời gian phát đề) Bài 1. Giải phương trình 3 1 2 x x + + − = Bài 2. Giải hệ phương trình ( ) ( ) 2 2 2 2 ( ) 15( ) 3 x y x y x y x y + + = − − = Bài 3. Tìm giá trị nhỏ nhất của biểu thức ( ) ( ) 3 3 2 2 ( 1)( 1) x y x y P x y + − + = − − trong đó, x, y là những số thực lớn hơn 1 . Bài 4. Cho hình vuông ABCD và điểm M nằm trong hình vuông. 1. Tìm tất cả các vị trí của điểm M sao cho     MAB MBC MCD MDA = = = 2. Xét điểm M nằm trên đường chéo AC. Gọi N là chân đường vuông góc hạ từ điểm M xuống A B và O là trung điểm của đoạn AM. Chứng minh rầng tỷ số OB CN có giá trị không đổi khi M di chuyển trên đường chéo A C. 3. Với giả thiết M nằm trên đường chéo AC, xét các đường tròn ( ) 1 S và ( ) 2 S có đường kính tương ứng là AM và CN. Hai tiếp tuyến chung của ( ) 1 S và ( ) 2 S tiếp xúc với ( ) 2 S tại P và Q . Chứng minh rằng đường thẳng PQ tiếp xúc với ( ) 1 S Bài 5. Với số thực a , ta định nghĩa phần nguyên của số a là số nguyên lớn nhất không vượt quá a và ký hiệu là [a]. Dãy các số 0 1 2 , , , , , n x x x x … …. được xác định bởi công thức 1 2 2 n n n x     + = −         Hỏi trong 200 số { } 0 1 199 , , , x x x … có bao nhiêu số khác 0? (Cho biết 1,41 2 1,42 < < ).
  • 27. Website: Tài liệu sưu tầm và tổng hợp bản word đầy đủ liên hệ TÀI LIỆU TOÁN HỌC 40 ĐẠI HỌC QUỐC GIA HÀ NỘI TRƯỜNG ĐẠI HỌC KHOA HỌC TỰ NHIÊN ĐỀ THI TUYỂN SINH LỚP 10 HỆ THPT CHUYÊN NĂM 2003 Đề số 38- MÔN : TOÁN (Vòng 1) Thời gian làm bài: 150 phút (không kể thời gian phát đề) Câu 1 (2 điểm): Giải phương trình: 2 ( 5 2)(1 7 10) 3 x x x x + − + + + + = Câu 2 (2 điểm): Giải hệ phương trình:      = + = + 7 xy 6 y 5 y x 3 x 2 2 3 2 3 Câu 3 (2 điểm): Tìm các số nguyên x, y thỏa mãn đẳng thức: xy y 2 x 1 y x x y 2 2 2 2 + + = + + + Câu 4 (3 điểm) Cho nửa đường tròn (O) đường kính AB = 2R (R là một độ dài cho trước). M, N là hai điểm trên nửa đường tròn (O) sao cho M thuộc cung AN và tổng các khoảng cách từ A, B đến đường thẳng MN bằng 3 . 1. Tính độ dài đoạn MN theo R. 2. Gọi giao điểm của hai dây AN và BM là I, giao điểm của các đường thẳng AM và BN là K. Chứng minh rằng bốn điểm M, N, I, K cùng nằm trên một đường tròn. Tính bán kính của đường tròn đó theo R. 3. Tìm giá trị lớn nhất của diện tích tam giác KAB theo R khi M, N thay đổi nhưng vẫn thỏa mãn giả thiết của bài toán. Câu 5 (1 điểm): x, y, z là các số thực thỏa mãn điều kiện: 6 zx yz xy z y x = + + + + + Chứng minh rằng: 3 z y x 2 2 2 ≥ + + Website: Tài liệu sưu tầm và tổng hợp bản word đầy đủ liên hệ TÀI LIỆU TOÁN HỌC 41 ĐẠI HỌC QUỐC GIA HÀ NỘI TRƯỜNG ĐẠI HỌC KHOA HỌC TỰ NHIÊN ĐỀ THI TUYỂN SINH LỚP 10 HỆ THPT CHUYÊN NĂM 2003 Đề số 39- MÔN : TOÁN (Vòng 2) Thời gian làm bài: 150 phút (không kể thời gian phát đề) Bài 1. Cho phương trình 4 2 2 4 0 x mx + + = Tìm giá trị của tham số m để phương trình có 4 nghiệm phân biệt 1 2 3 4 , , , x x x x thoả mãn 4 4 4 4 1 2 3 4 32 x x x x + + + = Bài 2. Giải hệ phương trình 2 2 2 2 2 5 2 0 4 0 x xy y x y x y x y  + − − + + =  + + + − =  Bài 3. Tìm các số nguyên x, y thoả mãn đẳng thức 2 2 2 2 x xy y x y + + = Bài 4. Cho đường tròn tâm O nội tiếp tam giác ABC tiếp xúc với các cạnh BC, CA, AB tương ứng tại các điểm D, E, F. Đường tròn tâm O′ bàng tiếp trong góc  BAC của tam giác ABC tiêp xúc với cạnh BC và phần kéo dài của các cạnh AB, AC tương ứng tại các điểm $P, M, N$. 1. Chứng minh rằng: BP CD = . 2. Trên đường thẳng MN ta lấy các điểm I và K sao cho / / , / / CK AB BI AC . Chứng minh rằng các tứ giác BICE và BKCF là các hình binh hành. 3. Gọi ( ) S là đường tròn đi qua ba điểm I, K, P. Chứng minh rằng ( ) S tiếp xúc với các đường thẳng BC, BI, CK. Bài 5. Số thực x thay đổi và thoả mãn điều kiện 2 2 (3 ) 5 x x + −  . Tìm giá trị nhỏ nhất của biểu thức 4 4 2 2 (3 ) 6 (3 ) p x x x x = + − + −